Tổng hợp đề chính thức và đề xuất kì thi hsg duyên hải và đồng bằng bắc bộ môn Hóa học 11 năm 2017

Page 1

ĐỀ THI HSG DUYÊN HẢI BẮC BỘ MÔN HÓA HỌC

vectorstock.com/3687784

Ths Nguyễn Thanh Tú eBook Collection DẠY KÈM QUY NHƠN OLYMPIAD PHÁT TRIỂN NỘI DUNG

Tổng hợp đề chính thức và đề xuất kì thi hsg khu vực duyên hải và đồng bằng bắc bộ môn Hóa học khối 11 năm 2017 có đáp án WORD VERSION | 2021 EDITION ORDER NOW / CHUYỂN GIAO QUA EMAIL TAILIEUCHUANTHAMKHAO@GMAIL.COM

Tài liệu chuẩn tham khảo Phát triển kênh bởi Ths Nguyễn Thanh Tú Đơn vị tài trợ / phát hành / chia sẻ học thuật : Nguyen Thanh Tu Group Hỗ trợ trực tuyến Fb www.facebook.com/DayKemQuyNhon Mobi/Zalo 0905779594


ĐỀ THI MÔN HÓA HỌC LỚP 11

HỘI CÁC TRƯỜNG THPT CHUYÊN KHU VỰC DUYÊN HÀI VÀ ĐỒNG BẰNG BẮC BỘ TRƯỜNG THPT CHUYÊN BIÊN HÒA, T. HÀ NAM

Thời gian làm bài:180 phút (Đề thi gồm có 04 trang)

ĐỀ THI ĐỀ XUẤT Câu 1: Tốc độ phản ứng – Cân bằng hóa học 1. Phân hủy nhiệt của 1,3-diphenylpropane tạo ra các sản phẩm chủ yếu là toluene và styrene cùng với các sản phảm phụ ethylbenzene và những hydrocarbon khác. Người ta đề nghị cơ chế sau đây cho phản ứng này: k1

(1) (chậm)

PhCH 2CH 2CH 2 Ph → PhCH 2 * + PhCH 2CH 2 * S

1

2

k2

PhCH 2CH 2 * + PhCH 2CH 2CH 2 Ph → PhCH 2CH 3 + Ph CH CH 2CH 2 Ph

(2)

3

k3

PhCH 2 * + PhCH 2CH 2CH 2 Ph → PhCH3 + Ph CH CH 2CH 2Ph

(3)

P1

k4

(4)

Ph CHCH 2CH 2 Ph → PhCH=CH 2 + PhCH 2 * P2

a. Áp dụng sự gần đúng nồng độ dừng đối với gốc 2 hãy rút ra định luật tốc độ cho phản ứng tạo thành ethylbenzene. b. Tìm tỉ số các nồng độ dừng của các gốc 1 và 3? Ngoài ra, hai gốc tự do có thể kết hợp với nhau với hằng số tốc độ kR được xem là không đổi với các gốc khác nhau:. kR

R1* + R 2 * → R1R 2 c. Vì sao trong các phương trình nồng độ dừng của 1 và 2 có thể bỏ qua tốc độ của các phản ứng này? d. Tìm định luật tốc độ của sự hình thành toluene, xác định bậc phản ứng. Biểu diễn năng lượng hoạt động hóa của phản ứng chung qua các năng lượng hoạt động hóa của các phản ứng sơ cấp. 2. Iot là một nguyên tố vi lượng quan trọng trong cuộc sống và là nguyên tố nặng nhất mà các cơ thể sống cần được cung cấp mỗi ngày. Ở nhiệt độ cao cân bằng giữa I2(k) và I(k) được thiết lập. Bảng sau ghi lại áp suất đầu của I2(k) và áp suất chung khi hệ đạt đến cân bằng ở nhiệt độ khảo sát. T (K) 1073 1173 P(I2) (atm) 0.0631 0.0684 Pchung (atm) 0.0750 0.0918 a. Tính ∆H°, ∆G°và ∆S°ở 1100 K. (Cho rằng ∆H° và ∆S° đều không phụ thuộc nhiệt độ trong khoảng nhiệt độ khảo sát.)

1


b. Tính phần mol của I(k) trong hỗn hợp cân bằng trong trường hợp trị số Kp lúc này bằng một nửa áp suất chung. c. Biết I2(k) và I(k) đều là khí lý tưởng. Tính năng lượng phân ly liên kết của I2 ở 298 K. Câu 2: Cân bằng trong dung dịch điện li Dung dịch X gồm Na2S 0,010M, KI 0,060M, Na2SO4 0,050M. 1. Tính pH của dung dịch X. 2. Thêm dần Pb(NO3)2 vào dung dịch X cho đến nồng độ 0,090M thì thu được kết tủa A và dung dịch B. a) Cho biết thành phần hoá học của kết tủa A và dung dịch B. b) Tính nồng độ các ion trong dung dịch B (không kể sự thuỷ phân của các ion, coi thể tích dung dịch không thay đổi khi thêm Pb(NO3)2). Câu 3: Điện hóa học Cho các giá trị thế khử Eo(Ag+/Ag) = 0,80 V; Eo(AgI/Ag) = –0,15 V; Eo(Au3+/Au+) = 1,26 V; Eo(Fe2+/Fe) = –0,037 V; Eo(Fe3+/Fe) = 0,44 V. 1. a. Lập sơ đồ pin xác định tích số tan của AgI. Viết phương trình mỗi điện cực và phản ứng xảy ra trong pin. b. Tính độ tan (S) của AgI trong nước ở 25oC. 2. Thiết lập pin có sự oxi hóa Fe2+ thành Fe3+, khử Au3+ về Au+. Tính Kcb và ∆Eopin. Câu 4: Nitơ – Photpho, Cacbon – Silic và hợp chất 1. a. Nguyên tố phi kim X phản ứng với Cl2 cho chất lỏng không màu A (tonóng chảy = -94 oC). Chất A tác dụng với Cl2 dư trong dung môi CCl4 khan cho B (chất rắn màu trắng, tothăng hoa = 160 oC). Cho biết khối lượng phân tử của B bằng 1,516 lần khối lượng phân tử của A. Hãy xác định A, B và vẽ cấu trúc phân tử của chúng. b. Một hợp chất dị vòng của X với cấu trúc phẳng do J. Liebig và F. Wohler tổng hợp từ năm 1834 được tạo thành từ NH4Cl với một chất pentacloro của X có công thức phân tử là (NXCl2)3, sản phẩm phụ của phản ứng này là một khí dễ tan trong nước và phản ứng như một axit mạnh. Hợp chất vô cơ này có tính chất khác thường khi bị đun nóng: nó sôi ở 256oC khi bị đun nóng nhanh, nếu đun nóng chậm nó bắt đầu nóng chảy ở 250oC, làm nguội nhanh chất lỏng này ta được một chất tương tự cao su. Hãy viết phương trình phản ứng, xác định cấu tạo của chất (NXCl2)3 và giải thích tính chất đặc biệt này 2. Năm 1851, S. Cannizzarro và F. Close đã tổng hợp thành công một chất mới B. Cuối thế kỉ 19, một phương pháp tổng hợp B trong công nghiệp được ứng dụng và phát triển. Các chất D và F được sử dụng như nguyên liệu đầu:

Chất Tonc (oC) Tos (oC) Tỉ khối Dung môi hòa tan A -6 12,7 1,1860 CCl4, C6H6... B 44 140 1,0729 H2O, C2H5OH, (C2H5)2O... C 132,7 1,3350 H2O, C2H5OH Xác định A đến F, viết phương trình phản ứng Câu 5: Phức chất, trắc quang 1. Ion phức bis(terpyridyl)coban(II) tồn tại một phần ở trạng thái spin cao, một phần ở trạng thái spin thấp phụ thuộc vào các ion liên kết trực tiếp với nguyên tử trung tâm ClO4-/Cl-/NCS-/Br-. a. Cho biết ba dạng hình học có thể có của phức b. Dựa vào thuyết trường phối tử hãy vẽ giản đồ obitan cho các trường hợp phức spin cao và thấp c. Tính momen từ (M.B) của các phức trên

2


d. Xét các phức sau: [Co(CN)6]3-, [Co(CO3)2(NH3)2]-, [Co(CO3)3]3- and [Co(NO2)6]3-. Màu của các ion phức này sẽ là: xanh, vàng, cam và da trời (không nhất thiết là phải ở cùng dạng với các phức trên). Hãy cho biết tên của từng phức và xác định màu của chúng: 2. Hệ số hấp thụ phân tử gam của anilin (C6H5NH2) ở bước sóng 280 nm là 1,48.103 mol.l-1.cm-1. Dạng proton hóa của anilin C6H5NH3+ không hấp thụ ở bước sóng này. Độ truyền quang của anilin 2,10-4 M trong cuvet l =20 mm cũng ở bước sóng này là 0,92. Tính pH của dung dịch chứa C6H5NH3+ Cho biết: Ka (C6H5NH3+) = 10-4,8 Câu 6: (2 điểm) Quan hệ giữa hiệu ứng cấu trúc và tính chất Cho các công thức cấu tạo sau:

1. Hãy viết công thức các đồng phân lập thể ứng với cấu tạo A 2. Ứng với công thức cấu tạo B có bao nhiêu đồng phân lập thể, vì sao? Dùng các kí hiệu thích hợp để chỉ rõ cấu hình mỗi đồng phân đó 3. Hãy viết cơ chế phản ứng để giải thích vì sao C và D khi tương tác với dung dịch NaOH thì đều tạo natri 3-metyl-4-nitrobenzoat 4. Chỉ rõ trạng thái lai hóa của từng nguyên tử N trong cấu tạo E và ghi giá trị pKa (ở 25oC): 1,8; 6,0; 9,2 vào từng trung tâm axit trong công thức tương ứng với E và giải thích Câu 7: (2 điểm) Hiđrocacbon Một số hiđrocacbon X có công thức phân tử C10H16 và có các tính chất sau: tác dụng với H2 (dư)/Ni ở 120oC cho C10H22; tác dụng với Br2/CCl4 cho C10H16Br6; 1 mol X tác dụng với ozon rồi thủy phân khử (nhờ Zn/HCl) hoặc thủy phân oxi hóa (nhờ H2O2) đều cho 2 mol một sản phẩm hữu cơ duy nhất Y có công thức phân tử là C5H8O; không thể tách X và Y thành các đối quang được. Hãy xác định công thức lập thể có thể có của X và viết phương trình hóa học xảy ra với một trong số các công thức tìm được của X Câu 8: (2 điểm) Xác định cấu trúc + đồng phân lập thể + danh pháp Hợp chất A có công thức phân tử CHON, ở thể khí độ dài liên kết CN bằng 121 pm, CO bằng 117 pm. A tan trong nước tạo thành dung dịch axit với Ka = 1,2.10-4. Trong dung dịch đặc A tự biến đổi thành X có vòng 6 cạnh với độ dài liên kết CN bằng 135 pm. Cho biết độ dài liên kết trung bình ở các hợp chất như sau Liên kết C-C C-N C-O C=N C=O C≡N Cacbon oxit Độ dài, pm 154 147 143 130 123 116 112 1. Xác định công thức cấu tạo của A ở thể khí 2. Dùng công thức cấu tạo viết phương trình phản ứng điện li của A trong dung dịch nước và giải thích vì sao lực axit của nó lớn hơn của axit axetic 3. Viết phương trình phản ứng tạo thành và công thức cấu tạo của X Câu 9: (2 điểm) Cơ chế phản ứng Đề xuất cơ chế để giải thích quá trình tạo ra sản phẩm của các phản ứng sau 1.

2.

3


3.

4.

Câu 10: (2 điểm) Tổng hợp các hợp chất hữu cơ Erythronolit B là chất đầu của quá trình sinh tổng hợp của chất kháng sinh erythromixin. Để tổng hợp erythronolit B người ta tiến hành phân tích tổng hợp lùi và thấy rằng cần xuất phát từ tiền chất A’. Tiền chất A’ có công thức

Rất lý thú là A’ được tổng hợp từ nguyên liệu đớn giản là 2,4,6-trimetylphenol theo sơ đồ sau: OH NaOMe, PhMe

H2O2, NaOH,

Br

F

Br2, KBr H2O

1. LiOH, H2O 2. CrO3, H2SO4 axeton

BH3,THF,0oC

A

G

N

n-Bu3SnH, AIBN toluen,

75oC

MeCOOOH EtOAc, 55oC

O

H

CrO3, H2SO4

B

0oC

axeton

Al/Hg, THF/H2O

I

o

0C

N

S

S

N

C

Br2, KBr

D

H2O

H2, Ni-Ra o

DMF, -20 C

K

BzCl Py

KOH, H2O

[E]

THF

L

LDA, THF, -78oC MeI, HMPA

M

A'

Ph3P, THF

----------------HẾT---------------Họ và tên thí sinh………………………………………SBD……………………… Họ và tên giám thị số 1……………………………………………………………… Họ và tên giám thị số 2 ……………………………………………………………. 4


5


ĐỀ THI MÔN HÓA HỌC LỚP 11

HỘI CÁC TRƯỜNG THPT CHUYÊN KHU VỰC DUYÊN HÀI VÀ ĐỒNG BẰNG BẮC BỘ TRƯỜNG THPT CHUYÊN BIÊN HÒA, T. HÀ NAM

Thời gian làm bài:180 phút (Đề thi gồm có 04 trang)

ĐỀ THI ĐỀ XUẤT Câu 1: Tốc độ phản ứng – Cân bằng hóa học 1. Phân hủy nhiệt của 1,3-diphenylpropane tạo ra các sản phẩm chủ yếu là toluene và styrene cùng với các sản phảm phụ ethylbenzene và những hydrocarbon khác. Người ta đề nghị cơ chế sau đây cho phản ứng này.: k1

(1) (chậm)

PhCH 2CH 2CH 2 Ph → PhCH 2 * + PhCH 2CH 2 * S

1

2

k2

PhCH 2CH 2 * + PhCH 2CH 2CH 2 Ph → PhCH 2CH 3 + Ph CH CH 2CH 2 Ph

(2)

3

k3

PhCH 2 * + PhCH 2CH 2CH 2 Ph → PhCH3 + Ph CH CH 2CH 2Ph

(3)

P1

k4

Ph CHCH 2CH 2 Ph → PhCH=CH 2 + PhCH 2 *

(4)

P2

a. Áp dụng sự gần đúng nồng độ dừng đối với gốc 2 hãy rút ra định luật tốc độ cho phản ứng tạo thành ethylbenzene. b. Tìm tỉ số các nồng độ dừng của các gốc 1 và 3? Ngoài ra, hai gốc tự do có thể kết hợp với nhau với hằng số tốc độ kR được xem là không đổi với các gốc khác nhau:. kR

R1* + R 2 * → R1R 2 c. Vì sao trong các phương trình nồng độ dừng của 1 và 2 có thể bỏ qua tốc độ của các phản ứng này? d. Tìm định luật tốc độ của sự hình thành toluene, xác định bậc phản ứng. Biểu diễn năng lượng hoạt động hóa của phản ứng chung qua các năng lượng hoạt động hóa của các phản ứng sơ cấp. 2. Iot là một nguyên tố vi lượng quan trọng trong cuộc sống và là nguyên tố nặng nhất mà các cơ thể sống cần được cung cấp mỗi ngày. Ở nhiệt độ cao cân bằng giữa I2(k) và I(k) được thiết lập. Bảng sau ghi lại áp suất đầu của I2(k) và áp suất chung khi hệ đạt đến cân bằng ở nhiệt độ khảo sát. T (K) 1073 1173 P(I2) (atm) 0.0631 0.0684 Pchung (atm) 0.0750 0.0918 a. Tính ∆H°, ∆G°và ∆S°ở 1100 K. (Cho rằng ∆H° và ∆S° đều không phụ thuộc nhiệt độ trong khoảng nhiệt độ khảo sát.)


b. Tính phần mol của I(k) trong hỗn hợp cân bằng trong trường hợp trị số Kp lúc này bằng một nửa áp suất chung. c. Biết I2(k) và I(k) đều là khí lý tưởng. Tính năng lượng phân ly liên kết của I2 ở 298 K. Câu 2: Cân bằng trong dung dịch điện li Dung dịch X gồm Na2S 0,010M, KI 0,060M, Na2SO4 0,050M. 1. Tính pH của dung dịch X. 2. Thêm dần Pb(NO3)2 vào dung dịch X cho đến nồng độ 0,090M thì thu được kết tủa A và dung dịch B. a) Cho biết thành phần hoá học của kết tủa A và dung dịch B. b) Tính nồng độ các ion trong dung dịch B (không kể sự thuỷ phân của các ion, coi thể tích dung dịch không thay đổi khi thêm Pb(NO3)2). Câu 3: Điện hóa học Cho các giá trị thế khử Eo(Ag+/Ag) = 0,80 V; Eo(AgI/Ag) = –0,15 V; Eo(Au3+/Au+) = 1,26 V; Eo(Fe2+/Fe) = –0,037 V; Eo(Fe3+/Fe) = 0,44 V. 1. a. Lập sơ đồ pin xác định tích số tan của AgI. Viết phương trình mỗi điện cực và phản ứng xảy ra trong pin. b. Tính độ tan (S) của AgI trong nước ở 25oC. 2. Thiết lập pin có sự oxi hóa Fe2+ thành Fe3+, khử Au3+ về Au+. Tính Kcb và ∆Eopin. Câu 4: Nitơ – Photpho, Cacbon – Silic và hợp chất 1. a. Nguyên tố phi kim X phản ứng với Cl2 cho chất lỏng không màu A (tonóng chảy = -94 oC). Chất A tác dụng với Cl2 dư trong dung môi CCl4 khan cho B (chất rắn màu trắng, tothăng hoa = 160 oC). Cho biết khối lượng phân tử của B bằng 1,516 lần khối lượng phân tử của A. Hãy xác định A, B và vẽ cấu trúc phân tử của chúng. b. Một hợp chất dị vòng của X với cấu trúc phẳng do J. Liebig và F. Wohler tổng hợp từ năm 1834 được tạo thành từ NH4Cl với một chất pentacloro của X có công thức phân tử là (NXCl2)3, sản phẩm phụ của phản ứng này là một khí dễ tan trong nước và phản ứng như một axit mạnh. Hợp chất vô cơ này có tính chất khác thường khi bị đun nóng: nó sôi ở 256oC khi bị đun nóng nhanh, nếu đun nóng chậm nó bắt đầu nóng chảy ở 250oC, làm nguội nhanh chất lỏng này ta được một chất tương tự cao su. Hãy viết phương trình phản ứng, xác định cấu tạo của chất (NXCl2)3 và giải thích tính chất đặc biệt này 2. Năm 1851, S. Cannizzarro và F. Close đã tổng hợp thành công một chất mới B. Cuối thế kỉ 19, một phương pháp tổng hợp B trong công nghiệp được ứng dụng và phát triển. Các chất D và F được sử dụng như nguyên liệu đầu:

Chất Tonc (oC) Tos (oC) Tỉ khối Dung môi hòa tan A -6 12,7 1,1860 CCl4, C6H6... B 44 140 1,0729 H2O, C2H5OH, (C2H5)2O... C 132,7 1,3350 H2O, C2H5OH Xác định A đến F, viết phương trình phản ứng Câu 5: Phức chất, trắc quang 1. Ion phức bis(terpyridyl)coban(II) tồn tại một phần ở trạng thái spin cao, một phần ở trạng thái spin thấp phụ thuộc vào các ion liên kết trực tiếp với nguyên tử trung tâm ClO4-/Cl-/NCS-/Br-. a. Cho biết ba dạng hình học có thể có của phức b. Dựa vào thuyết trường phối tử hãy vẽ giản đồ obitan cho các trường hợp phức spin cao và thấp c. Tính momen từ (M.B) của các phức trên d. Xét các phức sau: [Co(CN)6]3-, [Co(CO3)2(NH3)2]-, [Co(CO3)3]3- and [Co(NO2)6]3-. Màu của các ion phức này sẽ là: xanh, vàng, cam và da trời (không nhất thiết là phải ở cùng dạng với các phức trên). Hãy cho biết tên của từng phức và xác định màu của chúng:


2. Hệ số hấp thụ phân tử gam của anilin (C6H5NH2) ở bước sóng 280 nm là 1,48.103 mol.l-1.cm-1. Dạng proton hóa của anilin C6H5NH3+ không hấp thụ ở bước sóng này. Độ truyền quang của anilin 2,10-4 M trong cuvet l =20 mm cũng ở bước sóng này là 0,92. Tính pH của dung dịch chứa C6H5NH3+ Cho biết: Ka (C6H5NH3+) = 10-4,8 Câu 6: (2 điểm) Quan hệ giữa hiệu ứng cấu trúc và tính chất Cho các công thức cấu tạo sau:

1. Hãy viết công thức các đồng phân lập thể ứng với cấu tạo A 2. Ứng với công thức cấu tạo B có bao nhiêu đồng phân lập thể, vì sao? Dùng các kí hiệu thích hợp để chỉ rõ cấu hình mỗi đồng phân đó 3. Hãy viết cơ chế phản ứng để giải thích vì sao C và D khi tương tác với dung dịch NaOH thì đều tạo natri 3-metyl-4-nitrobenzoat 4. Chỉ rõ trạng thái lai hóa của từng nguyên tử N trong cấu tạo E và ghi giá trị pKa (ở 25oC): 1,8; 6,0; 9,2 vào từng trung tâm axit trong công thức tương ứng với E và giải thích Câu 7: (2 điểm) Hiđrocacbon Một số hiđrocacbon X có công thức phân tử C10H16 và có các tính chất sau: tác dụng với H2 (dư)/Ni ở 120oC cho C10H22; tác dụng với Br2/CCl4 cho C10H16Br6; 1 mol X tác dụng với ozon rồi thủy phân khử (nhờ Zn/HCl) hoặc thủy phân oxi hóa (nhờ H2O2) đều cho 2 mol một sản phẩm hữu cơ duy nhất Y có công thức phân tử là C5H8O; không thể tách X và Y thành các đối quang được. Hãy xác định công thức lập thể có thể có của X và viết phương trình hóa học xảy ra với một trong số các công thức tìm được của X Câu 8: (2 điểm) Xác định cấu trúc + đồng phân lập thể + danh pháp Hợp chất A có công thức phân tử CHON, ở thể khí độ dài liên kết CN bằng 121 pm, CO bằng 117 pm. A tan trong nước tạo thành dung dịch axit với Ka = 1,2.10-4. Trong dung dịch đặc A tự biến đổi thành X có vòng 6 cạnh với độ dài liên kết CN bằng 135 pm. Cho biết độ dài liên kết trung bình ở các hợp chất như sau Liên kết C-C C-N C-O C=N C=O C≡N Cacbon oxit Độ dài, pm 154 147 143 130 123 116 112 1. Xác định công thức cấu tạo của A ở thể khí 2. Dùng công thức cấu tạo viết phương trình phản ứng điện li của A trong dung dịch nước và giải thích vì sao lực axit của nó lớn hơn của axit axetic 3. Viết phương trình phản ứng tạo thành và công thức cấu tạo của X Câu 9: (2 điểm) Cơ chế phản ứng Đề xuất cơ chế để giải thích quá trình tạo ra sản phẩm của các phản ứng sau 1.

2.

3.


4.

Câu 10: (2 điểm) Tổng hợp các hợp chất hữu cơ Erythronolit B là chất đầu của quá trình sinh tổng hợp của chất kháng sinh erythromixin. Để tổng hợp erythronolit B người ta tiến hành phân tích tổng hợp lùi và thấy rằng cần xuất phát từ tiền chất A’. Tiền chất A’ có công thức

Rất lý thú là A’ được tổng hợp từ nguyên liệu đớn giản là 2,4,6-trimetylphenol theo sơ đồ sau: OH NaOMe, PhMe

BH3,THF,0oC

A

H2O2, NaOH, 0oC

Br

F

Br2, KBr H2O

1. LiOH, H2O 2. CrO3, H2SO4 axeton

G

N

n-Bu3SnH, AIBN toluen, 75oC

MeCOOOH o

EtOAc, 55 C

O

H

CrO3, H2SO4

B

axeton

Al/Hg, THF/H2O

I

o

0C

N

S

S

Ph3P, THF

N

C

Br2, KBr

D

o

DMF, -20 C

A'

K

BzCl Py

[E]

THF

H2O

H2, Ni-Ra

KOH, H2O

L

LDA, THF, -78oC MeI, HMPA

M


TRƯỜNG THPT CHUYÊN BIÊN HÒA ĐÁP ÁN ĐỀ ĐỀ XUẤT

THI CHỌN HỌC SINH GIỎI KHU VỰC MỞ RỘNG NĂM HỌC 2016- 2017 MÔN THI: HÓA HỌC LỚP 11.

Câu Đáp án 1. a. 1 d [ 2] = 0 = k1[S] − k2 [S][ 2] dt r = k2 [S][2] = k1[S] 1. b. d [1] = 0 = k1[S] − k3[S][1] + k4 [3] dt d [3] = 0 = k2 [S][ 2] + k3[S][1] − k4 [3] = k1[S] + k3[S][1] − k4 [3] dt Giai đoạn một chậm nhất, vì thế k1[S] << k3[S][1] , khi bỏ qua k1[S] ta có:

Điểm 0,25

0.25

k3[S][1] = k4 [3] [1] k = 4 [3] k3[S]

1. c. Vì tốc độ sinh ra các gốc tự do là nhỏ, nồng độ của các gốc nhỏ và tốc độ lan truyền mạch tỉ lệ bậc nhất với nồng độ gốc sẽ lớn hơn tốc độ kết hợp của các gốc (là đại lượng tỉ lệ 0.25 bậc 2 với nồng độ gốc). Người ta gọi sự gàn đúng này gọi là sự gần đúng mạch dài ( nhiều giai đoạn phát triển mạch xảy ra trước khi các gốc kết hợp với nhau). 1. d. Tốc độ của hình thành các gốc phải bằng tốc độ kết hợp của chúng. Vì nồng độ

PhCH 2 * phải lớn hơn nhiều so với các gốc khác nên chỉ cần quan tâm đến tốc độ kết hợp của hai gốc benzyl:

d [ R] = 0 = 2k1[S] − 2k R [1]2 dt [1] = r = k3[1][S] =

k1[S] kR k11/ 2k3[S]3 / 2 k R1 / 2

Bậc toàn phần của phản ứng là 1,5.


Hằng số tốc độ hiệu dụng (biểu kiến): 1/ 2

k k k = 1 1 / 23 kR Năng lượng hoạt động hóa:

E=

E1 E E + E3 − R ≈ 1 + E3 , 2 2 2

0,25

(Vì năng lượng hoat động hóa của sự kết hợp các gốc coi là bằng không). 2. a. Có cân bằng: P(I2)o – x

I2(k) ⇌

2I(k)

2x

Ở thời điểm cân bằng: P(I2)cb = P(I2)o – x Như vậy Pchung = P(I2)o + x

Ở 1073K x = 0,0750 – 0,0631 = 0,0119 atm P(I)cb = 2x = 0,0238 atm P(I2)cb = 0,0631 – 0,0119 = 0,0512 atm K=

P ( I )cb 2 = 0, 01106 P ( I 2 ) cb

Tính tương tự cho thời điểm 1173K thu được K = 0,04867

0,25

Từ đây ta có:

ln

K1173 ∆H o  1 1  o = −   ⇒ ∆H = 155, 052 J = 155 kJ K1073 R  1073 1173 

Như vậy ở 1100K ta có

ln

K1100 ∆H o  1 1  = −   ⇒ K1100 = 0, 0169 K1073 R  1073 1100 

∆Go = -RTlnK = 37248,8J ∆G = ∆H - T∆S ⇒∆S = 107,1 J.K-1 b. Có cân bằng:

I2(k)

0,25

2I(k)

P(I2)o – x

2x

Ở thời điểm cân bằng: P(I2)cb = P(I2)o – x Như vậy Pchung = P(I2)o + x

KP =

4 x2 1 1 = P = [ P( I 2 )o + x ] ⇒ P ( I 2 )o = 3x P ( I 2 )o − x 2 2

Vậy Pchung = 4x và P(I2)cb = 2x. Tức phần mol I(k) lúc cân bằng là 0,50 c. Với khí lý tưởng đơn nguyên tử Cv, m= CV,m + R = 3/2R + R = 5/2R

0,25


Với khí lý tưởng hai nguyên tử Cv,m= CV,m+ R = 7/2R

∆Ho298 = Năng lượng liên kết I – I Do sự thay đổi nhiệt độ trở nên đáng kể ∆T = -802K

∆Ho298 = ∆Ho1100 + Cp∆T = 155052 + (298 – 1100) x (2 x 2,5 – 3,5)R = 145,050 kJ = 145 kJ

0,25

x2 = 10 −1,1 → x 2 + 0,0794 x − 10 −3,1 = 0 0,01 − x

2

Na2S → 2 Na+ + S20,01 0,01 + KI → K + I0,06 0,06 + Na2SO4→ 2Na + SO420,05 0,05 S2+ H2O ⇌ HS- + OHKb(1) = 10-1,1 SO42- + H2O ⇌ H SO4- + OH- Kb(2) = 10-12 Kb(1)>> Kb(2) nên cân bằng (1) quyết định pH của dung dịch: S2- + H2O ⇌ HS- + OHK = 10-1,1 [ ] (0,01 -x) x x -3 -3 → x = 8,94. 10 →[OH ] = 8,94.10 → pH = 11,95

a) Tính pH của dung dịch

(1) (2)

0,5 -7,8

PbS : S = 10 -26 = 10 −13 PbI 2 : 3 10 −7, 6 / 4 = 10 −2,7 PbSO 4 : S = 10 = 10 −3,9 S2- → PbS ↓ (Ks-1) = 1026. b. Pb2+ + 0,09 0,01 0,08 Pb2+ + SO42→ PbSO4↓ (Ks-1) = 107,8. 0,08 0,05 0,03 Pb2+ + 2 I→ PbI2 (Ks-1) = 107,6. 0,03 0,06 Thành phần hỗn hợp: ↓A : PbS , PbSO4 , PbI2 Dung dịch B : K+ 0,06M Na+ 0,12M Ngoài ra còn có các ion Pb2+ ; SO42- ; S2- do kết tủa tan ra. Độ tan của Bởi vì độ tan của PbI2 là lớn nhất nên cân bằng chủ yếu trong dung dịch là cân bằng tan của 1 PbI2. 2+ PbI2↓= Pb + 2I Ks Do đó [Pb2+] = 10-47 = 2 x 10-3M và [I-] = 4.10-3M.

2-

[SO4 ] =

[S2-] =

10−7,8 2 × 10−3

=

5. 10−5,8 = 7,9.10−6M << [Pb2+]

=

5. 10−24<< [Pb2+]

10−26 −3

2 × 10 Các nồng độ SO42-, S2- đều rất bé so với nồng độ Pb2+, như vậy nồng độ Pb2+ do PbS và 0,5 PbSO4 tan ra là không đáng kể nên cách giải gần đúng trên là hoàn toàn chính xác.


3

0 0 a. Vì EAg + /Ag > E -

I /AgI

, pin điện có sơ đồ:

(-) Ag, AgI(r) | Ag+(aq),I-(aq) | Ag(r) (+) Phản ứng ở cực âm: Ag(r) + I-(aq) AgI(r) + e Phản ứng ở cực dương: Ag+(aq) + e Ag(r) Phản ứng xảy ra trong pin: Ag+(aq) + IAgI(r) (1) I TAgI = aAg+ ሺaqሻ .aI- (aq) = K Trong đó T là tích số tan, a là hoạt độ, K là hằng số cân bằng của phản ứng (1). Eopin = Eo(+) – Eo(-) = 0,80 – (-0,15) = 0,95 V ∆Go = -F.Eopin = -0,95F = -RTlnK lgK = 0,95/0,059 = 16 K = 1016 TAgI = 10-16 b. TAgI = s2; trong đó s là độ tan của AgI trong nước nguyên chất s = (TAgI)1/2 = 10-8 mol/L

1

2. Trước tiên cần tính thể khử chuẩn của cặp Fe3+/Fe2+ Fe3+ + 3e Fe Eo(1) = -0,037 V, ∆Go(1) = -3FEo(1) 2+ Fe Eo(2) = -0,440 V, ∆Go(2) = -2FEo(2) Fe + 2e -∆Go ሺ3ሻ

∆Go ሺ1ሻ- ∆Go ሺ2ሻ

Fe3+ + e Fe2+Eo ሺ3ሻ = == 3Eo ሺ1ሻ- 2Eoሺ2ሻ = 0,77V F F EoAu3+ /Au+ >EoFe3+ /Fe2+ nên pin điện có sơ đồ: (-) Pt | Fe3+(aq) 1M, Fe2+(aq) 1M || Au3+(aq) 1M, Au+(aq) 1M | Pt Phản ứng ở cực âm: Fe2+(aq) – e Fe3+(aq) 3+ Phản ứng ở cực dương Au (aq) + 2e Au+(aq) 3+ 2+ Phản ứng trong pin: Au (aq) + 2Fe (aq) Au+(aq) + 2Fe3+(aq) o o Eopin = EAu3+ /Au+ - EFe3+ /Fe2+ = 0,49 V

(+)

2×0,49

4

1

∆Gopư = -RTlnK = -2FEopin = 0,49 V → K = 10 0,059 = 3,98.1016 Đặt công thức đơn giản nhất của A, B là XCln và XClm ta có M X + m.35, 45 = 1, 516 ⇒ M X = 68, 70.( m − 1,516n) M X + n.35, 45 thay các giá trị nguyên m, n sẽ có m 2 3 4 4 5 5 5 6 n 1 1 1 2 1 2 3 1 MX 33.25 101.95 170.65 66.50 239.35 135.20 31.05 308.05 X (loại) (loại) (loại) (loại) (loại) (loại) P (loại) Đáp số phù hợp m =5, n = 3, Mx = 31,05 tương ứng với X là P Cl

A là PCl3B là PCl5 Cl

P

b. 3NH4Cl + 3PCl5 →Cl(NPCl2)3 + 12HCl

Cl Cl

6 2 203.90 (loại)

Cl P

6 3 99.75 (loại)

0,5 Cl

Cl

Khi đun nóng mạnh → chất nóng chảy không bị gãy vòng Khi đun nóng nhẹ → Vòng bị bẻ gãy thành các phân tử polime có hệ liên hợp pi


0,5

2. A: ClCN, B: H2NCN, C: (NH2)2CO, D: CaC2, E: CaCN2 Phương trình o

−10 C KCN + Cl2  → KCl + ClCN ClCN + 2NH3 → NH4Cl + H2NCN o

5

70 C H2NCN + H2O  → (NH2)2CO 3000o C CaCO3 + 4C → 3CO + CaC2 CaCl2 → C + CaCN2 CaC2 + N2  1100o C CaCN2 + CO2 + H2O → CaCO3 + H2NCN 1. a.Bát diện b. Ta có

1 0,25

eg

eg

t2g

t2g

spin cao

spin thấp

0,25

c. Phức spin thấp: µ mag = µ B 3 = 1.73µ B spin cao: µ mag = µ B 15 = 3.87µ B

0,25

d. Ta có bảng:

Công thức Tên gọi [Co(CN)6]3Hexaxianocobantat(III) [Co(NO2)6]3Hexa-N-nitritocobantat(III) 3[Co(CO3)3] Tricacbonatocobantat(III) [Co(CO3)2(NH3)2]- Dicacbonatodiamincobantat(III) 2. Gọi anilin = An D = -lgT = -lg0,92 = 0,0362 L = 20 mm = 2 cm D 0, 0362 D = •lC = •l[An] • = = 1, 22.10−5 M εl 2.1, 48.103 Theo cân bằng: C6H5NH2 + H2O C6H5NH3+ + OH-

6

Màu sắc Vàng Cam Xanh Xanh da trời

0,25

Kb = 10-9,2

[C6 H 5 NH 3+ ][OH − ] [C H NH 2 ].10−9,2 = K b = 10−9,2 → [OH − ] = 6 5 (1) [C6 H 5 NH 2 ] [C6 H 5 NH 3+ ] Mà [C6H5NH2] + [C6H5NH3+] = 2.10-4 • [C6H5NH3+] = 2.10-4 – 1,22.10-5 = 1,878.10-4 M Thay các giá trị vào (1) ta có 1,22.10−5 .10−9,2 [OH − ] = = 10−10,39 → [H + ] = 10→3,61 • pH = 3,61 −4 1,878.10 1.

1 1/8.4


2. (0,5 điểm) B có 3C bất đối, không có mặt phẳng và tâm đối xứng nên có 8 đồng phân lập thể. (0,25 điểm) (Cấu hình viết gọn trong bảng, ví dụ với B1 là (1R)-(2R)-(4R)) (0,25 điểm) B1 B2 B3 B4 B5 B6 B7 B8 C1 R S R R S R S S C2 R S R S R S R S C4 R S S R R S S R 3. C, C1, D và D1 là những đồng phân hỗ biến, xúc tác kiềm làm thuận lợi sự hỗ biến đó

Xuất phát từ C, C1, D hoặc D1 qua phản ứng chuyển vị benzylic rồi tự mất nước đều chuyển thành hợp chất thơm bền vững, đều dẫn đến cùng một sản phẩm, ví dụ:

0,25

0,25 4. – Nguyên tử N trong nhóm NH ở trạng thái lai hóa sp2, cặp electron chưa chia ở obitan p xen phủ với 5 obitan p khác tạo thành hệ thơm được lợi về mặt năng lượng nhưng mất tính bazơ Nguyên tử N thứ hai ở trạng thái lai hóa sp2, cặp electron chưa chia ở obitan sp2 không tham gia vào hệ thơm nên còn tính bazơ Nguyên tử N trong nhóm NH2 ở trạng thái lai hóa sp3

0,25


- Nhóm NH3+ là axit liên hợp của nhóm NH2, nhóm NH+ là axit liên hợp của nhóm Nsp2 Bazơ càng mạnh thì axit liên hợp càng yếu, vì thế giá trị 9,2 là của nhóm NH3+ còn 6,0 là của nhóm NH+, nhóm COOH có tính axit mạnh nhất nên có giá trị là 1,8

0,25 7

X có (π+v) = 3 → C10H22 (π+v) = 0 ⇒ X có liên kết bội và vòng 3 cạnh (nếu có) X + H2  X + Br2  → C10H16Br6 (π+v) = 0 1.O3 → Y (C5H8O (π+v) = 2) X  2.Zn / HCl 1.O3 X → Y (Y không có đối quang) 2.H 2 O2

⇒ Y có liên kết đôi C=O và 1 vòng 3 cạnh X có 2 vòng 3 cạnh và liên kết đôi C=C Các công thức cấu tạo của X (mỗi công thức được 0,25 điểm)

0,25

0,25.5

Viết phương trình hóa học của với 1 đồng phân (ví dụ với X1): 0,25 điểm

8

1. A có công thức phân tử CHNO Ở thể khí dC≡N (116 pm) < dCN(A) (121 pm) < dC=N (130 pm) dC≡O (112 pm) < dCO(A) (117 pm) < dC=O (123 pm) Liên kết CN và CO trong A vừa có một phần liên kết đôi, một phần liên kết ba

CTCT của A ở thể khí

1

2. Ka(A) > Ka(CH3COOH) là một axit có oxi - Trong nước, A nằm cân bằng với đồng phân là axit có oxi

Anion sinh ra bền vững, cấu trúc thẳng nên khả năng solvat hóa tốt nên lực axit của A lớn hơn CH3COOH - CH3COOH khi phân li tạo ra anion CH3COO- có khả năng solvat hóa kém, có nhóm kị nước

0,5


nên kém bền, làm giảm tính axit của CH3COOH 3. Trong dung dịch đặc A chuyển thành Y có dạng vòng 6 cạnh dC=N (130 pm) < dCN(A) (135 pm) < dC-N (147 pm) Liên kết CN trong Y có một phần liên kết đơn, một phần liên kết đôi CTCT của Y

9

0,5 0,5

1. O

O

COOH

O

O

N Me

H O

N

H

H H

Me

N

O

Me

N O

H

N

Me

O

-CH3COOH

O H

CH2 -CH3COOH OCOCH3 O

OAc

Me

O

O O

CH2

H

O

O

O N

O

O

N

Me CH2 N

O

Me

Me

2. MeO

0,5 C2H5 MeO

O H

O

benzen

MeO

MeO

O

H

OMe

OMe OH

OMe OH

OMe OH

MeO

O

OH

OMe MeO

OMe

MeO MeO

O

OH MeO

O

OMe

OH OMe

OMe

3.

0,5


4.

0,5 H N

N Ns

O H N

O

Bn

N Ns

Bn O

Bn N C

O

BnHN

O O

N Ns

-H+ +

+H

10

Ns

N Bn O

N Ns N Bn

H N

-H+ +H+

NHBn N Bn

NsHN

O

+H+

O

+H+

Đúng công thức chất A và E được 0,25 điểm Công thức các chất còn lại từ A đến O được 0,15 điểm

N Ns

N B O

H

NHBn N Bn

NHNs

O

CONHBn -H+

O

-H+

O NHBn N Bn O

HO

BnHN

Bn NH O

N Bn NsHN O


O

OH

O

O

o

BH3,THF,0 C

NaOMe, PhMe

CrO3, H2SO4 axeton

H2O2, NaOH, 0oC

Br A O

Br

C

O

Br

Br2, KBr

CH2OH

B

O

KOH, H2O

H2O

THF

O

COOH O

O

O

COO-

[E] O

D O

COOH

F O

Br Br2, KBr H2O

n-Bu3SnH, AIBN

O

o

Al/Hg, THF/H2O

O

toluen, 75 C

O

0oC

O

O

O

O H

G OH

DMF, -20 C

BzCl Py

O

HO

I OBz

OBz

H2, Ni-Ra o

O

HO

LDA, THF, -78oC MeI, HMPA

O

BzO

O

O

BzO

O

O

K

M

L

OBz OBz

OBz 1. LiOH, H2O

N

MeCOOOH

2. CrO3, H2SO4 axeton BzO

EtOAc, O COOH N

S

O Ph P, THF 3 O O

55oC BzO

N BzO

O O O S

OH O

,

S

A'

N


HỘI CÁC TRƯỜNG CHUYÊN VÙNG DUYÊN HẢI VÀ ĐỒNG BẰNG BẮC BỘ

ĐỀ THI MÔN HÓA HỌC – KHỐI 11

TRƯỜNG THPT CHUYÊN HẠ LONG TỈNH QUẢNG NINH

Thời gian làm bài: 180 phút (Đề này có 06 trang, gồm 10 câu)

NĂM 2017

ĐỀ THI ĐỀ XUẤT Câu 1: (2,0 điểm) Tốc độ phản ứng – Cân bằng hóa học 1. Trong môi trường axit, ion hidrosulfit bị ion hidrocromat oxi hóa. Các nghiên cứu chỉ ra phương − 2

trình tốc độ phản ứng: v = k[ HCrO−4 ][ HSO3 ] [ H + ] .Phản ứng thực hiện trong dung dịch đệm ở −

pH=5, nồng độ ban đầu các ion, [ HCrO4 ]o = 10

−4

M; [ HSO3 ]o = 0,1M thì sau 15s, nồng độ ion

HCrO−4 giảm một nửa. Nếu vẫn thực hiện trong dung dịch đệm ở pH=5 nhưng thay nồng độ ban đầu [HCrO−4 ]o = −

0,01M và [ HSO3 ]o = 0,015M. Tính thời gian để HCrO4 giảm còn 12,5% lượng ban đầu. 2. Sự nhiệt phân etanal tiến hành theo cơ chế đã được đơn giản hóa như sau: k1 CH 3CHO  → CH 3* + HCO*

(1)

k2 CH 3* + CH 3CHO  → CH 4 + CH 3CO*

(2)

k3 CH 3CO*  → CH 3* + CO

(3)

k4 HCO*  → H * + CO

(4)

k5 CH 3CHO + H *  → H 2 + CH 3CO*

(5)

k6 2CH 3*  → C2 H 6

(6)

a) Dùng phương pháp gần đúng các trạng thái ổn định của hợp chất trung gian để tìm công thức tính nồng độ của các dạng HCO*, H*, CH3* và CH3CO*. b) Hãy tìm qui luật về tốc độ hình thành metan, etan, hydro và CO theo nồng độ etanal. c) Theo cơ chế trên, etanal có hai cách phân hủy. Đối với mỗi cách, hãy tìm bậc của etanal. 3. Cho một bình kín dung tích 22,4 lít chứa sẵn 1 mol rắn A và 0,55 mol khí B. Đun nóng bình đến 2730C và dừng lại khi áp suất của bình giữ ổn định ở 2,9 atm. a. Tính áp suất riêng phần của từng khí trong hỗn hợp cân bằng b. Chuyện gì sẽ xảy ra nếu ban đầu chỉ có 0,1 mol rắn A? Biết các cân bằng xảy ra trong bình như sau:  → C( k ) + D( k ) (1) K P = 6 A( r ) + B( k ) ←  1  → E( k ) + D( k ) (2) K P = 9 C( k ) + B( k ) ←  2 5 Câu 2: (2,0 điểm) Cân bằng trong dung dịch điện li 1. Có hai dung dịch A chứa H2C2O4 0,1M và dung dịch B chứa Na2C2O4 0,1M. Tính pH và nồng độ ion C2O42có trong dung dịch A và B. 1


2. Thêm Fe(NO3)3 (tinh thể) vào dung dịch A và dung dịch B để đạt nồng độ (ban đầu) là 1,0.10-4M. Giả thiết thể tích dung dịch thay đổi không đáng kể. Hãy cho biết có xuất hiện kết tủa Fe(OH)3 không? Chứng minh. 3. Tính phần mol của phức Fe(C2O4)33- trong dung dịch A. Cho các giá trị: Hằng số tạo thành tổng hợp của phức Fe3+ với C2O42- là β1 = 1,0.108; β2 = 2,0.1014; β3 = 3,0.1018; KW = 10-14. Hằng số phân ly axit của H2C2O4 là Ka1 = 0,05; Ka2 = 5.10-5. Tích số tan của Fe(OH)3 Ks = 2,5.10-39. Câu 3: (2,0 điểm) Điện hóa học Trộn 10ml dd Ag+ 0,01M với 10ml dd NH3 0,12M thu được dd A. Trộn 10ml dd Ag+ 0,02M với 10ml dd CrO 24 − 0,02M được hỗn hợp B. Ghép điện cực Ag nhúng trong dd A với điện cực Ag nhúng trong hỗn hợp B thành pin 1. Pin 2 được ghép bởi điện cực hiđro nhúng trong dd NH4HSO4 0,01M và điện cực hiđro nhúng trong dd (NH4)2S 0,05M. a) Cho biết anot, catot của mỗi pin? Tính suất điện động và viết sơ đồ pin của 2 pin trên? d) Mắc xung đối pin 1 và pin 2. Hãy viết quá trình xảy ra trong 2 pin sau khi mắc xung đối. Từ đó cho biết có thể dùng NH3 làm thuốc thử để hòa tan Ag2CrO4 không?( không căn cứ vào hằng số cân bằng) Cho lg β Ag ( NH )+ = 7, 24; pK s ( Ag2CrO4 ) = 11,89; pK a ( HSO − ) = 2; pK a ( NH + ) = 9, 24; pK a ( H 2 S ) = 7, 02;12,90 3 2

4

4

Câu 4: (2,0 điểm) Nhóm N – P, nhóm C - Si 1. Một nguyên tố X có khả năng phản ứng với canxi cho chất Y. Mặt khác X tan được trong dung dịch kiềm tạo ra một hợp chất A và khí B đều có chứa nguyên tố X. A phản ứng với clorua vôi thu được một kết tủa C. Kết tủa này sẽ chuyển thành Y khi xử lý với nhôm ở nhiệt độ cao. Hòa tan chất Y trong dung dịch HCl loãng thu được B. Biết rằng khi xử lí C với SiO2 và than cốc thu được X, còn trong trường hợp không có than cốc thu được D. D tan được trong cả dung dịch axit loãng và kiềm loãng. Lập luận xác định cấu trúc các chất chưa biết và viết các phương trình phản ứng xảy ra. 2. Một hợp chất A rất độc có màu vàng được sinh ra khi trộn hai axit đặc B và C. Sản phẩm phụ của phản ứng là khí E màu vàng và nước. Mặc dù hai axit đã được biết đến từ rất lâu, nhưng mãi đến năm 1831 thì người ta mới quan sát thấy phản ứng tạo thành A. Một cách khác để thu được A là tiến hành phản ứng giữa hai khí E và D, quá trình này không sinh ra sản phẩm phụ. Tuy nhiên ở nhiệt độ cao phản ứng xảy ra theo chiều nghịch. Khí D được tạo thành khi cho kim loại chuyển tiếp I phản ứng với axit loãng C. Nếu dùng axit đặc thì thay vì D thu được một khí M màu đỏ nau. Trong cả hai trường hợp đều thu được một muối duy nhất J màu xanh da trời. Khí M phản ứng với nước thu được hai axit C và F. Axit F khi phản ứng với axit G thu được hợp chất H, hợp chất này khi phản ứng với axit B cho hai chất A và G. Hợp chất H có thể xem như anhydrit của hai axit này. Khi cho B phản ứng với KMnO4 thu được một lượng nhỏ khí E. Khí này được sử dụng để tổng hợp chất K bằng cách cho phản ứng với muối NaCN. Nhiệt phân Hg(CN)2 thu được L là một chất có tính đối xứng. Khi cho axit G phản ứng với BaCl2 thu được kết tủa trắng. Khí D chuyển thành khí M khi tiếp xúc với không khí. Xác định các chất chưa biết. Câu 5: (2,0 điểm) Phức chất, trắc quang 1. Một hợp chất của Cr được tổng hợp. Sự phân tích nguyên tố cho thấy thành phần của nó là: Cr 27,1%; C 25,2%; H 4,25% khối lượng, phần còn lại là oxi. a. Công thức thực nghiệm của hợp chất này là gì? b. Nếu công thức thực nghiệm gồm 1 phân tử nước, những phối tử khác là gì? Trạng thái số oxi hóa của Cr? 2


c. Nghiên cứu tính chất từ cho thấy rằng hợp chất này là nghịch từ, giải thích tính chất từ của hợp chất này như thế nào? Vẽ cấu trúc của hợp chất này. 2. Từ kim loại M (Z<37) thực hiện các sơ đồ chuyển hóa sau: 0

150 ,15 atm M + nCO  →A 0

t A + 4 KOH  → B + C + 2 H 2O

a) Xác định B. Biết một hợp phần trong B có cấu trúc tứ diện, cacbon chiếm 19,512% khối lượng B. b) Cho A phản ứng với dixyclopentadien, đun nóng thu được D và hỗn hợp khí Y (CO, H2) có dY/H2 = 85/7. Ở điều kiện thường, D là tinh thể màu đỏ tím đậm, dễ dàng hòa tan trong các dung môi hữu cơ phân cực vừa phải như clorofom, pyridin, ít tan trong các dung môi không cực (như CCl4), không tan trong nước. Sản phẩm của sự khử D bằng kim loại kiềm hoặc hidrua được sử dụng rộng rãi vì khả năng dễ ankyl hóa, axyl hóa của nó. Viết các đồng phân tương ứng của D. 3. Một phương pháp đơn giản để đo nồng độ của ozon trong khí quyển mặt đất được tiến hành như sau. Cho bọt không khí đi qua dung dịch nước đã axit hóa có chứa iot và ozon trong khí quyển sẽ oxi hóa iotdua thành triiodua theo phản ứng chưa cân bằng sau: O3(k) + I-(dd) + H+(dd) I3- (dd) + O2(k) + H2O (1) Sau khi phản ứng kết thúc, nồng độ triiodua được xác định bằng máy đo quang phổ UV-Vis tại 254 nm. Tiến hành thí nghiệm, như sau: Sục bọt không khí trong 30 phút vào 10mL dung dịch nước chứa KI dư tại điều kiện khí quyển như sau: áp suất = 750 torr, nhiệt độ 298K, tốc độ dòng = 250 mL.phút-1. Độ hấp thụ của dung dịch I3- tạo thành đo được trong tế bào có độ dày l = 1,1 cm khi sử dụng một máy trắc quang có trang bị tế bào quang điện. Điện trở của tế bào quang điện tỷ lệ nghịch với cường độ của ánh sáng. Trị số điện trở (của tế bào quang điện) khi bị chiếu bới chùm sáng đi qua cuvet trống và qua cuvet chứa mẫu hòa tan là 12,1 kΩ và 19,4 kΩ . Hệ số hấp thụ mol của I3- hòa tan được xác định là ε = 2,4.105 M −1.cm−1 a. Tính số mol ozon trong mẫu trong không khí. b. Giả thiết rằng các khí được sử dụng là khí lý tưởng. Tính nồng độ theo ppb của ozon có mặt trong mẫu không khí. Câu 6: (2,0 điểm): Quan hệ giữa hiệu ứng cấu trúc và tính chất 1.Cho các công thức cấu tạo sau: C(CH=CHF)2 CH CH2 F (A)

Me 1 2 4 5 Me 3 Et O O Me O (B)

Me

OH O

O 2N (C)

Me

O

COOH

N OH

O 2N (D)

N H

NH2 (E)

a) Hãy vẽ công thức các đồng phân lập thể ứng với cấu tạo A. b) Ứng với công thức cấu tạo B có bao nhiêu đồng phân lập thể, vì sao? Dùng các kí hiệu thích hợp để chỉ rõ cấu hình của mỗi đồng phân đó. c) Hãy viết cơ chế phản ứng để giải thích vì sao C và D khi tương tác với dung dịch NaOH thì đều tạo thành natri 3-metyl-4-nitrobenzoat. d) Hãy chỉ rõ trạng thái lai hóa của từng nguyên tử N ở cấu tạo E và ghi giá trị pKa (ở 25 oC): 1,8; 6,0; 9,2 vào từng trung tâm axit trong công thức tương ứng với E, giải thích. 2. Trong số các chất sau đây, chất nào thuộc loại hợp chất thơm, không thơm, phản thơm, vì sao? Chất nào tác dụng được với kali kim loại trong đietyl ete? Viết phương trình phản ứng và giải thích. 3


EtOOC

O

EtOOC

COOEt (F)

(G )

(H )

(I )

(J )

Câu 7: (2,0 điểm): Hidrocacbon 1. Dewar đã đề nghị cấu trúc C cho benzen, cấu trúc này gồm hai vồng xiclobutan ngưng tụ với nhau: Để làm giảm sức căng vòng, C dễ bị chuyển vị dưới tác dụng của nhiệt tạo thành hợp chất đơn vòng D. a. Xác định công thức cấu tạo và hóa lập thể của D. Gọi tên D theo danh pháp IUPAC, sử dụng kí hiệu lập thể E/Z. b. Điều thú vị là dù có sức căng rất lớn nhưng hợp chất C không chuyển hóa ngay thành hợp chất E (gọi là cấu trức Kekule) vốn rất bền. Cho C phản ứng với một axit vô cơ thu được chất E. Đề xuất một cơ chế phản ứng cho chuyển hóa này. c. Dự đoán công thức cấu tạo của hợp chất F (C8H8O4) biết F có khả năng phản ứng với Chì tetraaxetat để tạo thành C. d. Ladenburg cũng đề nghị một cấu trúc khác của benzen, sau này được biết đến với tên gọi Prismane H. Cấu trúc này cố gắng giải thích cho sự hình thành một sản phẩm thế mono với benzen và ba đồng phân thế 2 lần của benzen. Quang phân G tạo thành một lượng nhỏ H. Xác định công thức cấu tạo của H. Xác định CTCT dạng Ladenburg của tất cả các đồng phân đibrombenzen.

e. Một cấu trúc khác được đề nghị cho benzen là benzvalene I. I được điều chế theo sơ đồ sau, hãy xác định các chất J,K trong chuỗi tổng hợp đó:

f. Trimetylenxiclopropan N là một đồng phân khác của benzen, có thể tổng hợp được từ chất L theo sơ đồ sau, xác định chất M,N. Vẽ cấu trúc các đồng phân lập thể của L. Dự đoán công thức cấu tạo của sản phẩm O tạo thành từ phản ứng Diels-alder giữa N và axetilen. 4


2. Phân từ hidrocacbon G (C8H8) chỉ gồm các liên kết đơn và chỉ chứa cacbon bậc ba tương đương nhau. Cho chất G phản ứng với H2/Pd ở điều kiện khắc nghiệt, thu được chất H (C8H10) và chất I (C8H14). Trong phân tử của I. Tỉ lệ số nguyên tử C bậc hai và bậc ba là 3:1. Chất G phản ứng với CBr4/NaOH 50% thu được chất K (C8H7Br) và L (C8H6Br2). Đồng phân hóa G bằng AgClO4 thu được X. Trong phân từ X chỉ chứa cacbon bậc ba và chia làm ba nhóm tương đương nhau. Xác định cấu trúc các chất G,H,I,K,L và X. Câu 8: (2,0 điểm): Xác định cấu trúc Hai lacton thơm A, B có công thức phân tử (C10H10O4) đều tan trong dung dịch NaOH loãng nhưng không tan trong dung dịch NaHCO3. Cả A và B cho phản ứng màu với dung dịch FeCl3. Khi cho A phản ứng CH3I/K2CO3 tạo ra chất C (C11H12O4). Biết C chứa ba nhóm metyl không giống nhau, trong đó có một nhóm metyl liên kết với vòng thơm. Xử lí C với BCl3 để tách loại một nhóm metyl tạo ra D là một đồng phân mới của A. D có một nhóm hidroxi tạo liên kết hidro nội phân tử. Cho chất E (2-metyl-1,3-dihidroxibenzen) phản ứng với MeI/K2CO3 tạo F(C9H12O2), F được khử bằng Li/NH3 lỏng có mặt 2-metylpropan-2-ol cho một ddien đối xứng và không liên hợp G. Tiếp tục cho G phản ứng KNH2/NH3 lỏng tạo ra một sản phẩm H. Thực hiện phản ứng ozon phân H rồi xử lí tiếp thu được nhiều sản phẩm trong đó có metyl 2-oxopropanoat. Thực hiện phản ứng giữa H với dimetylbut-2-indioat, đun nóng tạo K(C15H20O6), tiếp tục đun nóng K để loại eten tạo được một este thơm L. Thủy phân L trong môi trường bazo rồi axit hóa dung dịch tạo thành M (C11H12O6), đun nóng M trong chân không tạo ra N (C11H10O5). Khử N bằng NaBH4 trong DMF tạo C và một lacton P đồng phân, P cũng có thể thu được nhờ metyl hóa B. Hãy biện luận xác định cấu tạo các chất từ A đến P Câu 9: (2,0 điểm): Cơ chế phản ứng 1. Khi đun nóng hợp chất A sẽ chuyển thành B,C,D. Phản ứng được thúc đẩy khi chiếu xạ. Hãy xác định cấu trúc của B,C,D và dung cơ chế phản ứng để giải thích các sản phẩm này.

2. Người ta đã điều chế 2-phenylxiclo [3.2.2] azin từ 2,6-lutidin qua ba giai đoạn: 1) 2-phenyl-5-metylpirocolin được tạo thành với hiệu suất 86% từ 2,6-lutidin và phenaxyl bromua:

2) Xử lí 2-phenyl-5-metylpirocolin bằng phenyl liti trong ete ở -300C. 3) Cộng hợp liên tiếp với N,N-đimetylfomamit và đun nóng trong thời gian ngắn với axit axetic sẽ thu được sản phẩm với hiệu suất 45%. 5


Viết cơ chế phản ứng xảy ra trong quá trình tổng hợp này. Hãy dẫn ra cấu trúc cộng hưởng quan trọng nhất phù hợp với chính công thức xiclo [3.2.2]azin. So sánh tính bazo của sản phẩm với anilin, so sánh khả năng nitro hóa với pirol và piridin. Nhóm nitro vào vị trí nào dễ nhất? Công thức của 2-phenylxiclo [3.2.2] azin:

N

Ph

Câu 10: (2,0 điểm): Tổng hợp các hợp chất hữu cơ 1. Thromboxan (C21H34O4) là một loại thuốc làm co mạch và là một tác nhân làm tăng mạnh huyết áp. Dược phẩm thromboxane A2 được tổng hợp theo sơ đồ sau đây:

Xác định cấu trúc các chất chưa biết, tên phản ứng sử dụng ở giai đoạn chuyển B thành C và đề nghị một cơ chế hợp lí cho phản ứng chuyển D thành E. 2. Huperzine A là một hợp chất được sử dụng trong điều trị chứng mất trí nhớ cũng như rối loạn đường máu. Các nhà khoa học Trung Quốc đã tổng hợp A theo sơ đồ sau. Hãy xác định cấu tạo các chất chưa biết trong sơ đồ. Biết G là đimetyl este có công thức C12H15NO5

**********************************

HẾT **********************************

6


HƯỚNG DẪN CHẤM ĐỀ ĐỀ XUẤT HÓA 11 (15 trang) Câu 1: (2,0 điểm) Tốc độ phản ứng – Cân bằng hóa học Phần Đáp án −5 1 +

a) [ H ] = 10 M = const. − [ HSO3 ]o

≫ [ HCrO−4 ]o

− → [ HSO3 ]o = const.

− − 2 + v = k[ HCrO4 ][ HSO3 ] [ H ]

k′ [ HCrO−4 ] ( k′ = k[ HSO3− ]2 [ H + ] )

=

→ Phản ứng bậc nhất theo

k′ =

HCrO−4 .

ln 2 −1 (s ) 15

− + PTHH: 2HCrO− → 2Cr 3+ + 3SO42− + 5H 2O 4 + 3HSO3 + 5H  − − Theo phương trình: Tỉ lệ HCrO 4 và HSO3 là 2:3. − − Lại có: [ HCrO 4 ]o : [ HSO3 ]o = 2:3. → Tại mọi thời điểm trong quá trình phản ứng luôn có:

3 [HSO3− ] = [HCrO−4 ] . 2 − − 2 + v = k[ HCrO 4 ][ HSO3 ] [ H ] 2

3  k.[H ]. [HCrO −4 ]  .[HCrO −4 ] 2  9 k[H + ].[HCrO4− ]3 4 +

=

=

k 2 .[HCrO 4− ]3

=

k′ = k.[H + ].0,12 → k.[H + ] = 100k ′. 9 ln 2 k 2 = .100. = 10, 4 ( M−2.s−1 ) 4 15

Có:

Lại có:

1 d[HCrO −4 ] d[HCrO −4 ] v=− . →− = 2v = 2.k 2 .[HCrO 4− ]3 2 dt dt

Tích phân 2 vế:

Điểm 0,5


 1 1 1 −   = 2k 2 t 2  [HCrO 4− ]2 [HCrO 4− ]o2  1  82 1  ⇔  −  = 2.10, 4.t 2  0,012 0,012  → t = 15144, 23(s). 2

0,75

a/ 0,25 điểm d  HCO* 

Theo (1) và (4):

dt d  H * 

Theo (4) và (5):

Theo d CH 3*  dt

dt

= k1[CH 3CHO] − k4 [HCO* ] = 0 ⇒ [HCO* ] =

= k 4 [HCO* ] − k5 [H* ][CH 3CHO] = 0 ⇒ [H* ] =

(1),

(2),

k1 [CH 3CHO] k4

k4 [HCO* ] k = 1 k5 [CH 3CHO] k5

(3)

(6):

= k1[CH 3CHO] − k 2 [ CH 3CHO ][CH *3 ] + k3 [CH 3CO* ] − 2k6 [CH*3 ]2 = 0

Theo (2), (3) và (5):

d CH 3CO* 

= k2 [CH*3 ][CH 3CHO ] − k3 [ CH 3CO* ] + k5 [H* ][CH3CHO ] = 0

dt Cộng hai biểu thức trên, rồi thế giá trị nồng độ của H* và HCO* vào ta được

2k1[CH 3CHO] − 2k6 [CH*3 ]2 = 0 ⇒ [CH*3 ] =

⇒ CH 3CO  = *

k2 CH 3*  + k5  H *  k3

k1 [CH 3CHO]1/2 k6

[CH 3CHO ] =

k2 k3

k1 k 3/2 [CH 3CHO] + 1 [CH 3CHO ] k6 k3

b) 0,25 điểm Tốc độ hình thành:

d [CH 4 ] dt

= k2 [CH*3 ][CH 3CHO ] = k2 .

d [C2 H 6 ]

dt d [H2 ]

dt

d [CO ] dt c) 0,25 điểm

k1 3/2 [CH 3CHO ] k6

= k 6 [CH *3 ]2 = k1[CH 3CHO ]

= k5 [H* ][CH 3CHO] = k1[CH 3CHO]

= k3 [CH 3CO* ]+k4 [HCO* ] = 2k1[CH 3CHO]+k2

k1 [CH 3CHO]3/2 k6


Etanal có hai cách phân hủy:

2CH 3CHO → C2 H 6 + H 2 +2CO

(a)

CH 3CHO → CH 4 + CO

(b)

+ sự hình thành etan và hydro là bậc một đối với etanal ⇒ phản ứng (a) là phản ứng bậc nhất + sự hình thành metan là bậc 3/2 ⇒ phản ứng (b) là phản ứng bậc 3/2 + CO được hình thành từ 2 phản ứng nên nồng độ của CO phụ thuộc vào nồng độ của etanal theo bậc 1 và bậc 3/2. 3

a.( 0,5đ) Ta có cân bằng sau ở 2730C:

0,75

 → C( k ) + D( k ) (1) K P = 6 A( r ) + B( k ) ←  1  → E( k ) + D( k ) (2) K P = C( k ) + B( k ) ←  2

Tại thời điểm ban đầu: PB 0 =

0 B

9 5

n RT = V

22, 4 .546 273 = 1,1 atm 22, 4

0,55.

Tại thời điểm cân bằng: PB + PC + PD + PE = 2, 9 atm Nhận xét: trong quá trình phản ứng, lượng chất B mất đi bằng với lượng chất D tạo thành. Hay nói cách khác, tổng lượng B và D tại thời điểm cân bằng với lượng ban đầu của chất B 0

Vậy ta được : PB + PD = PB = 1,1 atm Mặt khác:

PE PD K P2 PC PB PE KP 6 10 = = 2 ⇒ PE = 2 PC2 = PC2 = PC2 9 K P1 PC PD PC K P1 3 5 PB Từ các phương trình trên ta có được:

10 2 PC = 2,9 − 1,1 = 1,8 3 ⇒ PC = 0, 6 atm PC +

⇒ PE = 1, 2 atm Tính PB và PD dựa vào K1:

PB PC 0, 6 = = = 0,1 và PB + PD = 1,1 ⇒ PB = 0,1 atm; PD = 1, 0 atm PD K1 6

Tính lại để biết rằng phản ứng theo chiều thuận có kết thúc trước khi đạt cân bằng hay không Từ 2 cân bằng (1) và (2) ta nhận thấy: Lượng chất A mất đi bằng tổng lượng chất C tạo thành, mà tổng lượng chất C này chuyển hóa một phần vào E nên có thể kết luận rằng: nên phản ứng (1) theo chiều thuận chưa kết thúc khi đạt trạng thái cân bằng.

b)(0,25đ) Nếu nA = 0,1 mol thì lúc này

∆n A > n A nên phản ứng (1) theo chiều thuận đã kết thúc trước khi đạt

trạng thái cân bằng. Lúc này áp suất bình tại thời điểm cân bằng sẽ khác đi. Lưu ý: Do đề bài không yêu cầu tính lại áp suất bình nên chỉ cần có câu kết luận trên là được trọn điểm, nếu thí sinh có tính toán lại mà không kết luận cũng không được tính điểm

Câu 2: (2,0 điểm) Cân bằng trong dung dịch điện li


Phần 1

Đáp án

Điểm 1đ

2-

Tính pH và nồng độ ion C2O4 có trong dung dịch A Ta có: H2C2O4 H+ + HC2O4Ka1 = 0,05 2+ HC2O4 H + C 2 O4 Ka2 = 5,0.10-5 Vì Ka1 >> Ka2 nên (1) là cân bằng chính. Gọi C là nồng độ ban đầu của A.

K a1 =

(1) (2)

[H + ]2 [H + ][HC2 O-4 ] = [H 2 C2 O 4 ] CA - [H + ]

[H + ]2 + K a1[H + ] - K a1CA = 0 ⇒ [H+] = 0,05M ⇒ pH = 1,3.

K a2 = Mà ta có:

[H + ][C2 O 42- ] [HC 2 O -4 ] và [H + ] = [HC2 O -4 ] = 0,05M

−5 ⇒ [C 2 O 24 ] = K a2 = 5.10 M

Tính pH và nồng độ ion C2O42- có trong dung dịch B Ta có: H2O + C2O42HO- + HC2O4- Kb1=2,0.10-10 (1) H2O + HC2O4HO- + H2C2O4 Kb2=2,0.10-13 (2) Vì Kb1 >> Kb2 nên (1) là cân bằng chính.

[OH - ] = K b1C = 2,0.10-10 .0,1 = 4,5.10-6 M

⇒ pH = 8,7 ⇒

2

[C 2 O 42- ] = 0,1M

Chứng minh dung dịch A không có kết tủa Fe(OH)3: Có các cân bằng: Fe3+ + C2O42FeC2O4+ Fe3+ + 2C2O42Fe(C2O4)2-. Fe3+ + 3C2O42Fe(C2O4)23Bảo toàn nồng độ ion Fe3+ ta có:

CFe3+ = [Fe3+ ] + [FeC2O4+ ] + [Fe(C2O4 )2- ] + [Fe(C2O4 )33- ] Mà

[FeC2 O 4 + ] = β1[Fe3+ ][C2 O 2-4 ]

[Fe(C2 O 4 ) 2 - ] = β 2 [Fe3+ ][C2 O 2-4 ]2 [Fe(C2 O 4 )33- ] = β 3 [Fe3+ ][C2 O 2-4 ]3 Suy ra

CFe3+ = [Fe3+ ] + β1[Fe3+ ][C2O2-4 ] + β2 [Fe3+ ][C2O2-4 ]2 + β3[Fe3+ ][C2O2-4 ]3 [Fe3+ ] = ⇒

CFe3+ 24

1 + β1[C2 O ] + β 2 [C 2 O 42- ]2 + β 3 [C2 O 42- ]3


1,0.10−4 = = 1,1.10−10 M 8 −5 14 −5 2 18 −5 3 1 + 1,0.10 .5,0.10 + 2, 0.10 .(5,0.10 ) + 3,0.10 .(5,0.10 ) Mà [OH-]A = 2.10-13M

[Fe3+].[OH-]3 = 9,1.10-49 < Ks

⇒ không có kết tủa Fe(OH)3 ở dung dịch A. * Chứng minh dung dịch B có kết tủa Fe(OH)3: Có các cân bằng: Fe3+ + C2O42FeC2O4+ Fe3+ + 2C2O42Fe(C2O4)2-. Fe(C2O4)23Fe3+ + 3C2O42Cách 1: Bảo toàn nồng độ ion Fe3+ ta có:

CFe3+ = [Fe3+ ] + [FeC2O4+ ] + [Fe(C2O4 )2- ] + [Fe(C2O4 )33- ] Mà

[FeC2 O 4 + ] = β1[Fe3+ ][C2 O 2-4 ]

[Fe(C2 O 4 ) 2 - ] = β 2 [Fe3+ ][C2 O 2-4 ]2 [Fe(C2 O 4 )33- ] = β 3 [Fe3+ ][C2 O 2-4 ]3 Suy ra

CFe3+ = [Fe3+ ] + β1[Fe3+ ][C2O2-4 ] + β2 [Fe3+ ][C2O2-4 ]2 + β3[Fe3+ ][C2O2-4 ]3 [Fe3+ ] = ⇒

CFe3+ 24

1 + β1[C2 O ] + β 2 [C 2 O 42- ]2 + β 3 [C2 O 42- ]3

1, 0.10−4 = = 3,3.10−20 M 8 14 2 18 3 1 + 1, 0.10 .0,1 + 2, 0.10 .(0,1) + 3, 0.10 .(0,1) Cách 2: Cân bằng chủ yếu:

Fe3+ + 3C2O42 − Fe(C2O4 )33− 10-4 0,1 → Xét cân bằng ngược lại

10-4

Fe(C2O4 )33− Fe3+ + 3C2O42− 10-4 -x

Cân bằng:

x

0,1 + 3x

3

x.(0,1) = (3.1018 ) −1 −4 10 − x

vì x < 10-4 nên 3x << 0,1

→ x = [Fe3+] = 3,33.10-20 M Mà [OH-]A = 4,5.10-6M ⇒ [Fe3+].[OH-]3 = 3,0.10-36 > Ks = 2,5.10-39

⇒ có kết tủa Fe(OH)3 ở dung dịch B. 3. Phần mol của phức Fe(C2O4)33- trong dung dịch A. Phần mol của Fe(C2O4)33- được tính như sau:


[Fe(C2 O4 )33- ] β3[Fe3+ ][C2 O42- ]3 x= = CFe3+ [Fe3+ ] + β1[Fe3+ ][C2O2-4 ] + β 2 [Fe3+ ][C2O2-4 ]2 + β3[Fe3+ ][C2O2-4 ]3 =

β 3[C2O 42- ]3 1 + β1[C2 O 42- ] + β 2 [C 2 O 42- ]2 + β 3 [C2 O42- ]3 =

3, 0.1018.(5, 0.10−5 )3 = 0, 43 1 + 1, 0.108.5, 0.10−5 + 2, 0.1014.(5, 0.10−5 ) 2 + 3, 0.1018.(5, 0.10−5 )3

Vậy phần mol của Fe(C2O4)33- bằng 0,43.

Câu 3: (2,0 điểm) Điện hóa học a • Xét pin 1: + + Dung dịch A: NH3: 0,05M; Ag(NH3) 2 : 0,005M.

Ag ( NH 3 ) 2+ ⇌ Ag + + 2 NH 3 0,005 0,005 – x

0,05 0,05 + 2x

x

2

(0,05 + 2 x) .x = 10−7,24 0,005 − x

⇒ x = 1,15.10 −7 = [Ag+]A

2−

+ Dung dịch B: CrO 4 : 0,005M; Ag2CrO4

Ag 2CrO4 ⇌ 2 Ag + + CrO42− 2x

0,005 0,005 + x

⇒ (2 x) 2 .(0,005 + x) = 10−11,89 ⇒ x = 8,02.10 −6 = [Ag+]B Vì [Ag+]A < [Ag+]B nên trong pin 1: Ag/dd A là anot; Ag/dd B là catot Và E pin1 = Ec − Ea = 0,109(V ) Sơ đồ pin 1:

(−)Ag / NH3 0,05M

//

2−

CrO 4 0,105M / Ag (+)

+

Ag(NH3) 2 0,005M Ag2CrO4 • Xét pin 2: 0,5 − + + Dung dịch NH4HSO4 0,01M. So sánh pK của HSO 4 và của NH 4 , suy ra, tính H+ theo cân bằng của HSO 4 ⇒ [H+](1) = 6,18.10-3 M + Dung dịch (NH4)2S 0,05M. −

NH 4+ + S 2− → NH 3 + HS − 0,1 0,05 0,05 0 Xét các cân bằng:

0,05

0,05

K = 1012,9 - 9,24 >>


NH 4+ ⇌ NH 3 + H +

(1)

K=10−9,24

HS − ⇌ S 2− + H +

(2)

K=10−12,9

H 2O ⇌ OH − + H +

(3)

K=10−14

NH 3 + H 2O ⇌ NH 4+ + OH −

(4)

K=10−4,76

HS − + H 2O ⇌ H 2 S + OH −

(5)

K=10−6,98

So sánh (1), (2), (3) thì bỏ qua cân bằng (2), (3) So sánh (3), (4), (5) thì bỏ qua cân bằng (3), (5). Vậy cân bằng (1) và (4) quyết định pH của dung dịch, coi đây là hệ đệm Nồng độ của NH3 và NH 4 bằng nhau nên pH = 9,24 ⇒ [H+](2) = 10-9,24 M Vì [H+](1) > [H+](2) nên: Đ/c H2 nhúng trong dung dịch NH4HSO4 là catot Đ/c H2 nhúng trong dung dịch (NH4)2S là anot Và Epin2 = 0,416(V) Sơ đồ pin 2: +

+

(-) Pt(H2) / NH 4 0,05M

+

//

0,75

NH 4 0,01M / Pt(H2) (+) −

NH3 0,05M

HSO 4 0,01M

b

HS 0,05M Khi mắc xung đối, pin 2 sẽ là nguồn điện, pin 1 sẽ là bình điện phân( do Epin1 < Epin2) Vì vậy: + Đối với pin 2:

0,75

catot : 2 HSO4− + 2e → 2 SO42− + H 2 anot : H 2 + 2 NH 3 → 2 NH 4+ + 2e −

+

2−

Và quá trình phóng điện: HSO4 + NH 3 → NH 4 + SO4 + Đối với pin 1( là bình điện phân)

Catot : Ag ( NH 3 ) 2+ + 1e → Ag + 2 NH 3 Anot : 2 Ag + CrO42− → Ag 2CrO4 + 2e Và quá trình nạp điện: 2 Ag ( NH 3 ) 2 + CrO4 → Ag 2CrO4 + 4 NH 3 (*) Quá trình (*) không tự diễn biến nên quá trình ngược lại sẽ là tự diễn biến. Vì vậy, có thể dùng NH3 để hòa tan kết tủa Ag2CrO4. Câu 4: (2,0 điểm) Nhóm N – P, nhóm C - Si +

1

. Do X phản ứng được với Ca nên X là một phi kim. Trong dung dịch kiềm X hòa tan sinh ra 0,1 một muối tan và một khí. Nguyên tố x có mặt trong cả hai thành phần ấy. Trong hợp chất khí tồn tại liên kết X - H. Như vậy chỉ có thể có ba khả năng là Silan, Photphin và amoniac. X sinh ra khi cho than cốc tác dụng với muối C (Có chứa X) và SiO2 nên X chỉ có thể là photpho. Các phản ứng xảy ra như sau: 0,1x9 P4 + 3NaOH + 3H2O 3NaH2PO2 + PH3 P4 + 6Ca 2Ca3P2


2NaH2PO2 + 4CaOCl2 Ca3(PO4)2 + CaCl2 + 2NaCl + 4HCl 2Ca3(PO4)2 + 10C + 6SiO2 6CaSiO3 + 10CO+ P4 3Ca3(PO4)2 + 16Al 3Ca3P2 + 8Al2O3 Ca3P2 + 6HCl 3CaCl2 + PH3 2Ca3(PO4)2 + 6SiO2 6CaSiO3 + P4O10 P4O10 + 6H2O 4H3PO4 P4O10 + 12NaOH 4Na3PO4 + 6H2O 2

A: NOCl B: HCl C:HNO3 D:NO E:Cl2 F:HNO2 G:H2SO4 H:NOHSO4 I:Cu J:Cu(NO3)2 K:CNCl L:(CN)2 M:NO2 Tối đa được 1 điểm, thiếu một chất trừ 0,1đ. Làm được dưới 4 chất không được điểm nào

Câu 5: (2,0 điểm) Phức chất, trắc quang 1

0,75 a. CTTN: CrC4H8O5. b.Phức có dạng là [Cr(CH3COO)2(H2O)]. Do nhóm axetat có điện tích là 1- nên số oxi hóa của Cr ở đây là +2.n b c. Cr+2 có cấu hình e là d4. Đối với các phối tử trường yếu thì phức trên phải là phức spin cao. Tuy nhiên kết quả thu được hợp chất này nghịch từ vì nó tồn tại dưới dạng cấu trúc như sau: CH3 CH3 O

O O

OH2

Cr

O

O Cr

H2 O

O O

O

H3C CH3

2

a) A có CTTQ: M(CO)n Từ phương trình phản ứng: M(CO)n + 4KOH B + C + 2H2O C phải là muối K2CO3 B: K2[M(CO)n-1] mà [M(CO)n-1]2- có cấu trúc tứ diện n-1=4 hay n=5 Trong B: %C=19,512% M=56 (Fe) Vậy B là K2[Fe(CO)4] b) D là Fe2(CO)4(C5H5)2

0,25


0,5

3

0,5

Từ biểu thức định luật lambe-beer, ta có: A = -log T = -log(Imẫu/Icuvet trổng)=log(Rmẫu/Rcuvet trổng)=log(19,4/12,1)=0,205 [I3-]=A/εb=0,205/(240000 M-1.cm-1)(1,1 cm)=7,76.10-7 M Số mol O3=Vmẫu.[I3-]=(0,01 L)(7,76.10-7 mol/L)=7,76.10-9 mol Số mol của mẫu không khí=PV/RT=P(tlấy mẫuF)/RT =(750torr)(30min)(0,250L/min)/(62,4torr.L.mol-1.K-1)(298K)=0,302 mol Nồng độ O3 (ppb)=(7,76.10-9mol/0,302mol).109=25,7 Câu 6: (2,0 điểm): Quan hệ giữa hiệu ứng cấu trúc và tính chất

1

1,5

a) Hãy vẽ công thức các đồng phân lập thể ứng với cấu tạo A. 0,25 điểm: F

F

F

F

F F

F F

(A2)

(A1)

F

(A3)

F F F

(A4)

b) Ứng với công thức cấu tạo B có bao nhiêu đồng phân lập thể, vì sao? Dùng các kí hiệu thích hợp để chỉ rõ cấu hình của mỗi đồng phân đó. 0,25 điểm B có 3C bất đối, không có mặt phẳng và tâm đối xứng nên có 8 đồng phân lập thể. ví dụ: Cấu hình của B1 như chỉ ra trong bảng, viết gọn là (1R)-(2R)-(4R). Et

Me 1

O

2

O

3 4

5

Me

O

Me

B1

B2

B3

B4

B5

B6

B7

B8

C1

R

S

S

R

S

R

R

S

C2

R

S

R

S

S

R

S

R

C4

R

S

R

S

R

S

R

S

c) Hãy viết cơ chế phản ứng để giải thích vì sao C và D khi tương tác với dung dịch NaOH thì đều tạo thành natri 3-metyl-4-nitrobenzoat. 0,25 điểm: C , C1 , D và D1 là những đồng phân hỗ biến, xúc tác kiềm làm thuận lợi cho sự hỗ biến đó:


HO

O

O

O

O

O

O

-

H2O/- OH

OH /- H2O

O

-

( C1 ) Me

Me

( C)

Me

Me

NO2

NO2

NO2

NO2 O

O

O

H2O/- OH

Me

( D1 ) O

O

O

( D) O

Me

Me NO2

OH

-

Me NO2

NO2

NO2

0,25 điểm: Xuất phát từ C , C1 , D hoặc D1 qua phản ứng chuyển vị benzylic rồi tự mất nước đều chuyển thành hợp chất thơm bền vững, đều dẫn đến cùng một sản phẩm, ví dụ: O

OH

O

O

O

O OH

OHMe (D)

Me

NO2

O

COOH

HO

COO-

COO H - H 2O

Me NO2

Me NO2

Me

Me NO 2

NO 2

NO 2

d) Hãy chỉ rõ trạng thái lai hóa của từng nguyên tử N ở cấu tạo E và ghi giá trị pKa ở 25 oC: 1,8; 6,0; 9,2 vào từng trung tâm axit trong công thức tương ứng với E, giải thích. 0,5 điểm: sp 2 N

6,0

COOH sp 3 N sp 2

H

COOH 1,8

H N

NH 2

N

(E )

NH 3 9,2

H

- Nguyên tử N nhóm NH ở trạng thái lai hóa sp2, cặp e chưa chia ở obitan p xen phủ với 5 obitan p khác tạo thành hệ thơm được lợi về mặt năng lượng nhưng “mất” tính bazơ. - Nguyên tử N thứ hai ở trạng thái lai hóa sp2, cặp e chưa chia ở obitan sp2 không tham gia vào hệ thơm nên còn tính bazơ.

- Nhóm NH3+ là axit liên hợp của nhóm H2Nsp3 , nhóm NH+ là axit liên hợp của nhóm Nsp2. - Bazơ càng mạnh thì axit liên hợp càng yếu, vì thế giá trị 9,2 là thuộc nhóm NH3+ còn giá trị 6,0 thì thuộc nhóm NH+.

- Nguyên tử N nhóm NH2 ở trạng thái lai hóa sp3. 0,5

2 Khong tho'm: F, G, H, J

Tho'm: (I)

EtOOC EtOOC

O

EtOOC +K

COOEt

EtOOC

O

K+

+ 1/ 2H2 COOEt

EtOOC EtOOC

O COOEt

EtOOC EtOOC

O

COOEt


1 0 eπ 2

(K +)2

+ 2K

Câu 7: (2,0 điểm): Hidrocacbon 1

Mỗi ý 0,2

a.

0,2x6 =1,2

b.

+

H+

C

-H+ H

+

E

H

c.

H COOH COOH H F d.

Br

e.

Li+ J

f.

Br

Br

Br

H _

Br

.. CHCl K

Br


N 2

Br

G Br

Sai một chất trừ 0,1

I2

I1

H

Tổng 0,8.

K Br

Br

Br

L1

Br

L2

L3 Br

X

Câu 8: (2,0 điểm): Xác định cấu trúc

Có 11 chất: sai 1 chất trừ 0,2 điểm. Không biện luận trừ 0,5 điểm A,B có k =6 và là một lacton vòng thơm, tan trong NaOH, không tác dụng với NaHCO3, phản ứng màu với FeCl3 nên có nhóm -OH phenol. A phản ứng với MeI/K2CO3 tạo C vậy C đã thêm một nhóm metyl nên OH phenol chuyển thảnh Ome. C có ba nhóm metyl không giống nhau, có một nhóm liên kết trực tiếp với vòng benzen. Phản ứng ozon phân H thu được nhiều sản phẩm trong đó có metyl-2-oxo-propanoat nên H là:


ÔMe

MeO

H Xét phản ứng của E ta có các chất F,G, H như sau:

C loại một nhóm metyl tạo D, D có một nhóm hidroxi tạo liên kết hidro nội nên C có nhóm cacbonyl cạnh nhóm -Ome. Do đó C là (II) , P là (I), từ đó xác định được cấu tạo của B và D Metyl hóa B được P nên có hai cấu tạo B thỏa mãn là B1, B2


A là đồng phân của D nên A khác D ở vị trí nhóm hidroxi và metoxi, cấu tạo phù hợp A:

Câu 9: (2,0 điểm): Cơ chế phản ứng 1

0,5


1,5

2

Sản phẩm có tính bazo yếu hơn anilin vì cặp đôi electron tự do trên N bị giải tỏa nhiều hơn. Nó có khả năng phản ứng mạnh hơn piridin nhưng kém pirol. Phản ứng thế xảy ra như vị trí mũi tên:

Câu 10: (2,0 điểm): Tổng hợp các hợp chất hữu cơ 1 .


Giai đoạn chuyển từ B thành C là giai đoạn chuyển vị Johnson -Claisen và gỡ bỏ nhóm bảo vệ. Cơ chế giai đoạn chuyển D thành E là:

s

2



HỘI CÁC TRƯỜNG CHUYÊN VÙNG DUYÊN HẢI VÀ ĐỒNG BẰNG BẮC BỘ TRƯỜNG THPT CHUYÊN HOÀNG VĂN THỤ TỈNH HÒA BÌNH

ĐỀ THI MÔN HÓA HỌC - KHỐI 11 NĂM 2016- 2017 Thời gian làm bài: 180 phút (Đề này có 10 câu; gồm 04 trang)

ĐỀ THI ĐỀ XUẤT Câu 1. (2 điểm) Tốc độ phản ứng – Cân bằng hóa học 1. Tại 200C, phản ứng: H2 (khí) + Br2 (lỏng Kp= 9,0 .1016.

2 HBr (khí) (1) có hằng số cân bằng

a) Tính Kp của phản ứng: H2(khí) + Br2(khí)

2 HBr(khí) (2) tại 20OC và áp suất

p Br2 (k ) = 0, 25 atm.

b) Hãy cho biết sự chuyển dịch cân bằng hoá học của phản ứng (2) nếu giảm thể tích bình phản ứng ở hai trường hợp: - Trong bình không có Br2 (lỏng). - Trong bình có Br2 (lỏng). 2.Khi ®o vo cña ph¶n øng : .

I- + OCl-

Cl-

+ OI- diÔn ra ë 25oC

Trong dung dÞch cã pH cè ®Þnh vµ c¸c nång ®é I- , OCl- kh¸c nhau ng−êi ta thu ®−îc c¸c kÕt qu¶ sau : [I-]o

( 10-3 mol.l-1 )

1

1

1

1, 1

1,3

[OCl-]o ( 10-3 mol.l-1 )

1

1,2

1,4

1

1

6,1

7,3

8,5

6,7

7,9

v

( 10-5 mol.l-1 )

Chøng minh r»ng c¬ chÕ sau ®©y phï hîp víi c¸c d÷ kiÖn thùc nghiÖm : k1 OCl- + H2O

HOCl + OH-

( nhanh )

k-1 k2 HOCl + I-

HOI

+

Cl-

( chËm )


k3 HOI

+ OH-

+ IO-

H2O

( nhanh )

Câu 2. (2 điểm) Cân bằng trong dung dịch điện li 1. Axit photphoric là một axit ba chức. Nếu chuẩn độ một dung dịch H3PO4 0,1M với NaOH 0,1M. Tính pH tại các thời điểm sau: a) Giữa điểm bắt đầu và điểm tương đương thứ nhất. b) Tại điểm tương đương thứ hai. Cho K1 = 7,1.10-3

K2 = 6,2.10-8

K3 = 4,4.10-13

2. Một dung dịch chứa 530 milimol thiosunfat và một lượng chưa xác định kali iodua. Chuẩn độ dung dịch này với bạc nitrat, đã dùng 20 milimol bạc nitrat trước khi bắt đầu vẩn đục. Tính số mol KI. Biết thể tích phản ứng là 200ml. Ag(S2O3)23-

Ag+ + 2S2O32- (aq)

Ag+ (aq) + I- (aq)

AgI

Kd = 6.10-14 Ksp = 8,5.10-17

Câu 3. (2 điểm) Điện hóa học Cho giản đồ Latime của các hợp chất của nitơ: HNO2

1,00

NO

1,50

N2O

1,77

N2

-1,87

NH3OH+

?

NH4+

0,275

Trên các mũi tên là các thế khử chuẩn. Thí dụ, thế khử chuẩn của phản ứng HNO2 + H+ + e → NO + H2O , E0 = 1,00 V. Các hợp chất được xếp theo chiều tăng dần số oxi hóa của nitơ. 1. Điền giá trị của thế khử chuẩn vào vị trí có dấu chấm hỏi (?). 2. Trị số của thế khử chuẩn (tìm được ở trên) sẽ bằng bao nhiêu nếu dung dịch có pH = 3,00 và nhiệt độ 298o K. 3. Cân bằng phương trình phản ứng hình thành N2 từ HNO2 và NH4Cl, dung dịch có pH = 0 và tính hằng số cân bằng của phản ứng này ở 298o K. Câu 4. (2 điểm) Nhóm phi kim


1. Một nguyên tố X có thể tạo được nhiều oxit axit. Lấy muối natri của axit có chứa X phân tích thì thấy: Muối

Na (%)

X(%)

O (%)

1

32,4

21,8

45,1

2

36,5

24,6

38,1

3

20,7

27,9

50,5

4

26,1

5,2

36,4

Xác định công thức phân tử, viết công thức cấu tạo của các muối trên. 2. Y cấu tạo nên từ hidro và nitơ. Khối lượng của 2,462 lít khí Y ở 27,30C và 0,5 atm bằng khối lượng của 1,68 lít oxi đo ở 54,60C và 0,8 atm. a. Xác định công thức phân tử và tên của Y. Viết công thức Lewis của Y. Cho biết trạng thái lai hóa của N trong Y? Từ đó mô tả cấu trúc không gian của Y. b. Thực nghiệm cho biết trong nước Y có khả năng điện li 2 nấc tạo thành dung dịch có tính bazơ. Giải thích tạo sao Y có tính bazơ. Viết các nấc điện li của Y? c. Viết phương trình phản ứng của Y tác dụng lần lượt với: dung dịch HCl, dung dịch kali hipoclorit, Dung dịch KMnO4 trong H2SO4 loãng. 3. Nêu hiện tượng và viết phương trình phản ứng xảy ra khi tiến hành các thí nghiệm sau:( không cần tính toán) a. Thêm dần dung dịch NaNO2 vào dung dịch I2 (trong dung dịch KI) cho đến dư. Thêm từng giọt dung dịch HCl vào dung dịch thu được cho đến khi dung dịch chuyển màu. b. Dung dịch A có chứa các ion S2-0,01M; Cl- 0,01M. Cho dung dịch AgNO3 từ từ vào dung dịch A, sau đó thêm tiếp KCN từ từ đến dư vào dung dịch thu được. Cho T (AgCl) = 10-10; T (Ag2S) = 10-19,7; hằng số bền của phức Kb ([Ag(CN)2]-) = 10-21,1. Câu 5. (2 điểm) Phức chất Axit A gồm 3 nguyên tố. Cho một thanh kim loại vào bình chứa 75,0 gam dung dịch nước của axit A 16,4% . Kết quả thu được 0,672 dm3 (đktc) khí hiđro và dung dịch


chứa một muối. Khi thêm một lượng dư AgNO3 vào dung dịch thì có 25,81 gam kết tủa trắng tạo thành. Chất kết tủa này chứa 75,26% Ag về khối lượng. Không có sự thay đổi về khối lượng của thanh kim loại trong quá trình phân tích. 1. Hãy xác định kết tủa trắng và công thức axit A. 2. Thanh kim loại làm bằng kim loại gì? 3. Viết tất cả các phương trình phản ứng đã xảy ra. Câu 6. (2 điểm) Quan hệ cấu trúc – tính chất 1. Hợp chất H tham gia vào thành phần của một số alkaloid. Chất H được tổng hợp theo sơ đồ sau: C H3

O

O

O

xt

C H 2C F 3

o

A

t ,p

1 . L iA lH 4 2 . H 2O

B

1 . O sO 4 2 . N aIO 4

C (C 9 H 1 4 O 4 )

C H3

D

PCC

1. C sO H

E

2 . M e O H /H + pH = 4

I , Ph P

2 3 F im id a z o l (C 1 1 H 1 8 O 4 )

G

P h 3P = C H 2

1 . B H 3 .T H F 2 . H 2O 2, N a O H 3. Z n/ A cO H

Xác đinh công thức cấu tạo của các hợp chất từ A đến H. Biết C không chứa vòng 6 cạnh. 2. Metyl da cam là chất chỉ thị màu axit-bazơ có công thức: 1

(H3C)2N

2

3

N N

SO3Na

So sánh tính bazơ của các nguyên tử N? Giải thích. . Câu 7. (2 điểm) Hiđrocacbon Tecpenoit lµ c¸c hîp chÊt cã m¹ch cacbon gåm c¸c ®¬n vÞ isopren liªn kÕt ®Çu-®u«i víi nhau. Geraniol vµ nerol ®Òu lµ tecpenoit vµ lµ hai ®ång ph©n lËp thÓ cña rîu bËc mét cã hai liªn kÕt ®«i trong ph©n tö. Khi cã mÆt axit, geraniol khÐp vßng t¹o thµnh α-tecpineol, cßn nerol còng khÐp vßng cho cïng s¶n phÈm nhng x¶y ra nhanh h¬n nhiÒu. 1. ViÕt c«ng thøc cÊu tróc, tªn IUPAC cña nerol, geraniol vµ gi¶i thÝch v× sao nerol dÔ khÐp vßng h¬n? 2 . α-tecpineol khi läai níc t¹o thµnh tecpinolen. X¸c ®Þnh cÊu tróc cña tecpinolen 3 .Trong m«i trêng axit, tecpinolen ®ång ph©n hãa t¹o thµnh ba ®ång ph©n dien m¹ch mét vßng gäi chung lµ tecpinen. Trong sè nµy, chØ cã s¶n phÈm chÝnh lµ α-tecpinen cho ph¶n øng Diels-Alder. Tuy nhiªn c¶ ba ®ång ph©n t¸c dông víi hidro clorua d ë nhiÖt ®é thÊp cho cïng mét s¶n phÈm.

D

H


a.ViÕt c«ng thøc cÊu tróc vµ tªn IUPAC cña α-tecpinen. b.ViÕt ph¬ng tr×nh ph¶n øng cña α-tecpinen víi anhidrit maleic (anhidrit butendioic) c.ViÕt ph¬ng tr×nh ph¶n øng cña α-tecpinen víi anhidrit maleic (anhidrit butendioic) Câu 8. (2 điểm) Xác định cấu trúc 1) Hợp chất A( C7H10O4) không tác dụng với H2/Pd, to. A bị thủy phân trong môi trường axit đun nóng cho B (C4H8O2), A tác dụng với LiAlH4, sau đó thủy phân trong môi trường H+ thu được C (C5H10O3), C bị oxi hóa bởi K2Cr2O7/ H2SO4 thu được D (C5H6O5). Trong môi trường H+/to, D chuyển thành E (C3H6O), C tác dụng với H2/Ni cho F không quang hoạt. Xác định công thức cấu tạo từ A-F 2)Hợp chất A (C11H17NO3) không quang hoạt, không tan trong môi trường trung tính và kiềm nhưng dễ tan trong môi trường axit loãng. A có hai nguyên tử H linh động, A phản ứng với Ac2O tạo B(C13H19NO4) trung tính. A phản ứng với MeI dư sau đó thêm AgOH, sản phẩm thu được C có công thức là C14H25NO4. Đun nóng chất này thu được Me3N và D(C11H14O3) trung tính. D phản ứng với O3 thu được HCHO và E . Andehit thơm E phản ứng HI tạo sản phẩm chứa 3 nhóm –OH mà chúng không tạo được liên kết hidro nội phân tử bền vững. a) Xác định các chất chưa biết b) Từ E và các hợp chất vô cơ, hãy điều chế chất A Câu 9. (2 điểm) Cơ chế 1.Enamin có thể được tạo thành khi cho anđehit hoặc xeton phản ứng với amin bậc hai có xúc tác axit. Xiclohexanon phản ứng với piroliđin tạo ra enamin H theo sơ đồ sau:

H N

+

H

+

N

H

O

Đề xuất cơ chế giải thích quá trình tạo thành enamin H. 2. Viết công thức cấu tạo của A và trình bày cơ chế của hai giai đoạn phản ứng. b.

A

to 3,3

B

CCl4 FeCl3

C

tBuONa (C6H6)

D

KOH EtOH

HOOC

Cl E

Cl


Viết công thức cấu tạo của B, C, D và trình bày cơ chế phản ứng B → C và C → D. Câu 10. (2 điểm) Tổng hợp hữu cơ 1. Bupivacain (C18H28N2O) là amit của axit 1-butylpiperiđin-2-cacboxylic với 2,6đimetylanilin ở dạng S được dùng làm thuốc gây tê cục bộ. Tổng hợp (S)-bupivacain từ 2-metylpiriđin và các hóa chất cần thiết khác. 2. Cho s¬ ®å chuyÓn ho¸ c¸c chÊt sau:

Cl O

(AlCl3)

+

- HCl

H2SO4

A

B -HO 2 HO

HOO O X

+

Y

X

Cl D1 D2

(A, B, X, Y, D1 , D2 lµ c¸c hîp chÊt h÷u c¬) ViÕt c«ng thøc cÊu t¹o cña A, B, X, Y, D1 , D2

O

+

Cl (MnO4- )

X


HỘI CÁC TRƯỜNG CHUYÊN VÙNG DUYÊN HẢI VÀ ĐỒNG BẰNG BẮC BỘ TRƯỜNG THPT CHUYÊN HOÀNG VĂN THỤ TỈNH HÒA BÌNH

ĐỀ THI MÔN HÓA HỌC - KHỐI 11 NĂM 2016- 2017 Thời gian làm bài: 180 phút (Đề này có 10 câu; gồm 04 trang)

ĐÁP ÁNĐỀ THI ĐỀ XUẤT Câu 1. (2 điểm) Tốc độ phản ứng – Cân bằng hóa học Câu1

Hướng dẫn

Điểm

1a. a) Phản ứngH2 (k) + Br2 (lỏng) ⇌ 2 HBr (k) Kp1 =

(2)

P (a) PH 2

còn phản ứng: H2 (k)+ Br2 (k) ⇌ 2 HBr (k) Kp 2 =

(1)

2 HBr

2 HBr

P

PH 2 .PBr2

0,25

(b)

Xét cân bằng Br2 (lỏng) ⇌ Br2 (k)

(3) có Kp3 = PBr2 ( k ) (c)

Khi tổ hợp (1) với (3) ta có cân bằng (2): H2 (k)+ Br2 (lỏng) ⇌ 2 HBr (k)

(1)

Br2 (l) ⇌ Br2 (k)

(3)

(1) – (3): H2 (k)+ Br2 (k) ⇌ 2 HBr (k)

(2)

Kp 2 =

Kp1 9, 0.1016 = = 3, 6.1017 (atm) Kp3 0, 25

Khi giảm thể tích bình phản ứng nghĩa là tăng áp suất riêng phần của khí trong hệ. Xét Q =

2 PHBr PH 2 .PBr2

Trường hợp 1: Không có brom lỏng trong bình: Phản ứng (2) có tổng số mol khí trước và sau phản ứng bằng nhau (∆n = 0) nên sự thay đổi áp suất đó không dẫn tới chuyển dịch cân bằng (2).

0,25

0,25

Trường hợp 2: Có brom lỏng trong bình: áp suất riêng phần của các khí H2, HBr tăng; trong lúc đó áp suất riêng phần của Br2 khí lại không đổi do còn Br2 lỏng. Theo (d), vì số mũ của PHBr lớn hơn số mũ của PH2 nên sự tăng áp suất nói trên dẫn đến sự tăng Q và cân bằng (2) chuyển dịch theo chiều nghịch.

0,25


2b.

Gi¶ sö ph¶n øng cã bËc : v = k . [I-]x . [OCl-]y XÐt 3 thÝ nghiÖm ®Çu ta cã : v = k . [1]x . [OCl-]y = k .[OCl-]y , gi¶ sö y = 1 Víi thÝ nghiÖm 1 : k1 =6,1 . 10-2 s-1 Víi thÝ nghiÖm 2 : k2 =6,08 . 10-2 s-1 Víi thÝ nghiÖm 3 : k3 =6,07 . 10-2 s-1 XÐt 2 thÝ nghiÖm sau : v = k . [I-]x . [1]y = k . [I-]x , gi¶ sö x = 1 Víi thÝ nghiÖm 4 : k4 =6,09 . 10-2 s-1 Víi thÝ nghiÖm 5 : k5 =6,08 . 10-2 s-1 Ta thÊy : k1 ~ k2 ~ k3 ~ k4 ~ k5 , gi¶ thiÕt lµ ®óng . VËy , ph−¬ng tr×nh ®éng häc lµ : v = k . [I-] . [OCl-]

0,5

XÐt c¬ chÕ , giai ®o¹n chËm quyÕt ®Þnh tèc ®é ph¶n øng :

k1 . k2 .[I-].[OCl-].[H2O] v = k2. [HOCl].[I-] = k-1 . [OH-] Do pH cè ®Þnh nªn [OH-] cè ®Þnh , dung dÞch rÊt lo·ng , H2O kh«ng tham gia vµo ph−¬ng tr×nh ph¶n øng , [H2O] = 1000 / 18 = 55,3 VËy , v = k . [I-] . [OCl-] víi :

k1 . k2 .[H2O] k = k-1 . [OH-]

0,25

Câu 2. (2 điểm) Cân bằng trong dung dịch điện li Câu 2 1

Hướng dẫn

Điểm

a) Tại điểm bắt đầu và điểm tương đương thứ nhất ta có dung dịch đệm H3PO4 và H2PO4- và [H3PO4] = [H2PO4-] H3PO4 [H+] = K 1

H+ + H2PO4-

[ H 3 PO4 ] → [H+] = K1 = 7,1.10-3 → pH = 2,15 [ H 2 PO4− ]

b) Tại điểm tương đương thứ hai có HPO42H2PO4-

H+ + HPO42-

K2 = 6,2.10-8 (1)

0,5


HPO42-

H+ + PO43-

K3 = 4,4.10-13 (2)

K 2 .K 3 = 6,2.10 −8.4,4.10 −13 = 1,7.10-10

Tổ hợp (1) và (2) → [H+] =

0,5

→pH = 9,77. 2

Ta có: Ag+ + 2S2O32- (aq) 20 → 2.20

Ag(S2O3)2320

Kd-1 = 1,667.1013

milimol

Ta thấy hằng số tạo phức rất lớn nên hầu hết Ag+ thêm vào sẽ tạo phức với S2O32-. [Ag(S2O3)23-] = 20/200 = 0,1M → Số mol S2O32- tự do là: 530 – 2.20 = 490 mmol [S2O32-] = 490/200 = 2,45M Tính nồng độ

Ag+

tự do:

Ag(S2O3)23Kd =

0,5

Ag+ + 2S2O32- (aq)

Kd = 6.10-14

[ Ag + ].[ S 2 O32− ] 2 = 6.10-14 →[Ag+] = 10-15 [ Ag ( S 2 O3 ) 32− ]

Khi bắt đầu vẩn đục ta có: Ag+ + I- → AgI (r ) → [I-] =

K sp [ Ag + ]

=

8,5.10 −17 = 8,5.10-2M 10 −15

→ số mol của KI = 8,5.10-2. 200 = 17 mmol 0,5

Câu 3. (2 điểm) Điện hóa học Câu 3

Hướng dẫn

Điể m

1. E 0 (NH 3OH + /NH +4 ) =

n1 = 1

n2 = 3

n 2 . E 0 (N 2 /NH 4+ ) - n1 . E 0 (N 2 /NH 3OH + ) n3

n3 = 2

E 0 (NH 3OH + /NH 4+ ) =

3 . 0,275 V - (-1,87 V) = 1,35 V 2

0,5


2. NH3OH+ + 2e + 2H+ → NH4+ + H2O E = E0 -

RT 8,314 J / K . mol . 298 K ln (H + ) -2 = 1,35 V ln(1,0 . 10-3 ) -2 = 1,18 V 4 nF 2 . 96,5 . 10 C/mol

0,5

3. HNO2 + NH4Cl → N2 + 2 H2O + HCl HNO2 + 3 H+ + 3e → 1/2 N2 + H2O

∆G01

NH4+ → 1/2 N2 + 4 H+ + 3 e

∆G02 ∆G0 = ∆G01 + ∆G02

HNO2 + NH4Cl → N2 + 2 H2O + HCl

∆G01 = (-n1E0(HNO2 / NO) – n2E0(NO / N2O) – n3E0( N2O / N2)) . F n1 = n2 = n3 = 1

∆G01 = (- 1,00 V – 1,50 V – 1,70 V) . 9,65 . 104 C/mol = -405,3 kJ/mol ∆G02 = nFE0(N2 / NH4+)

n=3

∆G02 = 3 . 9,65 . 104 C/mol . 0,275 V = 79,6 kJ/mol ∆G0 = ∆G01 + ∆G02 = -405,3 kJ/mol + 79,6 kJ/mol = -326 kJ/mol K=e

- ∆G 0 RT

=e

- (-326.103 J/mol) 8,314 J/K.mol . 298K

= 1,4.10

57

1,0

Câu 4. (2 điểm) Nhóm phi kim Câu14

Hướng dẫn

Điểm

1. . Các muối đều có: %mNa+ %mX + %mO < 100% muối có H. - Muối 1: Gọi SOX của X là a. BT SOX: 1.32,4/23 + a.21,8/X + (-2).45,1/16 + 1.0,7/1 = 0 Mà a = 1; 2; 3; 4; 5; 6; 7 chỉ a = 5 X = 31: P thỏa mãn. Muối 1: Na : P : O : H = 2:1:4:1

Na2HPO4.

0,125


-

Muối 2: Na2HPO3.

0,125

-

Muối 3: Na2H2P2O7

0,125 O

ONa

HO

O P

OH

-

P ONa

O

0,125

Muối 4: NaH2PO2. NaO

H

P

O

H

2

a.MX = 32. Gọi X là NxHy N2H4: hidrazin. H H N

N H H

N: lai hóa sp3. tứ diện ghép đôi: N(1) là tâm của tứ diện gồm các đỉnh là H(1); H(2); N(2) và có đôi e chưa liên kết của N(1). N(2) là tâm tứ diện gồm các đỉnh H(3); H(4); N(1) và có đôi e chưa liên kết của N(2).

b. Mỗi nguyên tử N trong hidrazin còn một đôi e chưa liên kết do đó có khả năng nhận proton tính bazơ và phân li theo 2 nấc: N2H4 + H2O ⇄ N2H5+ + OH-. N2H5+ + H2O ⇄ N2H62+ + OH-.

0,5


c. N2H4 + HCl N2H5Cl hay N2H4 + 2HCl N2H6Cl2. N2H4 + 2KClO 2KCl + N2 + 2H2O 5N2H4 + 4KMnO4 + 6H2SO4 4MnSO4 + 2K2SO4 + 5N2 + 16H2O. 0,5

0,5 3.

a. Dung dịch I2 (trong KI) có màu nâu, khi thêm NaNO2 dung dịch mất màu do: NO2- + I3- + H2O NO3- + 3I- + 2H+. Thêm HCl dung dịch chuyển lại màu nâu đồng thời có khí không màu thoát ra sau đó lại hóa nâu trong không khí do: NO2- + H+ HNO2. 2HNO2 + 2I- + 2H+ 2NO + I2 + 2H2O. 0,25

NO + ½ O2 NO2.

b. Có kết tủa màu đen xuất hiện trước, sau đó đến kết tủa màu trắng. 2Ag+ + S2- Ag2S. Ag+ + Cl- AgCl. Khi thêm KCN vào AgCl tan tạo phức, còn lại màu đen do Ag2S không tan trong KCN: AgCl + 2KCN K[Ag(CN)2] + KCl

0,25

Câu 5. (2 điểm) Phức chất Câu1

Hướng dẫn

1a.

1. AgnX

Điểm


%(Ag)=

n × A r (Ag) = 0,7526 n × A r (Ag) + A r (X)

=>

A r (X) = 35,46n

Kết quả thích hợp là : n = 1 và X : Cl. Kết tủa trắng là AgCl. Axit A chứa H, Cl và nguyên tố chưa biết Y.

N(Cl) = N(AgCl) = 25,81 g ×

1 mol = 0,18 mol 143,3 g

N(H) = 2N(H 2 ) = 2 × 0,672 dm 3 ×

1 mol = 0,06 mol 22,4 dm 3

Vì N(H) : N(Cl) = 1 : 3, A : HxYyCl3x Phản ứng tổng quát : m HxYyCl3x + 3x M = mx/2 H2 + 3x MClm + my Y

M(A) = 75 g × 0,164 × Nếu x = 1, A r (Y) = Nếu y = 1, Y : Tc,

x = 205x g/mol 0,06 mol

205 - 1,008 - 3 × 35,45 97,642 = y y Nếu y = 2, Y : Ti,

...

Cả hai kết quả đều thích hợp với giả thiết Nếu x = 2, A r (Y) = Nếu y = 1, Y : Pt,

2 × 97,642 195,284 = y y Nếu y = 2, Y : Tc,

Nếu y = 3, Y :

Tc, ...

1,0

A : H2[PtCl6] (Kết quả khác không phù hợp với đề bài). 2b.

2.

6M + mH2[PtCl6] = 6MClm + mH2↑ + mPt↓

Vì không có sự thay đổi khối lượng của thanh kim loại:

m(M) = m(Pt) =

1 195,08 g × 0,06 mol × = 5,8524 g 2 1 mol

M(M) = 5,8524 g ×

1 = 32,51m g/mol 6/m×0,03 mol


Nếu m = 2, M : Zn

3.

(Giá trị khác không phù hợp với đề bài)

0,5

3Zn + H2[PtCl6] = 3ZnCl2 + H2↑ + Pt↓ 0,5

ZnCl2 + 2AgNO3 = Zn(NO3)2 + 2AgCl Câu 6. (2 điểm) Quan hệ cấu trúc – tính chất Câu1 6

Hướng dẫn

Điể m

1 Me

O

Me O

O

OH

C H 2C F 3

O

O

Me

Me

A

B

O O

CHO

HO

O HO

C

OH

D

I

O O COOM e

O O E

2.

O COOMe

O COOMe

F

H

G

1,0

Cặp electron không liên kết của N(1) có hiệu ứng +C vào vòng benzen nên N(1) có mật độ electron thấp hơn so với 2 nguyên tử nitơ còn lại. Trong các công thức cộng hưởng ta nhận thấy N(3) có mật độ electron cao nhất do điện tích âm được nằm trên nó trong khi N(2) thì không. N(3) có mật độ điện tích âm cao nhất, nó dễ dàng nhận H+ nhất so với 2 nguyên tử nitơ còn lại. Do đó N(3) có tính bazơ mạnh nhất. (hay hs có thể vẽ hiệu ứng (A, E))

O

O N

N N

1

S ONa O

N

N N

A

N

B

N N

E

S ONa O

O S ONa O

N

O S ONa O

N N

D

1,0


Câu 7. (2 điểm) Hiđrocacbon Câu 17

Hướng dẫn

Điể m

1 3 2

HO

2

3

HO

Nerol (Z)-3,7-dimetylocta-2,6-dien-1-ol

Geraniol (E)-3,7-dimetylocta-2,6-dien-1-ol

Nerol cã cÊu h×nh Z t¹i liªn kÕt ®«i 2,3 nªn c¸c nguyªn tö tham gia ph¶n øng khÐp vßng ë gÇn nhau h¬n lµm cho ph¶n øng diÔn ra nhanh: + 1. H HO

+

2. -H2O

+

OH2 + OH2

nerol

0,7 5

hoÆc

0,2 5

2. -H2O

tecpinolen OH

3.

a. Trong m«i trêng axit, tecpinolen ®ång ph©n hãa t¹o thµnh ba ®ång ph©n dien m¹ch mét vßng gäi chung lµ tecpinen. Trong sè nµy, chØ cã s¶n phÈm chÝnh lµ α-tecpinen cho ph¶n øng Diels-Alder. Tuy nhiªn c¶ ba ®ång ph©n t¸c dông víi hidro clorua d ë nhiÖt ®é thÊp cho cïng mét s¶n phÈm. ViÕt c«ng thøc cÊu tróc vµ tªn IUPAC cña -tecpinen V× à-tecpinen tham gia ph¶n øng Diels-Alder nªn ph¶i lµ mét dien liªn hîp. C«ng thøc cÊu t¹o phï hîp cña α-tecpinen:


0,5 α-tecpinen

b. ViÕt ph¬ng tr×nh ph¶n øng cña α-tecpinen víi anhidrit maleic (anhidrit butendioic) O +

O O

0,2 5

O

O

O

α-tecpinen

c. ViÕt c«ng thøc cÊu tróc cña hai tecpinen cßn l¹i?

0,2 5

Câu 8. (2 điểm) Xác định cấu trúc Câu 8

Hướng dẫn

1

O

O

O

OH

OH OH

O

O

B

C

Điể m 1,0

COOHCOOH

O

A

O

O

D

E

OH

F

2b.

a. E là andehit thơm , E phản ứng HI thu được sản phẩm có 3 nhóm –OH không tạo liên kết hidro nôi phân tử nên:


CHO OMe

OMe

OMe

OMe

E

D OMe

OMe

C nhiệt phân tạo D nên C có hai công thức là:

A không quang hoạt nên C không quang hoạt nên công thức cấu tạo của C là 1 Vậy A và B là: NH2

NHAc

OMe

OMe

A

OMe

OMe

0,5

B OMe

OMe

b. Sơ đồ tổng hợp A từ E ( coi E là R-CHO) 0,5

Câu 9. (2 điểm) Cơ chế Câu 9

Hướng dẫn

Điể m

1

1,0 H

O

O +H

+

..

HN

HO NH

H2O N

N

-H2O

H H2O

N +

-H3O


2

3

b. 3 2

EtO

O 1

2

to

CCl4

1

3,3

FeCl3

EtO

A

B

Cl EtO

O

O C

tBuONa (C6H6)

CCl3

KOH

Cl

CCl3

EtOOC

CCl3

EtOH

D

1,0

HOOC

E

Câu 10. (2 điểm) Tổng hợp hữu cơ Câu 10

Hướng dẫn

Điể m

1

1.

N

CH3

H2O

SOCl2

H2/Ni

KMnO4 N

N

COOH

COOH

H

H

N+

COCl

H (S)-Bup

NH2 H3C

CH3

H N H

N O CH3

H

CH3 C4H9Br piridin

N

N

C4H9 O CH3

CH3 Tách 2 ® b»ng axi

1,0


2b.

H CH2

CH2 CH

O A

OH

H

B

X

OH

HO H

H H

H

OH

HO

D1

D2

1,0


HỘI CÁC TRƯỜNG CHUYÊN VÙNG DUYÊN HẢI VÀ ĐỒNG BẰNG BẮC BỘ TRƯỜNG THPT CHUYÊN LÀO CAI

ĐỀ THI MÔN HÓA HỌC – KHỐI 11 NĂM 2017 Thời gian làm bài: 180 phút (Đề này có 04 trang, gồm 10 câu)

ĐỀ THI ĐỀ XUẤT Câu 1(2 điểm): Tốc độ phản ứng – Cân bằng hóa học 1. Để nghiên cứu động học của phản ứng 2[Fe(CN)6]3− + 2I−

2[Fe(CN)6]4− + I2 (*).

Người ta đo tốc độ đầu của sự hình thành iot ở 4 hỗn hợp dưới đây. Các hỗn hợp ban đầu không chứa iot. c([Fe(CN)6]3−) mol/L

c(I−) mol/L

c([Fe(CN)6]4−) mol/L

Tốc độ đầu mmol.L−1. h−1

Thí nghiệm 1

Hỗn hợp 1

1

1

1

1

Thí nghiệm 2

Hỗn hợp 2

2

1

1

4

Thí nghiệm 3

Hỗn hợp 3

1

2

2

2

Thí nghiệm 4

Hỗn hợp 4

2

2

1

16

Trong trường hợp tổng quát, tốc độ phản ứng được biểu thị bởi phương trình:

dc(I2 ) = k.ca([Fe(CN)6]3−).cb(I−).cd([Fe(CN)6]4−).ce(I2) dt Xác định giá trị của a, b, d, e và hằng số tốc độ phản ứng k. 2. Cơ chế sau đây đã được đề xuất cho phản ứng (*): [Fe(CN)6]3− + 2 I−

k1

[Fe(CN)6]4− + I2−

k−1 k2 [Fe(CN)6]4− + I2

(1)

[Fe(CN)6]3− + I2− (2) a. Trong 2 phản ứng trên, phản ứng nào diễn ra nhanh, phản ứng nào diễn ra chậm? b. Chứng minh rằng cơ chế trên phù hợp với phương trình biểu diễn tốc độ phản ứng tìm được ở 1. Câu 2(2 điểm): Cân bằng trong dung dịch điện li Axit HIn trong nước phân ly như sau: HIn (màu 1) H+ + In− (màu 2) Đo mật độ quang của dung dịch HIn 5,00.10−4 M trong NaOH 0,1 M và trong HCl 0,1 M ở bước sóng 485 nm và 625 nm với cuvet 1,00 cm. Trong dung dịch NaOH 0,1 M

A485 = 0,052

Trong dung dịch HCl 0,1 M A485 = 0,454

A625 = 0,823 A625 = 0,176

1. Tính hệ số hấp thụ mol của In− và HIn ở bước sóng 485 và 625 nm. 2. Tính hằng số phân ly axit của HIn, nếu trong dung dịch đệm pH 5,00 chứa một lượng nhỏ chất chỉ thị có mật độ quang là 0,472 ở 485 nm và 0,351 ở 625 nm 3. Tính pH của một dung dịch chứa một lượng nhỏ chất chỉ thị. Biết mật độ quang là 0,530 ở 485 nm và 0,216 ở 625 nm. 4. Mật độ quang của một dung dịch HIn 2,00.10−4 M tại 485 và 625 nm (cuvet 1,25 cm) là bao nhiêu nếu dung dịch được đệm ở pH bằng 6,00?


5. Chuẩn độ 25,00 mL dung dịch axit hữu cơ tinh khiết yếu HX với chất chỉ thị là phenolphtalein tới điểm cuối chuẩn độ thì cần 24,20 mL dung dịch NaOH chuẩn. Khi thêm đúng 12,10 mL dung dịch NaOH vào 25,00 mL dung dịch axit HX, trong dung dịch có một lượng nhỏ chất chỉ thị HIn, mật độ quang đo được là 0,306 ở 485 nm và 0,555 ở 625 nm (cuvet 1,00 cm). Tính pH của dung dịch và Ka cho axit yếu. Câu 3(2 điểm): Điện hóa học Cho giản đồ quá trình khử - thế khử: quá trình khử diễn ra theo chiều mũi tên, thế khử chuẩn được ghi trên các mũi tên và đo ở pH = 0. 1. Tính E 0x và E 0y . 2. Dựa vào tính toán, cho biết Cr(IV) có thể dị phân thành Cr3+ và Cr(VI) được không? 3. Viết quá trình xảy ra với hệ oxi hóa – khử Cr2 O72- /Cr3+ và tính độ biến thiên thế của hệ ở nhiệt độ 298 K, khi pH tăng 1 đơn vị pH. 4. Phản ứng giữa K2Cr2O7 với H2O2 trong môi trường axit (loãng) được dùng để nhận biết crom vì sản phẩm tạo thành có màu xanh. Viết phương trình ion của phản ứng xảy ra và cho biết phản ứng này có thuộc loại phản ứng oxi hóa – khử hay không? Vì sao? Ghi số oxi hóa tương ứng trên mỗi nguyên tố. 0 Cho: E Cr O2- /Cr3+ = 1,33 V; Hằng số khí R = 8,3145 J.K–1.mol–1; Hằng số Farađay F = 96485 C.mol–1. 2 7

Câu 4(2 điểm): Nhóm N – P, nhóm C – Si 1. Sục khí (A) vào dung dịch (B) có màu nâu vàng thu được chất rắn (C) màu vàng và dung dịch (D). Khí (X) có màu vàng lục tác dụng với khí (A) tạo ra (C) và (F). Nếu (X) tác dụng với khí (A) trong nước tạo ra (Y) và (F), rồi thêm BaCl2 vào dung dịch thì có kết tủa trắng. Khí (A) tác dụng với dung dịch chất (G) là muối nitrat kim loại tạo ra kết tủa (H) màu đen. Đốt cháy (H) bởi oxi ta được chất lỏng (I) màu trắng bạc. Xác định A, B, C, F, G, H, I, X, Y và viết phương trình hóa học của các phản ứng. 2. Cho một dung dịch X chứa các ion: Na+, Cl-, Br-, CO32 − , HCO3− , SO32− , SO 24− , NO3− . Trình bày phương pháp hóa học để nhận biết từng ion trong dung dịch X. Câu 5(2 điểm): Phức chất 1. Người ta đã tổng hợp được [NiSe4]2- , [ZnSe4]2- và xác định được rằng phức chất của Ni có dạng hình vuông phẳng, của Zn có dạng hình tứ diện đều. Hãy đưa ra một cấu tạo hợp lí cho mỗi trường hợp trên và giải thích. 2. Phức chất [PtCl2(NH3)2] được xác định là đồng phân trans-. Nó phản ứng chậm với Ag2O cho phức chất [PtCl2(NH3)2(OH2)2]2+(kí hiệu là X). Phức chất X không phản ứng được với etylenđiamin (en) khi tỉ lệ mol phức chất X : en = 1 : 1. Hãy giải thích các sự kiện trên và vẽ (viết) cấu tạo của phức chất X. 3. [Ru(SCN)2(CN)4]4– là ion phức của ruteni, được kí hiệu là P. a. Viết công thức Lewis của phối tử thioxianat SCN–. b. Cho biết dạng lai hóa của Ru trong P. Mô tả sự hình thành ion phức theo thuyết VB (Valence Bond). Giải thích tại sao trong P, liên kết được hình thành giữa Ru và N của phối tử SCN– mà không phải là giữa Ru và S. Cho biết phức có tính thuận từ hay nghịch từ, vì sao? Câu 6(2 điểm): Quan hệ cấu trúc – tính chất 1. Hãy sắp xếp các chất sau theo chiều tăng dần tính bazơ, giải thích ? H 2N

NH2

O 2N

NH2

NH2

NH2

CN

NO

NO 2

CN

NH 2

O 2N 2

NO 2 NO 2


(I)

2. Cho 4 axit:

(II)

(III)

(IV)

(V)

(VI)

CH3CH2COOH (A); CH3COCOOH (B); CH3COCH2COOH (C)

CH3CH(+NH3)COOH (D). Sắp xếp A, B, C, D theo trình tự tăng dần tính axit. Giải thích. Câu 7(2 điểm): Hiđrocacbon 1. Các nhà khoa học đầu những năm 1900 cho rằng chỉ cần có hệ liên hợp trong một vòng kín thì phân tử đó có tính thơm.Xiclooctatetraen (A) đã làm sụp đổ nhận định này. Nó lần đầu tiên được điều chế bởi nhà hóa học người Đức Richard Willstater năm 1911. a. Vẽ công thức của xiclooctatetraen, nó có phẳng không? Nếu không, hãy vẽ các dạng tồn tại của nó. b. Xiclooctatetraen tác dụng với 2 đương lượng Kali tạo ra dianion. Viết cấu tạo của dianion này và cho biết tính thơm của nó? Dianion này phản ứng với 2 mol axeton cho 2 sản phẩm C và D. Cho biết công thức cấu tạo của C và D. c. Xiclooctatetraen phản ứng với m-CPBA tạo ra chất E,xử lí E với axit Brontest thu được chất G (C8H8O) có phản ứng với thuốc thử Tollens. Xác định công thức cấu tạo các chất chưa biết. d. Hoàn thành dãy phản ứng sau biết K là chất trung gian.

2. Một ankin D quang hoạt chứa 89,52% C. Hợp chất D có thể bị hidro hóa/xúc tác tạo nbutylxiclohexan. Xử lí D với EtMgBr không thấy thoát khí. Hidro hóa D trên xúc tác Pd/C tronng quinolin( chất đầu độc xúc tác) và xử lí sản phẩm với O3/H2O2 cho axit tricacboxylic quang hoạt E (C8H12O6). Chất E khi đun nóng tách một phân tử nước và tạo F. Viết công thức cấu tạo các chất chất Câu 8(2 điểm): Xác định cấu trúc

1. Xác định cấu tạo các chất và hoàn thành sơ đồ phản ứng trên? 2. Viết cơ chế từ C D? Câu 9(2 điểm): Cơ chế


Hoàn thành các cơ chế phản ứng sau: O

O

Et

MeO Et

to

1.

Et

Et

MeO

O O

2.

3.

4. Câu 10(2 điểm): Tổng hợp hữu cơ 1. Từ β-metylnaptalen hãy tổng hợp A :

2.Từ phenol và các chất hữu cơ khác hãy tổng hợp thuốc giảm đau Profofol (B) ( được cho là thủ phạm gây ra cái chết của vua nhạc pop Michael Jackson):

Giáo viên biên soạn đề: Phạm Duy Đông


SĐT: 0977119268.


HỘI CÁC TRƯỜNG CHUYÊN VÙNG DUYÊN HẢI VÀ ĐỒNG BẰNG BẮC BỘ TRƯỜNG THPT CHUYÊN LÀO CAI

HDC ĐỀ THI MÔN HÓA HỌC – KHỐI 11 NĂM 2017 Thời gian làm bài: 180 phút (HDC này có 13 trang, gồm 10 câu)

ĐỀ THI ĐỀ XUẤT Câu 1(2 điểm): Tốc độ phản ứng – Cân bằng hóa học 1. Để nghiên cứu động học của phản ứng 2[Fe(CN)6]3− + 2I−

2[Fe(CN)6]4− + I2 (*).

Người ta đo tốc độ đầu của sự hình thành iot ở 4 hỗn hợp dưới đây. Các hỗn hợp ban đầu không chứa iot. c([Fe(CN)6]3−) mol/L

c(I−) mol/L

c([Fe(CN)6]4−) mol/L

Tốc độ đầu mmol.L−1. h−1

Thí nghiệm 1

Hỗn hợp 1

1

1

1

1

Thí nghiệm 2

Hỗn hợp 2

2

1

1

4

Thí nghiệm 3

Hỗn hợp 3

1

2

2

2

Thí nghiệm 4

Hỗn hợp 4

2

2

1

16

Trong trường hợp tổng quát, tốc độ phản ứng được biểu thị bởi phương trình:

dc(I2 ) = k.ca([Fe(CN)6]3−).cb(I−).cd([Fe(CN)6]4−).ce(I2) dt

Xác định giá trị của a, b, d, e và hằng số tốc độ phản ứng k. 2. Cơ chế sau đây đã được đề xuất cho phản ứng (*): [Fe(CN)6]3− + 2 I−

k1

[Fe(CN)6]4− + I2−

k−1 k2 [Fe(CN)6]4− + I2

(1)

[Fe(CN)6]3− + I2− (2) a. Trong 2 phản ứng trên, phản ứng nào diễn ra nhanh, phản ứng nào diễn ra chậm? b. Chứng minh rằng cơ chế trên phù hợp với phương trình biểu diễn tốc độ phản ứng tìm được ở 1. HDC 1. Từ thí nghiệm 1 và 2 a=2 Từ thí nghiệm 1 và 3 d = -1 Từ thí nghiệm 1 và 4 b=2 e=0

dc(I2 ) = k.c2([Fe(CN)6]3−).c2(I−).c−1([Fe(CN)6]4−).c0(I2) dt

Thí nghiệm 1: 1.10−3 mol.L−1. h−1 = k . 1 mol2.L−2.1 mol2.L−2.(1 mol.L−1) −1 K = 1.10−3 mol−2.L2. h−1 2. k1 [Fe(CN)6]3− + 2 I− [Fe(CN)6]4− + I2− k−1 k2 [Fe(CN)6]4− + I2

0,5

(1)

[Fe(CN)6]3− + I2− (2) (1) diễn ra nhanh; (2) diễn ra chậm. Nếu chấp nhận như vậy thì chứng minh được rằng cơ chế đề xuất phù hợp với định luật tốc độ rút ra từ thực nghiệm. Chứng minh: Từ cân bằng (1) được xác lập rất nhanh:


c([Fe(CN)6 ]4- ) × c(I2 - ) k1 = k -1 c([Fe(CN)6 ]3- ) × c2 (I- ) Phản ứng (2) diễn ra chậm:

c(I2−) =

k1 c([Fe(CN)6 ]3- ) × c2 (I- ) × k -1 c([Fe(CN)6 ]4- )

(a)

dc(I ) 2 = k2 × c([Fe(CN)6]3−) × c(I2−) (b) dt

Thay (a) vào (b) ta được: dc(I ) c 2 ([Fe(CN)6 ]3- ) × c 2 (I- ) 2 = k1 × k2 × c([Fe(CN)6 ]4- ) k -1 dt Kết quả này phù hợp với kết quả thực nghiệm. Nếu phản ứng (1) là chậm, phản ứng (2) nhanh: dc(I ) 2 = k2.c([Fe(CN)6]3−).c(I2−) và c(I2−) = k1.c([Fe(CN)6]3−).c2(I−) dt dc(I ) 2 = k1.k2.c2([Fe(CN)6]3−).c2(I−). Kết quả này không phù hợp với kết quả thực nghiệm. dt

0,5

0,5

0,5

Câu 2(2 điểm): Cân bằng trong dung dịch điện li Axit HIn trong nước phân ly như sau: HIn (màu 1) H+ + In− (màu 2) Đo mật độ quang của dung dịch HIn 5,00.10−4 M trong NaOH 0,1 M và trong HCl 0,1 M ở bước sóng 485 nm và 625 nm với cuvet 1,00 cm. Trong dung dịch NaOH 0,1 M

A485 = 0,052

Trong dung dịch HCl 0,1 M A485 = 0,454

A625 = 0,823 A625 = 0,176

1. Tính hệ số hấp thụ mol của In− và HIn ở bước sóng 485 và 625 nm. 2. Tính hằng số phân ly axit của HIn, nếu trong dung dịch đệm pH 5,00 chứa một lượng nhỏ chất chỉ thị có mật độ quang là 0,472 ở 485 nm và 0,351 ở 625 nm 3. Tính pH của một dung dịch chứa một lượng nhỏ chất chỉ thị. Biết mật độ quang là 0,530 ở 485 nm và 0,216 ở 625 nm. 4. Mật độ quang của một dung dịch HIn 2,00.10−4 M tại 485 và 625 nm (cuvet 1,25 cm) là bao nhiêu nếu dung dịch được đệm ở pH bằng 6,00? 5. Chuẩn độ 25,00 mL dung dịch axit hữu cơ tinh khiết yếu HX với chất chỉ thị là phenolphtalein tới điểm cuối chuẩn độ thì cần 24,20 mL dung dịch NaOH chuẩn. Khi thêm đúng 12,10 mL dung dịch NaOH vào 25,00 mL dung dịch axit HX, trong dung dịch có một lượng nhỏ chất chỉ thị HIn, mật độ quang đo được là 0,306 ở 485 nm và 0,555 ở 625 nm (cuvet 1,00 cm). Tính pH của dung dịch và Ka cho axit yếu. HDC Trong dung dịch NaOH, chất chỉ thị chỉ tồn tại chủ yếu ở dạng In−; Ttrong dung dịch axit, chất chỉ thị chỉ tồn tại chủ yếu ở dạng HIn−.


Theo định luật Beer: 1. Vì cuvet đều có bề dày dd là 1cm nên A = εi.[i]. − Trong dung dịch NaOH:

In In A485 = ε 485 .C = 0,052 → ε 485 = 104 In In A625 = ε 625 .C = 0,823 → ε 625 = 1646

− Trong dung dịch HCl:

HIn HIn A485 = ε 485 .C = 0,454 → ε 485 = 908

A625 = ε

HIn 625

.C = 0,176 → ε

HIn 625

0,25

= 352

2. Ở pH = 5. HIn In A485 = ε 485 .[HIn] + ε 485 .[In−] = 908. [HIn] + 104 .[In−] = 0,472 HIn In A625 = ε 625 .[HIn] + ε 625 .[In−] = 352. [HIn] +1645 .[In−] = 0,351

Giải hệ phương trình tìm được: [HIn] = 5,078.10−4 → Ka =

[In−] = 1,047.10−4.

1, 047.10−4.10−5 = 2,06. 10−5 5, 078.10−4

0,25

3. Tính pH: HIn In A485 = ε 485 .[HIn] + ε 485 .[In−] = 908. [HIn] + 104 .[In−] = 0,530 HIn In A625 = ε 625 .[HIn] + ε 625 .[In−] = 352. [HIn] +1645 .[In−] = 0,216

Giải hệ phương trình tìm được: [HIn] = 5,83.10−4 → [H+] =

[In−] = 6,57.10−6.

5,83.10−4.2, 06.10−5 = 1,82. 10−3 → pH = 2,738 6,57.10−6

4. Ở pH = 6,000 [HIn] = 2.10−4 .

10 −6 = 9,26 . 10−6 (M) −6 −4,686 10 + 10

[In−] = 2.10−4 .

10−4,686 = 1,9074 . 10−4 (M) −6 −4,686 10 + 10

HIn In A485 = ε 485 .[HIn] + ε 485 .[In−] = 908. 9,26 .10−6 + 104 . 1,9074 . 10−4 = 0,028 HIn In A625 = ε 625 .[HIn] + ε 625 .[In−] = 352. 9,26 .10−6 +1645. 1,9074 . 10−4 = 0,330

5. − Chuẩn độ tới đổi màu phenolphtalein cần 24,20 mL dung dịch bazơ → khi thêm 12,10 mL thì dung dịch chứa HX và X− với hai lượng tương đương. → Áp dụng công thức có thể suy ra pKa: pKa = pH + lg

[HX ] = pH. [X − ]

HIn In .[HIn] + ε 485 .[In−] = 908. [HIn] + 104 .[In−] = 0,306 A485 = ε 485

0,5


HIn In A625 = ε 625 .[HIn] + ε 625 .[In−] = 352. [HIn] +1645 .[In−] = 0,555

Giải hệ phương trình tìm được: [HIn] = 3,06.10−4

[In−] = 2,72.10−4.

3,06.10−4.2,06.10−5 → [H ] = = 2,3175. 10−3 → pH = 4,635 → pKa (HX) = 4,635. −4 2, 72.10 +

0,5

Câu 3(2 điểm): Điện hóa học Cho giản đồ quá trình khử - thế khử: quá trình khử diễn ra theo chiều mũi tên, thế khử chuẩn được ghi trên các mũi tên và đo ở pH = 0. 1. Tính E 0x và E 0y . 2. Dựa vào tính toán, cho biết Cr(IV) có thể dị phân thành Cr3+ và Cr(VI) được không? 3. Viết quá trình xảy ra với hệ oxi hóa – khử Cr2 O72- /Cr3+ và tính độ biến thiên thế của hệ ở nhiệt độ 298 K, khi pH tăng 1 đơn vị pH. 4. Phản ứng giữa K2Cr2O7 với H2O2 trong môi trường axit (loãng) được dùng để nhận biết crom vì sản phẩm tạo thành có màu xanh. Viết phương trình ion của phản ứng xảy ra và cho biết phản ứng này có thuộc loại phản ứng oxi hóa – khử hay không? Vì sao? Ghi số oxi hóa tương ứng trên mỗi nguyên tố. 0 Cho: E Cr O2- /Cr3+ = 1,33 V; Hằng số khí R = 8,3145 J.K–1.mol–1; Hằng số Farađay F = 96485 C.mol–1. 2 7

HDC 1. Từ giản đồ ta có: 3.(-0,744) = -0,408 + 2 E 0y → E 0y = -0,912 (V) 0,55 + 1,34 + E 0x – 3.0,744 = 6.0,293 → E 0x = +2,1 (V)

0,5

2. Cr(IV) có thể dị phân thành Cr3+ và Cr(VI) khi ∆G0 của quá trình < 0. 2Cr(IV) + 2 e → 2Cr3+ (1) E10 = E 0x = 2,1 V → ∆G10 = -n E10 F = - 2.2,1.F Cr(VI) + 2 e → Cr(IV) (2)

E 02 =

0,55 + 1,34 = 0,945 (V) → ∆G 02 = -n E 02 F = 2

2.0,945.F Từ (1) và (2) ta có: 3Cr(IV) → 2Cr3+ + Cr(VI)

∆G 30

∆G 30 = ∆G10 - ∆G 02 = - 2.(2,1 - 0,945).F < 0 → Vậy Cr(IV) có dị phân.

3.

Cr2 O72- + 14H+ + 6e

2Cr3+ + 7H2O

RT [Cr2 O72- ].(10-pH )14 E1 = 1,33 + ln 6.F [Cr 3+ ]2 RT [Cr2 O72- ].(10-(pH + 1) )14 E 2 = 1,33 + ln 6.F [Cr 3+ ]2

0,5


b. Độ biến thiên của thế: E 2 - E1 =

4.

+6 -2

0,5

8,3145 . 298 .14ln10-1 = -0,138 (V). 6 . 96485

+1 -1

+1

+6,-2/-1

+1 -2

Cr2 O72- + 4H2O2 + 2H+ → 2CrO5 + 5H2O

Phản ứng trên không phải là phản ứng oxi hóa-khử vì số oxi hóa của các nguyên tố không 0,5 thay đổi trong quá trình phản ứng. Trong CrO5, số oxi hóa của crom là +6 và của oxi là -2, -1 do peoxit CrO5 có cấu trúc:

Câu 4(2 điểm): Nhóm N – P, nhóm C – Si 1. Sục khí (A) vào dung dịch (B) có màu nâu vàng thu được chất rắn (C) màu vàng và dung dịch (D). Khí (X) có màu vàng lục tác dụng với khí (A) tạo ra (C) và (F). Nếu (X) tác dụng với khí (A) trong nước tạo ra (Y) và (F), rồi thêm BaCl2 vào dung dịch thì có kết tủa trắng. Khí (A) tác dụng với dung dịch chất (G) là muối nitrat kim loại tạo ra kết tủa (H) màu đen. Đốt cháy (H) bởi oxi ta được chất lỏng (I) màu trắng bạc. Xác định A, B, C, F, G, H, I, X, Y và viết phương trình hóa học của các phản ứng.

2. Cho một dung dịch X chứa các ion: Na+, Cl-, Br-, CO32 − , HCO3− , SO32− , SO 24− , NO3− . Trình bày phương pháp hóa học để nhận biết từng ion trong dung dịch X.

HDC 1. A: H2S; B: FeCl3; F: HCl; G: Hg(NO3)2; I: Hg; X: Cl2; Phương trình hóa học của các phản ứng : H2S + 2FeCl3 → 2FeCl2 + S + 2HCl Cl2 + H2S → S + 2HCl 4Cl2 + H2S + 4H2O → 8HCl + H2SO4

C: S; H: HgS; Y: H2SO4 (1) (2) (3)

BaCl2 + H2SO4 → BaSO4 ↓ + 2HCl

(4)

H2S + Hg(NO3)2 → HgS ↓ + 2HNO3

(5)

t0

HgS + O2 → Hg + SO2 2. Sơ đồ nhận biết:

1,0

(6)

0,5


0

daâ y Pt, t

löû a maø u vaø ng

Na+

(X) + (CH3COO)2Ba dö

(Na+, Cl-, Br-, HCO3-, NO3-, Ba2+, CH3COO-) (BaCO3, BaSO3, BaSO4) + CH3COOH dö (Na+, Cl-, Br-,NO3-, Ba2+, CH3COOH, CH3COO-) CO2 + Cu +

-

-

2+

2+

HCO3-

+ HCl dö

BaSO4

(CO2, SO2) dd Br2 dö

SO42-

-

(Na , Cl , Br ,Ba , Cu , CH3COOH, CH3COO ) NO +AgNO3 dö NO 2 (AgCl, AgBr) +NH3 dö

(SO2: mm) SO32-

CO2 +Ca(OH)2 dö keát tuûa

NO3CO32-

[Ag(NH3)2]Cl +HNO3

AgBr: vaø ng Br-

AgCl: traé ng Cl-

Phương trình phản ứng xảy ra: CO32 − + Ba2+ → BaCO3 SO32− + Ba2+ → BaSO3

0,5

SO 24− + Ba2+ → BaSO4

BaCO3 + 2HCl → BaCl2 + CO2 + H2O BaSO3 + 2HCl → BaCl2 + SO2 + H2O BaSO4 + 2HCl → không phản ứng SO2 + Br2 + 2H2O → 2HBr + H2SO4 CO2 + Ca(OH)2 → CaCO3 + H2O CH3COOH + NaHCO3 → CH3COONa + CO2 + H2O 3Cu + 2NaNO3 + 8CH3COOH → 3(CH3COO)2Cu + 2NO + 2CH3COONa + 4H2O 2NO + O2 → 2NO2 NaBr + AgNO3 → AgBr + NaNO3 NaCl + AgNO3 → AgCl + NaNO3 AgCl + 2NH3 → [Ag(NH3)2]Cl [Ag(NH3)2]Cl + 2HNO3 → AgCl + 2NH4NO3

Câu 5(2 điểm): Phức chất 1. Người ta đã tổng hợp được [NiSe4]2- , [ZnSe4]2- và xác định được rằng phức chất của Ni có dạng hình vuông phẳng, của Zn có dạng hình tứ diện đều. Hãy đưa ra một cấu tạo hợp lí cho mỗi trường hợp trên và giải thích. 2. Phức chất [PtCl2(NH3)2] được xác định là đồng phân trans-. Nó phản ứng chậm với Ag2O cho phức chất [PtCl2(NH3)2(OH2)2]2+(kí hiệu là X). Phức chất X không phản ứng được với etylenđiamin (en) khi tỉ lệ mol phức chất X : en = 1 : 1. Hãy giải thích các sự kiện trên và vẽ (viết) cấu tạo của phức chất X. 3. [Ru(SCN)2(CN)4]4– là ion phức của ruteni, được kí hiệu là P. a. Viết công thức Lewis của phối tử thioxianat SCN–.


b. Cho biết dạng lai hóa của Ru trong P. Mô tả sự hình thành ion phức theo thuyết VB (Valence Bond). Giải thích tại sao trong P, liên kết được hình thành giữa Ru và N của phối tử SCN– mà không phải là giữa Ru và S. Cho biết phức có tính thuận từ hay nghịch từ, vì sao? HDC 1. Niken có mức oxi hoá phổ biến nhất là +2; kẽm cũng có mức oxi hoá phổ biến nhất là +2. Selen có tính chất giống lưu huỳnh do đó có khả năng tạo thành ion polyselenua Se 22− hay [-Se— Se-]2-. Cấu tạo vuông phẳng của phức chất [NiSe4]2- là do cấu hình electron của ion Ni2+ cho phép sự lai hoá dsp2. Cấu tạo tứ diện đều của phức chất [ZnSe4]2- là do cấu hình electron của Zn2+ cho phép sự lai hoá 0,5 sp3. Tổng hợp của các yếu tố trên cho phép đưa ra cấu tạo sau đây của 2 phức chất: Se

Se

Se

Ni Se

Zn Se

Se

Se

Se

trong đó ion điselenua đóng vai trò phối tử 2 càng.

2. [PtCl2(NH3)2] (1) là đồng phân trans- đòi hỏi phức chất phải có cấu tạo vuông phẳng: Cl

│ H3N—Pt—NH3

(1)

│ Cl -

Phản ứng của (1) với Ag2O: Trans-[PtCl2(NH3)2] + Ag2O + H2O → Trans-[PtCl2(NH3)2(H2O)2]2+ + 2OH-

-

Etylenđiamin là phối tử hai càng mạch ngắn. Khi phối trí với các ion kim loại nó chỉ chiếm 2 vị trí phối trí cạnh nhau (vị trí cis). Hiện tượng en không thể phản ứng với [PtCl2(NH3)2(H2O)2]2+ theo phản ứng: [PtCl2(NH3)2(H2O)2]2+ + en → [PtCl2(NH3)2(H2O)2en]2+ + 2H2O

chứng tỏ rằng 2 phân tử H2O nằm ở 2 vị trí trans đối với nhau. Như vậy công thức cấu tạo của phức chất phải là: H2O Cl

NH3

Pt

NH3

Cl H2O

3a. Tổng số electron để xây dựng công thức Lewis cho SCN– là 6 + 4 + 5 + 1 = 16. Công thức

0,5


Lewis của SCN– là:

S C N 0 0 3b. Ru có cấu hình electron [Kr]4d 5s 5p , là ion trung tâm trong phức bát diện. 2+

0,5

6

Vì CN– là phối tử trường mạnh nên ở phân lớp 4d6 của Ru2+ có sự ghép đôi tất cả các electron, tạo ra 2 AO 4d trống. Do đó xảy ra sự lai hóa d2sp3 để tạo 6AO lai hóa hướng tới 6 đỉnh của 1 hình bát diện. Các phối tử (L) sử dụng cặp electron tự do của nguyên tử N gửi vào các obitan lai hóa đó để tạo các liên kết cho nhận giữa phối tử và ion Ru2+. d2sp3 [Ru(SCN)2(CN)4]4-

4d6

L

5s ...

5p

L

So với S, N có độ âm điện lớn hơn và bán kính nguyên tử nhỏ hơn, do đó mật độ điện tích âm trên nguyên tử N sẽ lớn hơn, ái lực phản ứng với ion dương Ru2+ lớn hơn, vì vậy trong phức 0,5 chất P, liên kết phức được hình thành giữa Ru và N mà không phải là giữa Ru và S. Phức P có tính nghịch từ vì trong ion phức không có electron độc thân.

Câu 6(2 điểm): Quan hệ cấu trúc – tính chất 1. Hãy sắp xếp các chất sau theo chiều tăng dần tính bazơ, giải thích ?

O 2N

NH2

H 2N

NH2

NH2

NH2

CN

NO

NO 2

O 2N

CN

(I)

2. Cho 4 axit:

NH 2

(II)

(III)

(IV)

(V)

2

NO 2 NO 2

(VI)

CH3CH2COOH (A); CH3COCOOH (B); CH3COCH2COOH (C)

CH3CH(+NH3)COOH (D). Sắp xếp A, B, C, D theo trình tự tăng dần tính axit. Giải thích. HDC

1. Sắp xếp: I < VI < V < IV < III < II

0,5

Giải thích: Tính bazơ của N càng giảm khi có mặt các nhóm có hiệu ứng –C càng mạnh. Hiệu ứng –C của NO2 > CN. - Xiclopentadienyl chỉ có hiệu ứng –I. –I làm giảm tính bazơ kém hơn –C - Các hợp chất I và VI đều có 2 nhóm NO2 ở vị trí meta so với nhóm NH2 gây ra hiệu ứng không gian làm cản trở sự liên hợp–C của nhóm NO2ở vị trí para nhiều hơn nhóm CN ở vị 0,5 trí para. Do đó hiệu ứng – C nhóm CN ở vị trí 4 > nhóm NO2 ở vị trí 4

2. Tính axit tăng dần: A < C < B < D Giải thích:

0,5


O

O O CH3CH2

CH3

C

CH2

C O

H

O

O

CH3 CH

C

H

NH3

A

O

H

-I m¹nh

-I

+I

C

B

C

D

0,5

Câu 7(2 điểm): Hiđrocacbon 1. Các nhà khoa học đầu những năm 1900 cho rằng chỉ cần có hệ liên hợp trong một vòng kín thì phân tử đó có tính thơm.Xiclooctatetraen (A) đã làm sụp đổ nhận định này. Nó lần đầu tiên được điều chế bởi nhà hóa học người Đức Richard Willstater năm 1911. a. Vẽ công thức của xiclooctatetraen, nó có phẳng không? Nếu không, hãy vẽ các dạng tồn tại của nó. b. Xiclooctatetraen tác dụng với 2 đương lượng Kali tạo ra dianion. Viết cấu tạo của dianion này và cho biết tính thơm của nó? Dianion này phản ứng với 2 mol axeton cho 2 sản phẩm C và D. Cho biết công thức cấu tạo của C và D. c. Xiclooctatetraen phản ứng với m-CPBA tạo ra chất E,xử lí E với axit Brontest thu được chất G (C8H8O) có phản ứng với thuốc thử Tollens. Xác định công thức cấu tạo các chất chưa biết. d. Hoàn thành dãy phản ứng sau biết K là chất trung gian.

2. Một ankin D quang hoạt chứa 89,52% C. Hợp chất D có thể bị hidro hóa/xúc tác tạo nbutylxiclohexan. Xử lí D với EtMgBr không thấy thoát khí. Hidro hóa D trên xúc tác Pd/C tronng quinolin( chất đầu độc xúc tác) và xử lí sản phẩm với O3/H2O2 cho axit tricacboxylic quang hoạt E (C8H12O6). Chất E khi đun nóng tách một phân tử nước và tạo F. Viết công thức cấu tạo các chất chất HDC 1.a.

Xiclooctatetraen không phẳng và tồn tại ở 2 dạng: ghế và thuyền.

0,25

b. Dianion thơm do thỏa mãn các quy tắc Hulkel: các e thảo mãn quy tắc e = 4n + 2, phẳng, có hệ e liên hợp kín.

0,25

c. Cơ chế.


0,5

0,5

d. 0,5

2. Câu 8(2 điểm): Xác định cấu trúc


1. Xác định cấu tạo các chất và hoàn thành sơ đồ phản ứng trên? 2. Viết cơ chế từ C D?

HDC

COOMe

HN COOH

COOH

S

S

NH

N

COOMe

COOt-Bu

S

N

N

MeOOC

COOMe

S

COOt-Bu

N

COOt-Bu

0,5 A

D

C

B

Cl3CH2COOC

O

HO

COOMe

N3

COOMe H

S

N

COOt-Bu

S

E

CHO

O

NH

N

H H

N

COOt-Bu

S

N

COOt-Bu

H

F

H S

N

CHO

0,5

COOt-Bu

I

0,5 Cơ chế từ C sang D: COOMe MeOOC N

H COOMe S

S

N COOMe

Câu 9(2 điểm): Cơ chế

1.

N

N

COOMe

N

HN S

COOMe

COOMe S

N COOMe

NH

0,5


2.

3.

4. HDC

1.

0,5

2.

0,5


HO - H+

H+ - H2O

3.

A

0,5

B

C

4. 0,5

Câu 10(2 điểm): Tổng hợp hữu cơ 1. Từ β-metylnaptalen điều chế A :

2.Từ phenol và các chất hữu cơ khác hãy tổng hợp thuốc giảm đau Profofol B( được cho là thủ phạm gây ra cái chết của vua nhạc pop Michael Jackson):


HDC O

O

PCC

1,O3

0

t

1.

O

2,H2O2

O

1,0

OH

O 1,NaOH

2.

OH t0

2,Crotyl clorua

O 1,NaOH 2,Crotyl clorua

1,0

*********************************

HẾT **********************************



TRƯỜNG ĐẠI HỌC SƯ PHẠM HÀ NỘI TRƯỜNG THPT CHUYÊN ĐHSP ĐỀ NGUỒN KÌ THI HỌC SINH GIỎI VÙNG DUYÊN HẢI VÀ ĐỒNG BẰNG BẮC BỘ NĂM 2017 MÔN THI: HÓA HỌC LỚP 11 Thời gian làm bài: 180 phút (không kể thời gian phát đề) Câu 1: (2 điểm) Động học (Cơ chế phản ứng)-Cân bằng hóa học Tốc độ phản ứng thủy phân trong môi trường axit:

không phụ thuộc [H+] được mô tả bởi phương trình động học:v = có hằng số tốc độ: k = 2,4.10-4 (250C) và k = 8,1.10-4 (350C) đối với phức chất dạng cis; k = 3,5.10-4 (250C) và k = 1,5.10-4 (350C) đối với phức chất dạng trans. a. Mô tả sự thủy phân Co(en)2Cl22+ bằng cơ chế 2 giai đoạn ( sử dụng các hằng số tốc độ k1, k-1, k2) và chứng minh cơ chế đưa ra là phù hợp với thực nghiệm v=

b. Vẽ các cấu trúc có thể có của tiểu phân trung gian trong cơ chế của TransCo(en)2Cl22+. c. Tính năng lượng hoạt hóa EA cho cis- và trans- Co(en)2Cl2+. Tính entanpi ∆H#, entropi ∆S# cho sự hình thành phức hoạt động của cis- Co(en)2Cl22+.

Câu 2: (2 điểm) Cân bằng dung dịch điện li Dung dịch X gồm Na2S 0,010M, KI 0,060M, Na2SO4 0,050M. a. Tính pH của dung dịch X. b. Thêm dần Pb(NO3)2 vào dung dịch X cho đến nồng độ 0,090M thì thu được kết tủa A và dung dịch B, không kể sự thuỷ phân của các ion, coi thể tích dung dịch không thay đổi khi thêm Pb(NO3)2. i. Cho biết thành phần hoá học của kết tủa A và dung dịch B. ii. Tính nồng độ các ion trong dung dịch B.


iii. Nhận biết các chất có trong kết tủa A bằng phương pháp hoá học, viết các phương trình phản ứng (nếu có). c. Axit hoá chậm dung dịch X đến pH = 0, coi thể tích dung dịch không thay đổi. Thêm FeCl3 cho đến nồng độ 0,10M. i. Tính thế của cực platin nhúng trong dung dịch thu được so với cực calomen bão hoà (Hg2Cl2/2Hg,2Cl-). ii. Biểu diễn sơ đồ pin, viết phương trình phản ứng xảy ra tại các điện cực và phản ứng tổng quát khi pin hoạt động. Cho : axit có H2S pK1 = 7,00, pK2 = 12,90; HSO4- có pKa = 2,00; Tích số tan của Ks(PbS) = 10-26 ; Ks(PbSO4) = 10-7,8 ; Ks(PbI2) = 10-7,6. Eo Fe3+/Fe2+ = 0,77 V ; Eo S/H2S = 0,14V ; Eo I2/2I- = 0,54V ; Ecal bão hoà = 0,244V Câu 3: (2 điểm) Điện hóa học Ắc quy Chì-Axit được sử dụng khá phổ biến trong xe hơi, phản ứng khi phóng điện là Sự phụ thuộc của thế khử chuẩn E0 vào nhiệt độ được biểu diễn bằng đồ thị dưới đây

a. Viết sơ đồ pin khi ắc quy phóng điện. b. Tính ∆G0, ∆H0, ∆S0 của phản ứng trên ở 298K. c. Tính thế điện cực E của phản ứng ở 50C tại pH=5 và nồng độ ion sunfat là 0,10M. Câu 4: (2 điểm) N-P, C-Si và hợp chất


Hydro mới sinh là một tác nhân khử có hiệu quả nhất. Xử lý một lượng natri nitrit bằng hỗn hống natri kim loại cho ra một muối X có 43,38% natri và 26,43% nitơ về khối lượng. Một sản phẩm khác của phản ứng này là natri hydroxit. Để tránh sự làm bẩn sản phẩm cuối này thì quá trình tổng hợp được tiến hành trong khí quyển trơ như môi trường nitơ hay argon. a) Xác định công thức muối X. b) Vẽ công thức Lewis anion của muối X. c) Nếu phản ứng được tiến hành trong không khí thì những tạp chất nào có thể sinh ra? d) Viết phương trình tổng hợp muối X. e) Tương tác giữa X với cacbon dioxit sinh ra một chất khí. Viết phương trình phản ứng. Câu 5: (2 điểm) phức chất, trắc quang a) Phức chất A là phức chất của Pt(II), có cấu trúc vuông phẳng, chứa Pt, NH3 và Cl với % khối lượng lần lượt là 65,00%; 11,33% và 23,67%. Viết công thức cấu trúc của phức chất thỏa mãn điều kiện trên, biết phân tử khối của A nhỏ hơn 1000. b) Khi cho phức chất cis-[Pt(NH3)2Cl2] (cấu trúc vuông phẳng) tương tác với tetrametylen điamin (tetraen) người ta tách ra được một phức chất 2 nhân B của Pt(II) có công thức [Pt2Cl4(NH3)2(tetraen)] (B). Kết quả thử nghiệm cho thấy chất B có khả năng kháng tế bào ung thư cao. Viết công thức cấu tạo, mô tả sự hình thành liên kết (theo thuyết VB) trong phức chất B. c) Khi nghiên cứu cơ chế phản ứng thế phối tử trong phức chất vuông phẳng người ta nhận thấy rằng tốc độ thay thế phối tử X bởi một phối tử Y chịu ảnh hưởng của phối tử Z (Z nằm ở vị trí trans so với X). Nếu Z càng hoạt động về ảnh hưởng trans thì X càng linh động và càng dễ bị thế. L

Z Pt L

+ Y X

L

Z

+

Pt L

X

Y

Từ K2[PtCl4], viết sơ đồ điều chế phức chất cis và phức chất trans-[Pt(C2H4)(2ampy)Cl2] (trong đó 2-ampy là 2-aminopiriđin). Biết thứ tự ảnh hưởng trans của các phối tử: C2H4 > Cl- > 2-ampy. Trong phức chất trên, nguyên tử kim loại trung tâm Pt liên kết với phối tử 2-aminopiriđin qua nguyên tử nitơ nào? Giải thích. 3


Cho biết: Cấu hình electron của Pt: [Xe]4f145d96s1 Câu 6: (2 điểm) Quan hệ giữa hiệu ứng cấu trúc và tính chất a. Sắp xếp các chất sau đây theo chiều tăng dần lực axit, giải thích: O

O

O

O

O

O

H

O OEt

O Cl

O

O

NO 2

CF3

b. Hãy so sánh tính bazơ của các nguyên tử N trong mỗi phân tử các hợp chất sau: O N H

N

O N O NH

N N N CH3

N N

N(CH3)2

COOH

Câu 7: (2 điểm)Hidrocacbon Chất A có công thức phân tử C7H12. Khi thực hiện phản ứng ozon phân A tạo HCHO và xiclohexanon.Thực hiện quá trình chuyển hoá A liên tiếp sau: 1, HBr ; 2, Mg/ ete; 3, CO2 sau đó với H3O+ ta thu được hợp chất B (C8H14O2) .Nếu cho A tác dụng với HBr/ peoxit tiếp theo cho sản phẩm tác dụng với KCN sau đó xử lý bằng dung dịch axit loãng ta thu được chất C (C8H14O2). C cũng có thể tạo ra bằng cách cho sản phẩm của A với HBr/ peoxit tác dụng với Mg/ete, tiếp theo CO2 và xử lý bằng dung dịch axit. a. Xác định cấu trúc các chất trung gian và viết tên A, B, C b. So sánh tính axit của các B,C với các chất D (axit Benzoic) và E (axit Phenyletanoic) ; F(axit 3-phenylpropanoic) Câu 8: (2 điểm) xác định cấu trúc, đồng phân lập thể, danh pháp Tinh dầu tràm được dùng trong sản xuất dược phẩm và mỹ phẩm. Hai hợp phần đóng vai trò chính cho ứng dụng đó là eucaliptol (kí hiệu là A) và một đồng phân cấu tạo của nó kí hiệu là B. Hợp chất A không làm mất màu dung dịch Br2/CCl4. Khi đun nóng A với dung dịch axit sunfuric loãng người ta thu được hợp chất không quang hoạt C kết tinh trong nước thành D công thức C10H20O2.H2O. Đun nóng D với axit photphoric người ta thu được sản phẩm chính là E không quang hoạt và có bộ khung cacbon giống như D (biết rằng có thể dùng phương pháp thích

4


hợp để tách lấy từng đối quang từ E). Sau khi ozon phân chế hóa khử thì từ E và từ B đều thu được 3-(1-hydroxy-1-metyletyl)-6-oxoheptanal. a. Hãy xác định công thức cấu tạo của A, B, C và E. b. Giải thích vì sao B quang hoạt còn E thì không. So sánh có giải thích nhiệt độ nóng chảy, nhiệt độ sôi của A, B, C và E. c. Hãy vẽ công thức phối cảnh của A, C và giải thích vì sao A dễ dàng bị chuyển hóa bởi axit sunfuric loãng thành C, chất này lại dễ kết tinh trong nước thành D. d. Hãy đề nghị một sơ đồ tổng hợp E từ những hợp chất chứa không quá 5C. Có thể dùng sơ đồ đó để tổng hợp B được không, vì sao? e. Hãy đề nghị 2 phương pháp tổng hợp A từ những hợp chất chứa không quá 6C. Câu 9: (2 điểm) Cơ chế phản ứng Giải thích các phản ứng sau bằng cơ chế phản ứng? a. H/H2O

2 OH

b.

c. O

OH OH

-

OH (A)

O (B)


d. CO2H O CO2Me

O DBU, MeOH

MeO2C

DBU: 1,8-Diazabicyclo[5.4.0]undec-7-ene

N N

Câu 10: (2 điểm) Tổng hợp các chất hữu cơ (Dạng sơ đồ phản ứng) (+)-Artemisinin (Quinhaosu) là một hoạt chất tự nhiên có tác dụng chữa bệnh sốt rét mà không gây tác dụng phụ được tách ra từ cây Thanh hao hoa vàng (Artemisia annua L., họ Asteriaceae). Đây là một lacton dạng secquitecpen. Hoạt tính kháng sốt rét của hợp chất này là do nó có cấu trúc cầu endoperoxit. Do nguồn thu tự nhiên hạn chế, trong khi yêu cầu chữa bệnh sốt rét vẫn luôn cấp thiết nên các nhà hóa dược đặt ra vấn đề phải tổng hợp hoặc bán tổng hợp hoạt chất này. Năm 2003, một nhóm nghiên cứu tại viện Hóa kỹ thuật Ấn Độ đã đề xuất một qui trình tổng hợp (+)-Artemisinin từ (+)-Isolimonen theo sơ đồ dưới đây:

Trong sơ đồ trên, PCC là piriđinium clorocromat; KHMDS là kali hexametilen đisilan (hay kali bis(trimetylsilyl) amiđua. Đây là một bazơ rất mạnh, tương tự NaNH2, nhưng cồng kềnh nên không thể hiện tính nucleophin).

6


Hãy cho biết cấu trúc các hợp chất từ A đến I trong sơ đồ tổng hợp (+)Artemisinin trên. Cho: H = 1; C = 12; N = 14; O = 16; Na = 23; Mg = 24,3; Al = 27; P = 31; S = 32; Cl = 35,5; K = 39,1; Ca = 40,1; Ti = 47,9; Cr = 52; Mn = 54,9; Fe = 55,8; Co = 58,9; Ni = 58,7; Cu = 63,5; Zn = 65,4; Ag = 107,9; Ba = 137,3. ---------------------------------------------------

Thí sinh không được sử dụng tài liệu. Cán bộ coi thi không giải thích gì thêm. Giáo viên: Quách Phạm Thùy Trang 0975855880

7


TRƯỜNG ĐẠI HỌC SƯ PHẠM HÀ NỘI TRƯỜNG THPT CHUYÊN ĐHSP ĐỀ NGUỒN KÌ THI HỌC SINH GIỎI VÙNG DUYÊN HẢI VÀ ĐỒNG BẰNG BẮC BỘ NĂM 2017 MÔN THI: HÓA HỌC LỚP 11 Thời gian làm bài: 180 phút (không kể thời gian phát đề) Câu 1: (2 điểm) Động học (Cơ chế phản ứng)-Cân bằng hóa học Câu 1: (2 điểm) Động học (Cơ chế phản ứng)-Cân bằng hóa học Tốc độ phản ứng thủy phân trong môi trường axit:

không phụ thuộc [H+] được mô tả bởi phương trình động học:v = có hằng số tốc độ: k = 2,4.10-4 (250C) và k = 8,1.10-4 (350C) đối với phức chất dạng cis; k = 3,5.10-4 (250C) và k = 1,5.10-4 (350C) đối với phức chất dạng trans. a. Mô tả sự thủy phân Co(en)2Cl22+ bằng cơ chế 2 giai đoạn ( sử dụng các hằng số tốc độ k1, k-1, k2) và chứng minh cơ chế đưa ra là phù hợp với thực nghiệm v=

b. Vẽ các cấu trúc có thể có của tiểu phân trung gian trong cơ chế của TransCo(en)2Cl22+. c. Tính năng lượng hoạt hóa EA cho cis- và trans- Co(en)2Cl2+. Tính entanpi ∆H#, entropi ∆S# cho sự hình thành phức hoạt động của cis- Co(en)2Cl22+.

Hướng dẫn giải a. Cơ chế

Áp dụng phương trình nồng độ dừng cho Co(en)2Cl22+ ta có →


Từ đó rút ra v =

, Nếu

thì v=

(đpcm)

Học sinh có thể làm theo cách hằng số cân bằng vẫn cho điểm tối đa. b. Cấu trúc của tiểu phân trung gian của Trans

c. Năng lượng hoạt hóa Đồng phân Trans: Đồng phân Cis: ;

Câu 2: (2 điểm) Cân bằng dung dịch điện li Dung dịch X gồm Na2S 0,010M, KI 0,060M, Na2SO4 0,050M. a. Tính pH của dung dịch X. b. Thêm dần Pb(NO3)2 vào dung dịch X cho đến nồng độ 0,090M thì thu được kết tủa A và dung dịch B, không kể sự thuỷ phân của các ion, coi thể tích dung dịch không thay đổi khi thêm Pb(NO3)2. i. Cho biết thành phần hoá học của kết tủa A và dung dịch B. ii. Tính nồng độ các ion trong dung dịch B. iii. Nhận biết các chất có trong kết tủa A bằng phương pháp hoá học, viết các phương trình phản ứng (nếu có). c. Axit hoá chậm dung dịch X đến pH = 0, coi thể tích dung dịch không thay đổi. Thêm FeCl3 cho đến nồng độ 0,10M. i. Tính thế của cực platin nhúng trong dung dịch thu được so với cực calomen bão hoà (Hg2Cl2/2Hg,2Cl-). ii. Biểu diễn sơ đồ pin, viết phương trình phản ứng xảy ra tại các điện cực và

phản ứng tổng quát khi pin hoạt động.


Cho : axit có H2S pK1 = 7,00, pK2 = 12,90; HSO4- có pKa = 2,00; Tích số tan của Ks(PbS) = 10-26 ; Ks(PbSO4) = 10-7,8 ; Ks(PbI2) = 10-7,6. Eo Fe3+/Fe2+ = 0,77 V ; Eo S/H2S = 0,14V ; Eo I2/2I- = 0,54V ; Ecal bão hoà = 0,244V Hướng dẫn giải Na2S → 2 Na+ + S2-

a) Tính pH của dung dịch

0,01

0,01

KI → K+ + I0,06 Na2SO4

0,06 → 2Na+ + SO42-

0,05 S2-

HS- + OH-

+ H2O

SO42- + H2O ⇌ H SO4- + OH-

0,05 Kb(1) = 10-1,1

(1)

Kb(2) = 10-12

(2)

Kb(1) >> Kb(2) nên cân bằng (1) quyết định pH của dung dịch: S2-

+

H2O

HS- + OH-

K = 10-1,1

x2 = 10 −1,1 → x 2 + 0,0794 x − 10 −3,1 = 0 0,01 − x

[ ] (0,01 -x)

x

→x

= 8,94. 10-3 →

b)

Pb2+

+

0,09

S2-

x

[OH-] = 8,94.10-3 →

pH = 11,95

(Ks-1) = 1026.

PbS ↓

0,01

0,08 Pb2+

+ SO42- →

0,08

PbSO4 ↓

(Ks-1) = 107,8.

PbI2

(Ks-1) = 107,6.

0,05

0,03 Pb2+ 0,03

+

2 I- → 0,06

Thành phần hỗn hợp: ↓A :

PbS , PbSO4 , PbI2

Dung dịch B :

K+ 0,06M

Na+ 0,12M

Ngoài ra còn có các ion Pb2+ ; SO42- ; S2- do kết tủa tan ra. PbI 2 : 3 10 −7, 6 / 4 = 10 −2,7

Độ tan của 3

PbSO 4 : S = 10

-7,8

= 10 −3,9

PbS : S = 10 -26 = 10 −13


Bởi vì độ tan của PbI2 là lớn nhất nên cân bằng chủ yếu trong dung dịch là cân bằng tan của PbI2. PbI2↓ = Pb2+ + 2I-

Ks

Do đó [Pb2+] = 10-47 = 2 x 10-3M và [I-] = 4.10-3M. 10−7,8 [SO42-] =

=

5. 10−5,8 = 7,9.10−6M << [Pb2+]

=

5. 10−24 << [Pb2+]

2 × 10−3 10−26 [S2-] = 2 × 10−3 Các nồng độ SO42-, S2- đều rất bé so với nồng độ Pb2+, như vậy nồng độ Pb2+ do PbS và PbSO4 tan ra là không đáng kể nên cách giải gần đúng trên là hoàn toàn chính xác. − Nhận biết các chất có trong kết tủa A: PbS; PbSO4; PbI2. Cho kết tủa hoà tan trong NaOH dư : ↓ PbS không tan, có màu đen. Dung dịch có PbO22-, SO42-, I-, OHPbSO4 + 4 OH+ 4 OH-

PbI2

PbO22- + SO42- + 2 H2O

PbO22- + 2 I-

+

2 H2O

Nhận ra ion SO42-: cho BaCl2 dư: có kết tủa trắng BaSO4, trong dung dịch có PbO22, OH-, Ba2+, I-. Nhận ra I-, Pb2+: axit hoá dung dịch bằng HNO3 dư sẽ có kết tủa vàng PbI2 xuất hiện: OH- + H+

PbO22- + 4 H+ → Pb2+ + 2H2O

→ H2O

Pb2+

+ 2 I- →

PbI2↓ c) Axit hoá dung dịch X: S2- + 2H+ → H2S (C H2S = 0,010 < S H2S nên H2S chưa bão hoà, không thoát ra khỏi dung dich) Phản ứng:

2 Fe3+ + H2S → 2 Fe2+ + 0,1

0,01

0,08

2 Fe3+ + 2I- → 4

S + 2 H+

0,02 2 Fe2+

K=1021

0,02 +

I2

K=107,8


0,08

0,06

0,02

0,02

0,08

0,030

Thành phần trong dung dịch: Fe3+ 0,020 ; Fe2+ 0,080 ;I2 0,030M ;H+ 0,02M E Fe3+/Fe2+ = 0,77 + 0,059 lg 0,02/0,08 = 0,743V (cực dương) Ecal = 0,244V ( cực âm) Epin = E+ − E− = 0,743 − 0,244 = 0,499V Sơ đồ pin: Hg Hg2Cl2 KCl bh Phản ứng: −

2 Hg

+ 2x Fe3+

Fe3+, Fe2+

Pt

+

2 Cl-

= Hg2Cl2 + 2 e

+

e

= Fe2+

2 Hg + 2 Fe3+ + 2 Cl- =

Hg2Cl2↓

Câu 3: (2 điểm) Điện hóa học Ắc quy Chì-Axit được sử dụng khá phổ biến trong xe hơi, phản ứng khi phóng điện là Sự phụ thuộc của thế khử chuẩn E0 vào nhiệt độ được biểu diễn bằng đồ thị dưới đây

a. Viết sơ đồ pin khi ắc quy phóng điện. b. Tính ∆G0, ∆H0, ∆S0 của phản ứng trên ở 298K. c. Tính thế điện cực E của phản ứng ở 50C tại pH=5 và nồng độ ion sunfat là 0,10M.


Hướng dẫn giải a. Sơ đồ pin: Catot (+) PbO2(r) │H2SO4(dd)│Pb(r) (-) Anot b.

,

tại 298K có Eo = 2,05 (V).

; c. E = Eo – (RT/nF)lnQ = 1,64 (V) + (0,00138V/K).278(K) Thay số: E = 1,42 (V) Câu 4: (2 điểm) N-P, C-Si và hợp chất Hydro mới sinh là một tác nhân khử có hiệu quả nhất. Xử lý một lượng natri nitrit bằng hỗn hống natri kim loại cho ra một muối X có 43,38% natri và 26,43% nitơ về khối lượng. Một sản phẩm khác của phản ứng này là natri hydroxit. Để tránh sự làm bẩn sản phẩm cuối này thì quá trình tổng hợp được tiến hành trong khí quyển trơ như môi trường nitơ hay argon. a) Xác định công thức muối X. b) Vẽ công thức Lewis anion của muối X. c) Nếu phản ứng được tiến hành trong không khí thì những tạp chất nào có thể sinh ra? d) Viết phương trình tổng hợp muối X. e) Tương tác giữa X với cacbon dioxit sinh ra một chất khí. Viết phương trình phản ứng. Hướng dẫn giải a) Na2N2O2 – natri hyponitrit. b) Theo lý thuyết thì anion N2O 22− thể có đồng phân cis, trans. Thực nghiệm đã chứng minh được rằng ion này chủ yếu ở dạng trans. O N O

O

N

N O

N

N O

N

N O

c) Sản phẩm có thể chứa tạp chất NaNO3, NaNO2, Na2CO3, NaHCO3.

O

N O


e) Na2N2O2 + CO2 → Na2CO3 + N2O Câu 5: (2 điểm) phức chất, trắc quang a) Phức chất A là phức chất của Pt(II), có cấu trúc vuông phẳng, chứa Pt, NH3 và Cl với % khối lượng lần lượt là 65,00%; 11,33% và 23,67%. Viết công thức cấu trúc của phức chất thỏa mãn điều kiện trên, biết phân tử khối của A nhỏ hơn 1000. b) Khi cho phức chất cis-[Pt(NH3)2Cl2] (cấu trúc vuông phẳng) tương tác với tetrametylen điamin (tetraen) người ta tách ra được một phức chất 2 nhân B của Pt(II) có công thức [Pt2Cl4(NH3)2(tetraen)] (B). Kết quả thử nghiệm cho thấy chất B có khả năng kháng tế bào ung thư cao. Viết công thức cấu tạo, mô tả sự hình thành liên kết (theo thuyết VB) trong phức chất B. c) Khi nghiên cứu cơ chế phản ứng thế phối tử trong phức chất vuông phẳng người ta nhận thấy rằng tốc độ thay thế phối tử X bởi một phối tử Y chịu ảnh hưởng của phối tử Z (Z nằm ở vị trí trans so với X). Nếu Z càng hoạt động về ảnh hưởng trans thì X càng linh động và càng dễ bị thế. L

Z

L

Z

Pt

+ Y

L

+

Pt

X

L

X

Y

Từ K2[PtCl4], viết sơ đồ điều chế phức chất cis và phức chất trans-[Pt(C2H4)(2ampy)Cl2] (trong đó 2-ampy là 2-aminopiriđin). Biết thứ tự ảnh hưởng trans của các phối tử: C2H4 > Cl- > 2-ampy. Trong phức chất trên, nguyên tử kim loại trung tâm Pt liên kết với phối tử 2-aminopiriđin qua nguyên tử nitơ nào? Giải thích. Cho biết: Cấu hình electron của Pt: [Xe]4f145d96s1 Hướng dẫn giải a) Tỉ lệ Pt : NH3 : Cl =

,

,

,

,

≈ 1 ∶ 2 ∶ 2

Các phức chất A có công thức phân tử là [Pt(NH3)2Cl2]n. (M = 300n) Vì khối lượng phân tử của phức chất này nhỏ hơn 1000 u nên n = 1 hoặc n = 2 hoặc n = 3. - Nếu n = 1: công thức phân tử là [Pt(NH3)2Cl2], có 2 đồng phân cis và trans. NH3

Cl Pt Cl

NH3

Cl Pt

NH3

H3N

Cl

7 cis

trans


- Náşżu n = 2: cĂ´ng thᝊc phân táť­ lĂ Pt2(NH3)4Cl4, cĂł 2 Ä‘áť“ng phân: [Pt(NH3)4][PtCl4]; [Pt(NH3)3Cl][PtCl3(NH3)]. 2+

+

2-

NH3

H3N

Cl

Cl

Pt

;

Pt

H3N

NH3

Cl

-

Cl

H3N

NH3

Cl

Pt

Pt

H3N

Cl

NH3

Cl

Cl

- Náşżu n = 3: cĂ´ng thᝊc phân táť­ lĂ Pt3Cl6(NH3)6: cĂł 2 Ä‘áť“ng phân: [Pt(NH3)4][PtCl3(NH3)]2; [Pt(NH3)3Cl]2[PtCl4]. (váş˝ tĆ°ĆĄng táťą nhĆ° trĂŞn). b) cis-[Pt(NH3)2Cl2] + H2N-(CH2)4-NH2

[Pt2Cl4(NH3)2(tetraen)] (B)

CẼu trúc bᝠn cᝧa B: NH3

Cl Pt Cl

Cl

H 3N Pt

NH2-CH2CH2CH2CH2-H2N

Cl

-VĂŹ chẼt Ä‘ầu cĂł cẼu hĂŹnh cis nĂŞn trong phᝊc chẼt B, 2 nguyĂŞn táť­ Cl cĹŠng áť&#x; váť‹ trĂ­ cis, phân táť­ tetrametilen Ä‘iamin lĂ m cầu náť‘i giᝯa 2 nguyĂŞn táť­ trung tâm Pt. -MĂ´ tả sáťą hĂŹnh thĂ nh liĂŞn káşżt (theo thuyáşżt VB) trong phᝊc chẼt B: Pt2+ [Xe]5d8. lai hĂła dsp2

lai hĂła dsp2 5d

6s

Cl-

Cl-

6s

6p

6p

NH3 NH2-CH2-CH2-CH2-CH2-NH2 Cl-

5d

Cl-

NH3

c) SĆĄ Ä‘áť“ phản ᝊng Ä‘iáť u cháşż phᝊc chẼt cis:

K2[PtCl4] K[Pt(2-ampy)Cl3] cis-[Pt(C2H4)(2-ampy)Cl2] 2-

-

Cl

Cl

+ 2-ampy

Pt Cl

Cl

+ C2H4

Pt Cl

C2H4

Cl

Cl

Cl

Pt Cl

Am

Am

VĂŹ Cl- cĂł ảnh hĆ°áť&#x;ng trans mấnh hĆĄn 2-ampy nĂŞn nguyĂŞn táť­ Cl áť&#x; váť‹ trĂ­ trans váť›i Am khĂł báť‹ tháşż, phản ᝊng tháşż tiáşżp theo C2H4 tháşż vĂ o nguyĂŞn táť­ Cl áť&#x; váť‹ trĂ­ cis so váť›i Am tấo ra Ä‘áť“ng phân cis. - SĆĄ Ä‘áť“ phản ᝊng Ä‘iáť u cháşż phᝊc chẼt trans:

K2[PtCl4] K[Pt(2-ampy)Cl3] trans-[Pt(C2H4)(2-ampy)Cl2] 2Cl

8

-

Cl Pt

+ C2H4

Pt

Cl

Am

Cl

Cl

+ 2-ampy

Pt


Vì C2H4 có ảnh hưởng trans mạnh hơn Cl- nên nguyên tử Cl ở vị trí trans so với C2H4 dễ bị thế, phản ứng thế tiếp theo 2-ampy thế vào nguyên tử Cl ở vị trí trans so với C2H4 tạo ra đồng phân trans. Trong phức chất trên, nguyên tử kim loại trung tâm Pt liên kết với phối tử 2aminopiriđin qua nguyên tử N của vòng piriđin mà không liên kết qua nguyên tử N của nhóm NH2 vì cặp electron chưa liên kết trên nguyên tử N của nhóm NH2 đã bị vòng piriđin hút nên mật độ electron giảm, khó có khả năng tạo phức. Còn cặp electron chưa liên kết trên nguyên tử N của vòng piriđin (nằm trên obitan lai hóa sp2), mật độ electron lớn hơn nên dễ tạo liên kết phối trí với nguyên tử kim loại trung tâm. Câu 6: (2 điểm) Quan hệ giữa hiệu ứng cấu trúc và tính chất a. Sắp xếp các chất sau đây theo chiều tăng dần lực axit, giải thích:

b. Hãy so sánh tính bazơ của các nguyên tử N trong mỗi phân tử các hợp chất sau: O N H

N

O N O NH

N N N CH3

Hướng dẫn giải a HD: Giá trị pKa của các chất:

N N COOH

N(CH3)2


O

O Cl

O

<

O OEt

16.5

O

<

O <

9

10.7

O

O

O H 5.9

NO2

<

O

<

CF3 4.7

5.1

Tất cả các axit cacbon này cho anion mà đt (-) chủ yếu trên oxi. 3 chất đầu đều chứa nhóm xeton bên trái bên phải là các nhóm xeton, and, este tương ứng. Điện tích (-) đều được giải tỏa trên cả 2 nhóm cacbonyl. And có lực axit mạnh nhất trong nhóm này vì là dạng cacbonyl đơn giản, xeton có nhóm CH3 tạo hiệu ứng siêu liên hợp đẩy vào làm giảm hiệu quả giải tỏa đt (-), este giải tỏa kém nhất do sự cạnh tranh của nguyên tử O trong nhóm este.

3 chất cuối chỉ có 1 nhóm cacbonyl. Với chất chứa Cl chỉ có hiệu ứng cảm ứng âm của Cl không có hiệu ứng liên hợp ở nhóm này nên lực axit yếu nhất. 2 chất cuối có 2 nhóm hút e mạnh, sự giải tỏa đt (-) mạnh nhất nên lực axit mạnh nhất. Trong đó chất chứa CF3 có 3F hút e mạnh lực axit mạnh nhất.

b. So sánh tính bazơ của các nguyên tử N trong mỗi hợp chất dưới đây: O 2 N H

1 N

O 1 N O

NH 3 Axit Lixegic dietylamit 1> 3>2

N3 2 N CH 3

N 4

Cafein

1 2 N N

3 N(CH 3)2

COOH 1>2>3

4> 3>2> 1

Câu 7: (2 điểm)Hidrocacbon Chất A có công thức phân tử C7H12. Khi thực hiện phản ứng ozon phân A tạo HCHO và xiclohexanon.Thực hiện quá trình chuyển hoá A liên tiếp sau: 1, HBr ; 2, Mg/ ete; 3, CO2 sau đó với H3O+ ta thu được hợp chất B (C8H14O2) .Nếu cho A tác


dụng với HBr/ peoxit tiếp theo cho sản phẩm tác dụng với KCN sau đó xử lý bằng dung dịch axit loãng ta thu được chất C (C8H14O2). C cũng có thể tạo ra bằng cách cho sản phẩm của A với HBr/ peoxit tác dụng với Mg/ete, tiếp theo CO2 và xử lý bằng dung dịch axit. a. Xác định cấu trúc các chất trung gian và viết tên A, B, C b. So sánh tính axit của các B,C với các chất D (axit Benzoic) và E (axit Phenyletanoic) ; F(axit 3-phenylpropanoic) Hướng dẫn giải a. CH3

CH2

Br

HBr

CH3

MgBr H3O+

CH2Br 1. KCN 2. H3O+

COOH

1. CO2

Mg/ ete

HBr/peoxit

CH3

CH2COOH

(hoÆc 1. Mg/ ete ) 2. CO2 3. H3O+

A. Metylen xiclohexen B. Axit 1- Metyl xiclocacboxylic C. Axit 2- xiclophenyl etanoic b. Tính axit B< C < F< E < D H3C COOH

+I2

CH2COOH

+I1

< <

+I1

< +I2

-I1CH2CH2COOH

CH2COOH

- I2

<

< -I1

COOH

-I3

<

-I2

<

-I3

Câu 8: (2 điểm) xác định cấu trúc, đồng phân lập thể, danh pháp Tinh dầu tràm được dùng trong sản xuất dược phẩm và mỹ phẩm. Hai hợp phần đóng vai trò chính cho ứng dụng đó là eucaliptol (kí hiệu là A) và một đồng phân cấu tạo của nó kí hiệu là B. Hợp chất A không làm mất màu dung dịch Br2/CCl4. Khi đun nóng A với dung dịch axit sunfuric loãng người ta thu được hợp chất không quang hoạt C kết tinh trong nước thành D công thức C10H20O2.H2O. Đun nóng D với axit photphoric người ta thu được sản phẩm chính là E không quang hoạt và có bộ khung cacbon giống như D (biết rằng có thể dùng phương pháp thích hợp để tách lấy từng đối quang từ E). Sau khi ozon phân chế hóa khử thì từ E và từ B đều thu được 3-(1-hydroxy-1-metyletyl)-6-oxoheptanal. 11


a. Hãy xác định công thức cấu tạo của A, B, C và E. b. Giải thích vì sao B quang hoạt còn E thì không. So sánh có giải thích nhiệt độ nóng chảy, nhiệt độ sôi của A, B, C và E. c. Hãy vẽ công thức phối cảnh của A, C và giải thích vì sao A dễ dàng bị chuyển hóa bởi axit sunfuric loãng thành C, chất này lại dễ kết tinh trong nước thành D. d. Hãy đề nghị một sơ đồ tổng hợp E từ những hợp chất chứa không quá 5C. Có thể dùng sơ đồ đó để tổng hợp B được không, vì sao? e. Hãy đề nghị 2 phương pháp tổng hợp A từ những hợp chất chứa không quá 6C. a. Hãy xác định công thức cấu tạo của A, B, C và E. E có 10 C như D. Sản phẩm ozon phân E và B là 3-(1-hydroxy-1-metyletyl)6-oxoheptanal cũng có 10 C suy ra chúng có cấu tạo là:

Công thức phân tử của A (đồng phân của B) là C10H18O, của C là C10H20O2 chứng tỏ A đã cộng với 1 phân tử H2O để thành C. Độ không no của A bằng 2, nó không làm mất màu dung dịch Br2/CCl4 suy ra không có liên kết bội cacbon-cacbon. Các chuyển hóa từ A thành C, D rồi E cho thấy A có bộ khung cacbon như B. Vậy A là một ete 2 vòng no, trong đó 1 vòng chứa liên kết ete (C-O-C). Nhờ xúc tác H+ vòng ete cộng nước mở ra tạo thành diol không quang hoạt C. Hydrat D mất 2 phân tử H2O tạo ra E, tức là C mất 1 phân tử H2O tạo ra E. Sự phân tích trên cho phép suy ra cấu tạo của các chất A, C, E và sự chuyển hóa giữa chúng là như sau:

b. Giải thích vì sao B quang hoạt còn E thì không. So sánh có giải thích nhiệt độ nóng chảy, nhiệt độ sôi của A, B, C và E. B và E có cùng công thức cấu tạo với 1 nguyên tử cacbon bất đối. B được tạo ra trong lá cây tràm nên cacbon bất đối chỉ có thể ở một cấu hình (hoặc R, hoặc S), 12


E được hình thành trong bình phản ứng qua ion cacboni C2, khả năng mất H+ từ 2 nhóm CH2 là như nhau, tạo ra cacbon cấu hình R và cacbon cấu hình S với sác xuất như nhau nên E là biến thể raxemic. Nhiệt độ sôi và nóng chảy của từng đối quang thường thấp hơn biến thể raxemic (vì ở biến thể raxemic hai phân tử đối quang kiên kết chặt chẽ với nhau tạo ta hợp chất mới chứ không phải là hỗn hợp thông thường). Hợp chất A là ete không tạo được liên kết hydro liên phân tử, diol C tạo được nhiều liên kết hydro liên phân tử hơn so với monoalcol B và E. Do đó nhiệt độ sôi và nhiệt độ nóng chảy biến đổi theo trật tự sau: A<B<E<C c. Hãy vẽ công thức phối cảnh của A, C và giải thích vì sao A dễ dàng bị chuyển hóa bởi axit sunfuric loãng thành C, chất này lại dễ kết tinh trong nước thành D.

Ba vòng 6 cạnh ở A và A1 đều ở dạng thuyền kém bền nên ion oxoni A1 dễ mở vòng thành ion cacboni bậc ba, sau đó cộng với nước thành ancol C có cấu dạng ghế bền hơn. Các monoancol tecpen như B, mentol, … không kết tinh ngậm nước. Hai nhóm OH ở diancol C dễ ở cùng một phía của vòng 6 cạnh dễ hình thành được 2 liên kết hydro với 1 phân tử nước (H2O vừa cho H vừa nhận H) tạo ra cấu trúc bền thắng được tính kị nước của gốc hydrocacbon C10H18. d. Hãy đề nghị một sơ đồ tổng hợp E từ những hợp chất chứa không quá 5C. Có thể dùng sơ đồ đó để tổng hợp B được không, vì sao?

Trong sơ đồ trên, ở giai đọan phản ứng Diels-Alder, nhóm metyl của dien và nhóm axetyl của dienophin phân bố ở xa nhau để tránh hiệu ứng không gian nên sản 13


phẩm chính là xeton có cấu tạo như trên sơ đồ. Tuy nhiên sác xuất hình thành cacbon bất đối R và S là như nhau đối với xeton đó nên sau phản ứng vơi MeMgBr thì tạo ra biến thể raxemic E mà không tạo ra B quang hoạt được. e. Hãy đề nghị 2 phương pháp tổng hợp A từ những hợp chất chứa không quá 6C. Phương pháp 1 [K. Alder, 1949]: OH

H 2O/H +

1) MeMgBr/ete

to

P2O5

O

+

2) H 3O

OH

O

O

OH

Phương pháp 2 [W. Perkin, 1907]:

Câu 9: (2 điểm) Cơ chế phản ứng Giải thích các phản ứng sau bằng cơ chế phản ứng? a. H/H2O

2 OH

b.

c. O

OH OH-

OH (A)

d.

O (B)

(A)


CO2H O CO2Me

O DBU, MeOH

MeO2C

DBU: 1,8-Diazabicyclo[5.4.0]undec-7-ene

N N

Hướng dẫn giải a.

H 2O

OH

b.

c.

d. Giai đoạn đầu tiên của quá trình chuyển hóa là phản ứng cộng Michael:


Câu 10: (2 điểm) Tổng hợp các chất hữu cơ (Dạng sơ đồ phản ứng) (+)-Artemisinin (Quinhaosu) là một hoạt chất tự nhiên có tác dụng chữa bệnh sốt rét mà không gây tác dụng phụ được tách ra từ cây Thanh hao hoa vàng (Artemisia annua L., họ Asteriaceae). Đây là một lacton dạng secquitecpen. Hoạt tính kháng sốt rét của hợp chất này là do nó có cấu trúc cầu endoperoxit. Do nguồn thu tự nhiên hạn chế, trong khi yêu cầu chữa bệnh sốt rét vẫn luôn cấp thiết nên các nhà hóa dược đặt ra vấn đề phải tổng hợp hoặc bán tổng hợp hoạt chất này. Năm 2003, một nhóm nghiên cứu tại viện Hóa kỹ thuật Ấn Độ đã đề xuất một qui trình tổng hợp (+)-Artemisinin từ (+)-Isolimonen theo sơ đồ dưới đây:

Trong sơ đồ trên, PCC là piriđinium clorocromat; KHMDS là kali hexametilen 16


đisilan (hay kali bis(trimetylsilyl) amiđua. Đây là một bazơ rất mạnh, tương tự NaNH2, nhưng cồng kềnh nên không thể hiện tính nucleophin). Hãy cho biết cấu trúc các hợp chất từ A đến I trong sơ đồ tổng hợp (+)Artemisinin trên. Hướng dẫn giải Hoàn thành sơ đồ phản ứng tổng hợp (+)-Artemisinin:

Cho: H = 1; C = 12; N = 14; O = 16; Na = 23; Mg = 24,3; Al = 27; P = 31; S = 32; Cl = 35,5; K = 39,1; Ca = 40,1; Ti = 47,9; Cr = 52; Mn = 54,9; Fe = 55,8; Co = 58,9; Ni = 58,7; Cu = 63,5; Zn = 65,4; Ag = 107,9; Ba = 137,3. --------------------------------------------------Thí sinh không được sử dụng tài liệu. Cán bộ coi thi không giải thích gì thêm. Giáo viên: Quách Phạm Thùy Trang 0975855880

17


ĐỀ ĐÓNG GÓP CHO KỲ THI C10 Môn Hoá học lớp 11 Năm học : 2016-2017 Thời gian làm bài: 180 phút (Đề gồm 03 trang) Bài 1: (2 điểm) Tốc độ phản ứng- Cân bằng hóa học k k Có hai phản ứng bậc nhất nối tiếp nhau A  → B  → C nồng độ của B có giá SỞ GIÁO DỤC – ĐÀO TẠO THÁI BÌNH Trường THPT Chuyên Thái Bình

1

2

trị cực đại ở thời điểm τ tính theo phương trình τ = ln [( k2/ k1) / (k2 − k1)] a) Viết phương trình động học vi phân cho các chất A, B, C. _ . b) Tỷ số k2 / k1 phải như thế nào để τ bằng nửa - chu kỳ chuyển hoá chất A? Bài 2: (2 điểm) Cân bằng trong dung dịch điện li 1 Cho 0,01 mol NH3, 0,1 mol CH3NH2 và 0,11 mol HCl vào nước được 1 lít dung dịch. Tính pH của dung dịch thu được? Cho: pK NH = 9,24; pK CH NH = 10,6; pK H O = 14. + 4

3

+ 3

2

2 Tính độ tan của AgSCN trong dung dịch NH3 0,003 M. Cho T AgSCN = 1,1.10-12 và hằng số phân li của phức [Ag(NH3)2]+ bằng 6.10-8. Bài 3: (2 điểm) Điện hoá học Lắp 1 pin bằng cách nối điện cực hydro chuẩn với một nửa pin bởi 1 dây đồng nhúng vào 40ml ddCuSO4 0,01M có thêm 10ml ddNH3 0,5M. Chấp nhận rằng chỉ tạo phức Cu(NH3 ) 4 2+ với nồng độ NH 4 + là không đáng kể so với nồng độ NH3. a. Xác định E Cu 2+ /Cu . b. Tính E oCu(NH3 )4 2+ /Cu . Biết E oCu 2+ /Cu = 0,34v;

Cu(NH3 ) 4 2+ /Cu

lgβ Cu(NH3 ) 4 2+ =13,2 và ECu 2+ /Cu=ECu(NH 3 ) 4 2+ /Cu

Bài 4: (2 điểm) Nhóm N-P, nhóm C-Si A là một hợp chất của nitơ và hidro với tổng điện tích hạt nhân bằng 10. B là một oxit của nitơ, chứa 36,36% oxi về khối lượng. a. Xác định các chất A, B, X, D, E, G và hoàn thành các phương trình phản ứng: A + NaClO → X + NaCl + H2O X + HNO2 → D + H2O D + NaOH → E + H2O A + Na → G + H G + B → E + H2O b. Viết công thức cấu tạo của D. Nhận xét về tính oxi hóa - khử của nó. c. D có thể hòa tan Cu tương tự HNO3. Hỗn hợp D và HCl hòa tan được vàng tương tự cường thủy. Viết phương trình của các phản ứng tương ứng. Bài 5: (2 điểm) Phức chất Hòa tan 2,00 gam muối CrCl3.6H20 vào nước, sau đó thêm lượng dư dung dịch AgNO3 và lọc nhanh kết tủa AgCl cân được 2,1525 gam. Cho biết muối crom nói trên tồn tại dưới dạng phức chất.

1


1. Hãy xác định công thức của phức chất đó. 2. Hãy xác định cấu trúc (trạng thái lai hóa, dạng hình học) và nêu từ tính

của phức chất trên. Bài 6: (2 điểm) Quan hệ cấu trúc - tính chất a) Sắp xếp các chất trong dãy sau đây theo trình tự tăng dần tính bazơ : 2,4,6- trinitro- N,N – đimetylanilin, p- nitroanilin, N,N – đimetylanilin và anilin,? Giải thích? b) Hãy giải thích vì sao chất béo thực vật thường có nhiệt độ đông đặc thấp hơn chất béo động vật? Bài 7: (2 điểm) Hidrocacbon. Ankin A có công thức phân tử C6H10, có đồng phân quang học. Hidro hóa hoàn toàn A thu được A1. a. Viết công thức cấu tạo của A và A1. Cho biết A1 có đồng phân quang học hay không? b. Ankin B cũng có công thức phân tử C6H10. B tác dụng với H2 (xúc tác Ni, t0) thu được 2-metylpentan. B không tác dụng với dung dịch AgNO3/NH3. B tác dụng với H2O (xúc tác HgSO4, t0) tạo chất C6H12O (B1). Xác định công thức cấu tạo của B và B1. c. Hidro hóa B (xúc tác Pd/PbCO3, t0) thu được chất C. Chất C tác dụng với H2SO4 rồi thủy phân tạo chất D. Viết công thức cấu tạo của C và D. Biết C và D là sản phẩm chính. Cho biết C là đồng phân cis hay trans? d. Tách nước chất D với xúc tác H2SO4 đặc và đun nóng. Viết phương trình hóa học và nêu sản phẩm chính. Cho biết tên cơ chế phản ứng. Bài 8: (2 điểm) Xác định cấu trúc Thủy phân hợp chất A (C13H18O2) trong môi trường axit HCl loãng cho hợp chất B (C11H14O). Khi B phản ứng với brom trong NaOH, sau đó axit hóa thì thu được axit C. Nếu đun nóng B với hỗn hợp hiđrazin và KOH trong glicol thì cho hiđrocacbon D. Mặt khác, B tác dụng với benzanđehit trong dung dịch NaOH loãng (có đun nóng) thì tạo thành E (C18H18O). Khi A, B, C, D bị oxi hóa mạnh thì đều cho axit phtalic. Hãy viết công thức cấu tạo của các hợp chất từ A đến E. Bài 9: (2 điểm) Cơ chế Ancol tert-butylic cũng như iso-butilen khi đun nóng với metanol có H2SO4 xúc tác đều cho sản phẩm chính là A (C5H12O). Ngoài ra, tuỳ thuộc chất đầu là ancol tertbutylic hay iso-butilen mà còn tạo ra các sản phẩm phụ khác như B (C8H18O), C (C9H20O), D (C8H16)... Khi cho A, B, C, D tác dụng với CH3MgI đều không thấy khí thoát ra. a) Hãy xác định công thức cấu tạo của A và giải thích vì sao A lại là sản phẩm chính trong cả hai trường hợp đã cho. b) Dùng công thức cấu tạo để viết sơ đồ phản ứng giải thích sự hình thành sản phẩm phụ trong trường hợp dùng ancol tert-butylic và trường hợp dùng iso-butilen. Bài 10: (2 điểm) Tổng hợp hữu cơ Thành phần chính của dầu thông là α-pinen (2,6,6-trimetylbixiclo[3.1.1]hept-2-en).

2


Cho α-pinen tác dụng với axit HCl được hợp chất A, sau đó cho A tác dụng với KOH/ancol thu được hợp chất camphen (B). Viết cơ chế phản ứng chuyển hóa α-pinen thành A và A thành B. Trong môi trường axit, B quang hoạt chuyển hóa thành B raxemic. Giải thích hiện tượng này.

Cl

OAc A

B

C

Viết sơ đồ các phản ứng tổng hợp B từ xiclopentađien và acrolein cùng các hóa chất cần thiết khác, biết rằng một trong số các sản phẩm trung gian của quá trình tổng hợp là một enol axetat C.

------------------------------HẾT-------------------------------

Gv: Phạm Văn Tuân Điện thoại di động: 0973 429 641

3


SỞ GIÁO DỤC – ĐÀO TẠO THÁI BÌNH Trường THPT Chuyên Thái Bình

ĐÁP ÁN ĐỀ ĐÓNG GÓP CHO KỲ THI C10

Môn Hoá học lớp 11 Năm học : 2016-2017 Thời gian làm bài : 180 phút (Đề gồm 02 trang)

*****

Bài 1: (2 điểm) Tốc độ phản ứng- Cân bằng hóa học k k Có hai phản ứng bậc nhất nối tiếp nhau A  → B  → C nồng độ của B có giá 1

2

trị cực đại ở thời điểm τ tính theo phương trình τ = ln [( k2/ k1) / (k2 − k1)] a) Viết phương trình động học vi phân cho các chất A, B, C. _ . b) Tỷ số k2 / k1 phải như thế nào để τ bằng nửa - chu kỳ chuyển hoá chất A? Câu Câu 1

Hướng dẫn

Điểm

a) Ph−¬ng tr×nh vi ph©n ®éng häc:

dC A = − k1.CA ; dt

dCB dCC = k1.CB – k2.CB ; = k2.CC . dt dt k ( 2) ln2 ln 2 k k1 b) τ = t 1/2 = ®Æt 2 = a ta cã  → ln = k1 k 2 − k1 k1 k1 ln

1,0

1,0

a =ln 2 → a = 2 a −1

Bài 2: (2 điểm) Cân bằng trong dung dịch điện li 1 Cho 0,01 mol NH3, 0,1 mol CH3NH2 và 0,11 mol HCl vào nước được 1 lít dung dịch. Tính pH của dung dịch thu được? Cho: pK NH = 9,24; pK CH NH = 10,6; pK H O = 14. + 4

+ 3

3

2

2 Tính độ tan của AgSCN trong dung dịch NH3 0,003 M. Cho T AgSCN = 1,1.10-12 và hằng số phân li của phức [Ag(NH3)2]+ bằng 6.10-8.

Câu 2a.

Hướng dẫn +

1.

NH3 + H  → NH 0,01 0,01 0,01 mol + CH3NH2 + H  → CH3NH 3+ 0,1 0,1 0,1 mol Tổng số mol H+ phản ứng: n = 0,01 + 0,1 = 0,11 mol. Vậy H+ vừa hết. Dung dịch thu được gồm: 0,01 mol NH +4 và 0,1 mol CH3NH 3+ Vì K H O << K CH NH+ < K NH+ nên có thể bỏ qua cân bằng điện ly của nước 2

+ 4

3

3

Điểm 1,0đ

4

trong dung dịch. ⇀ CH3 NH2 + H+ CH3NH 3+ ↽ Ban đầu 0,1 Điện ly x x x Cân bằng 0,1 - x x x+y

4


⇀ NH3 + H+ NH +4 ↽ Ban đầu 0,01 Điện ly y y y Cân bằng 0,01 – y y x+y Áp dụng định luật tác dụng khối lượng cho các cân bằng điện li ta có : [CH3 NH 2 ].[H + ] x(x + y) K CH NH+ = = = 10-10,6 + 3 3 0,1- x [CH3 NH3 ] ⇒ x (x + y) = 10-11,6 (1) (giả sử coi x << 0,1) + [NH3 ][H ] y(x + y) K NH+ = = = 10-9,24 + 4 0, 01y [NH 4 ]

⇒ y (x + y) = 10-9,24 . 10-2 = 10-11.24 (2) (giả sử coi y << 0,01) Từ (1) và (2) ta có x (x + y ) + y (x + y) = 10-11,6 + 10-11,24 = 8,27.10-12

⇔ x + y = 8,27 .10 −12 ⇒ [ H+ ] = x + y = 2,88 . 10-6 mol.lit-1. ⇒ pH = -lg [ H+ ] = - lg 2,88 . 10-6 = 5,54

2b.

Các cân bằng xảy ra trong dung dịch: ⇀ Ag+ + SCNAgSCN ↽ ⇀ [Ag(NH3)2]+ Ag+ + 2NH3 ↽

1,0đ TAgSCN = 1,1.10-2 1 1 Kb = = K pl 6.10−8

(2) ⇀ [Ag(NH3)2]+ + SCNAgSCN + 2NH3 ↽

0,003 – 2s

s

Kcb = TAgSCN.

1 K pl

s

Áp dụng định luật tác dụng khối lượng cho cân bằng (3) ta có: T s2 = AgSCN 2 (0, 003 − 2s) K pl Thay số vào ta có s = 1,27.10-5 mol.lit-1.đ

Bài 3: (2 điểm) Điện hoá học Lắp 1 pin bằng cách nối điện cực hydro chuẩn với một nửa pin bởi 1 dây đồng nhúng vào 40ml ddCuSO4 0,01M có thêm 10ml ddNH3 0,5M. Chấp nhận rằng chỉ tạo phức Cu(NH3 ) 4 2+ với nồng độ NH 4 + là không đáng kể so với nồng độ NH3. a. Xác định E Cu 2+ /Cu . b. Tính E oCu(NH3 )4 2+ /Cu . Biết E oCu 2+ /Cu = 0,34v; Cu(NH3 )4 2+ /Cu lgβ Cu(NH3 ) 4 2+ =13,2 và ECu 2+ /Cu=ECu(NH 3 ) 4 2+ /Cu Câu Hướng dẫn 3a. Cu 2+  = 0,8.10−2 

Điểm 1đ 5


[ NH3 ] = 0,1 Cu 2+ +2e → Cu

E o =0,34(v) 0,059  2+  o E Cu 2+ /Cu =E Cu + lg Cu  (1) 2+ /Cu 2 Cu 2+ +4NH3 ⇌ Cu(NH 3 ) 4 2+ bñ

0,8.10−2

0,1

0

pöù

0,8.10−2

3, 2.10−2

0,8.10−2

cb

0

6,8.10−2

0,8.10−2

⇒ Cu (1)

3b.

β=1013,2

2+ Cu(NH3 ) 4 ] [ 0,8.10−2 = = =2,4.10−11 4 4 13,2  [ NH3 ] .β (0,068) .10

2+ 

⇒ E Cu 2+ /Cu =0,34+

b/ E Cu 2+ /Cu =E Cu(NH

3 )4

0,059 lg2,4.10−11 =0,026(v) 2 2+

(2)

/Cu

Cu(NH 3 ) 4 2+ +2e → Cu+4NH3 0,059 o E Cu(NH ) 2+ /Cu =E Cu(NH + lg 2+ 3 4 3 ) 4 /Cu 2

(1)(2)(3)

⇒ E oCu(NH

3 )4

2+

/Cu

[Cu(NH3 )4 ]2+ [ NH3 ]4

(3)

= -0,05(V)

Bài 4: (2 điểm) Nhóm N-P, nhóm C-Si A là một hợp chất của nitơ và hidro với tổng điện tích hạt nhân bằng 10. B là một oxit của nitơ, chứa 36,36% oxi về khối lượng. a. Xác định các chất A, B, X, D, E, G và hoàn thành các phương trình phản ứng: A + NaClO → X + NaCl + H2O X + HNO2 → D + H2O D + NaOH → E + H2O G + B → D + H2O b. Viết công thức cấu tạo của D. Nhận xét về tính oxi hóa - khử của nó. c. D có thể hòa tan Cu tương tự HNO3. Hỗn hợp D và HCl hòa tan được vàng tương tự cường thủy. Viết phương trình của các phản ứng tương ứng. Câu Hướng dẫn Điểm 4a. * Do N có Z=7 và H có Z=1; mà chất A có tổng ĐTHN là 10. A là NH3. * Đặt oxit nito là NxOy => N2O * Các phản ứng: NH3 + NaClO → N2H4 + NaCl + H2O 0,25 đ N2H4 + HNO2 → HN3 + 2H2O 0,25 đ HN3 + NaOH → NaN3 + H2O 0,25 đ 2NH3 + 2Na → 2NaNH2 + H2 0,25 đ NaNH2 + N2O → NaN3 + H2O 0,25 đ 6


4b.

4c.

A là NH3; B là N2O; X là N2H4; D là HN3; E là NaN3; G là NaNH2. D: Axit hidrazoic 0,25đ (-3) (+5) (-3) H-N =N ≡N Trong phân tử HN3 vừa có N(+5), vừa có N(-3) nên nó vừa có tính oxi hóa, vừa có tính khử. Về tính oxi hóa nó giống HNO3 nên hòa tan Cu: Cu + 3HN3 → Cu(N3)2 + N2 + NH3 0,25đ Khi trộn với HCl: 2Au + 3HN3 +8HCl → 2H[AuCl4] + 3N2 + 3NH3 0,25đ

Bài 5: (2 điểm) Phức chất Hòa tan 2,00 gam muối CrCl3.6H20 vào nước, sau đó thêm lượng dư dung dịch AgNO3 và lọc nhanh kết tủa AgCl cân được 2,1525 gam. Cho biết muối crom nói trên tồn tại dưới dạng phức chất. 1. Hãy xác định công thức của phức chất đó. 2. Hãy xác định cấu trúc (trạng thái lai hóa, dạng hình học) và nêu từ tính của phức chất trên.

Câu 1.

2.

Hướng dẫn

Điểm n(AgCl) = (2,1525:143,5) = 0,015; n(CrCl3 . 6H2O) = (2:266,5) = 1đ

7,5.10-3 n(Cl- tạo phức) = 3(7,5.10-3) - 0,015 = 7,5.10-3 Trong phân tử phức chất tỷ lệ mol Cl − : Cr3+ = (7,5.10-3) : (7,5.10-3) = 1:1 Công thức của phức: [Cr(H2O)5Cl]2+ Cr3+ (1s2 2s2 2p6 3s2 3p6 3d3) →

24

3d3

4s

24

Cr3+ : [Ar] 3d3

4p

Ar

Cr lai hãa sp3d2

Phøc thuËn tõ

(0,25đ)

Cl 900

H2 O

H2 O

A H 2O

H 2O 900

H2 O B¸t diÖn ®Òu

(0,25đ)

Bài 6: (2 điểm) Quan hệ cấu trúc - tính chất a) Sắp xếp các chất trong dãy sau đây theo trình tự tăng dần tính bazơ : 2,4,6- trinitro- N,N – đimetylanilin, p- nitroanilin, N,N – đimetylanilin và anilin,? Giải thích? b) Hãy giải thích vì sao chất béo thực vật thường có nhiệt độ đông đặc thấp hơn chất béo động vật?

Hướng dẫn

Điể 7


u 6a. / a) Trình tự tăng dần tính bazơ : p- nitroanilin < pKa: 1,02

do nhóm - NO2 có –I -C hút e mạnh =>tính bazơ giảm

anilin < 4,58

m 1,5đ

N,N – đimetylanilin < 2,4,6- trinitro- N,N – đimetylanilin 5,06 9,3 có 2 nhóm –CH3 hiệu ứng +I => tính bazơ tăng

nhóm –NO2 có hiệu ứng –I, nh không gian loại II: 2 nhóm – trong nhân benzen và 2 nhóm N đã gây cản trở không gian làm cho các e p c N không liên hợp được với các e c nhân thơm (trục của e p không song song v trục của e π , vi phạm nguy hợp) nên tính bazơ tăng.

6b. Do thành phần của dầu thực vật chủ yếu este chứa gốc axit không no, còn 0,5 mỡ động vật chủ yếu chứa este chứa gốc axit no. đ Do thành phần của dầu thực vật chủ yếu este chứa gốc axit không no, còn mỡ động vật chủ yếu chứa este chứa gốc axit no.

Bài 7: (2 điểm) Hidrocacbon. Ankin A có công thức phân tử C6H10, có đồng phân quang học. Hidro hóa hoàn toàn A thu được A1. a. Viết công thức cấu tạo của A và A1. Cho biết A1 có đồng phân quang học hay không? b. Ankin B cũng có công thức phân tử C6H10. B tác dụng với H2 (xúc tác Ni, t0) thu được 2-metylpentan. B không tác dụng với dung dịch AgNO3/NH3. B tác dụng với H2O (xúc tác HgSO4, t0) tạo chất C6H12O (B1). Xác định công thức cấu tạo của B và B1. c. Hidro hóa B (xúc tác Pd/PbCO3, t0) thu được chất C. Chất C tác dụng với H2SO4 rồi thủy phân tạo chất D. Viết công thức cấu tạo của C và D. Biết C và D là sản phẩm chính. Cho biết C là đồng phân cis hay trans? d. Tách nước chất D với xúc tác H2SO4 đặc và đun nóng. Viết phương trình hóa học và nêu sản phẩm chính. Cho biết tên cơ chế phản ứng. Câu 7a. 7b. 7c. 7d.

Hướng dẫn A: CH3-CH2-CH(CH3)-C≡CH A1: CH3-CH2-CH(CH3)-CH2-CH3 A1 không có đồng phân quang học. B: (CH3)2CH-C≡C-CH3 B1: (CH3)2CH-CO-CH2-CH3 C: (CH3)2CH-CH=CH-CH3 C có cấu hình cis D: (CH3)2CH-CH2-CH(OH)-CH3 * Viết ptpư tạo: (CH3)2CH-CH=CH-CH3 (SPC) và: (CH3)2CH-CH2-CH=CH2 * Cơ chế tách E1.

Điểm 0,25 đ 0,25 đ 0,25 đ 0,25 đ 0,25 đ 0,25 đ 0,25 đ 0,25 đ

8


Bài 8: (2 điểm) Xác định cấu trúc Thủy phân hợp chất A (C13H18O2) trong môi trường axit HCl loãng cho hợp chất B (C11H14O). Khi B phản ứng với brom trong NaOH, sau đó axit hóa thì thu được axit C. Nếu đun nóng B với hỗn hợp hiđrazin và KOH trong glicol thì cho hiđrocacbon D. Mặt khác, B tác dụng với benzanđehit trong dung dịch NaOH loãng (có đun nóng) thì tạo thành E (C18H18O). Khi A, B, C, D bị oxi hóa mạnh thì đều cho axit phtalic. Hãy viết công thức cấu tạo của các hợp chất từ A đến E. Câu Hướng dẫn Điểm Sự tạo thành axit phtalic cho thấy các hợp chất là dẫn xuất của benzen bị thế hai 1đ 8. lần ở vị trí ortho. B là một xeton có nhóm CH3CO-. HO

-

B + C6H5CHO E (C18H18O) Cho thấy B chỉ ngưng tụ với một phân tử benzanđehit, vậy nhóm CH3CO- sẽ đính trực tiếp vào nhân benzen và xeton B phải là o-C3H7C6H4COCH3. C3H7 CH3 O

O

A

C3H7

C3H7

C3H7

C3H7

COCH3

COOH

C2H5

COCH=CHPh

B

C

D

E

Bài 9: (2 điểm) Cơ chế Ancol tert-butylic còng nh− iso-butilen khi ®un nãng víi metanol cã H2SO4 xóc t¸c ®Òu cho s¶n phÈm chÝnh lµ A (C5H12O). Ngoµi ra, tuú thuéc chÊt ®Çu lµ ancol tert-butylic hay isobutilen mµ cßn t¹o ra c¸c s¶n phÈm phô kh¸c nh− B (C8H18O), C (C9H20O), D (C8H16)... Khi cho A, B, C, D t¸c dông víi CH3MgI ®Òu kh«ng thÊy khÝ tho¸t ra. a) H·y x¸c ®Þnh c«ng thøc cÊu t¹o cña A vµ gi¶i thÝch v× sao A l¹i lµ s¶n phÈm chÝnh trong c¶ hai tr−êng hîp ®· cho. b) Dïng c«ng thøc cÊu t¹o ®Ó viÕt s¬ ®å ph¶n øng gi¶i thÝch sù h×nh thµnh s¶n phÈm phô trong tr−êng hîp dïng ancol tert-butylic vµ tr−êng hîp dïng iso-butilen.

Câ u 9a, b

Hướng dẫn

(CH3)3C - OH

H

+

- H 2O

+

CH3 C

CH3

H

Điể m

+

CH2 C

CH3

CH3

CH3 CH3OH

CH3 CH3 C CH3

O

CH3

(A)

CH3OH

H

+

- H 2O

(B) CH3+

(CH3)3C+ bÒn h¬n +CH3 ; (CH3)3C – OH cång kÒnh h¬n CH3 – OH V× vËy, ph¶n øng chÝnh lµ:

1đ 1đ

9


H+

+

(CH3)3C + CH3OH

CH3 O

CH3 C CH3

CH3 +

(CH3)3C + CH3 C

OH

CH3

CH3

H+

O

CH3 C

C

+

+

(CH3)3C + CH2 C

CH3

CH2 C

(CH3)3C

CH3 (B)

CH3

CH3

CH3

(A)

CH3

H+

CH3

(CH3)3C

CH3

CH3 H

+

(D)

CH3OH

CH3

CH3 CH3 C

CH

CH2

CH3

C

O-CH3 (C)

CH3

Bài 10: (2 điểm) Tổng hợp hữu cơ Thành phần chính của dầu thông là α-pinen (2,6,6-trimetylbixiclo[3.1.1]hept-2-en).

Cho α-pinen tác dụng với axit HCl được hợp chất A, sau đó cho A tác dụng với KOH/ancol thu được hợp chất camphen (B). Viết cơ chế phản ứng chuyển hóa α-pinen thành A và A thành B. Trong môi trường axit, B quang hoạt chuyển hóa thành B raxemic. Giải thích hiện Viết sơ đồ các phản ứng tổng hợp B từ tượng này. xiclopentađien và acrolein cùng các hóa chất Cl cần thiết khác, biết rằng một trong số các sản OAc phẩm trung gian của quá trình tổng hợp là A B C một enol axetat C. Hướng dẫn: Sơ đồ phản ứng tổng hợp camphen (B) từ α-pinen: Cl HCl

chuyÓn vÞ - HCl

A

α - Pinen

B

Cơ chế phản ứng chuyển hóa α-pinen thành A: Cl

H+

chuyÓn vÞ

Cl

-

α− Pinen

A

Cơ chế tạo thành B từ A: 3

7

Cl

1

2

3

-

chuyÓn vÞ

4

1

2

-Cl

3

A

4

-H

+

4

B

Giải thích hiện tượng raxemic hóa hợp chất B:

10


H+

-H+

chuyÓn vÞ

H+

-H+

B quang ho¹t

B ®èi quang

Tổng hợp camphen (B) từ xiclopentađien và acrolein: CHO

CHO CHO

H2

+ O Na, CH3I

OAc Ac2O

O

O3

HO CH3MgI -HOH

Camphen

------------------------------HẾT-------------------------------

Gv: Phạm Văn Tuân Điện thoại di động: 0973 429 641

11


TRƯỜNG THPT CHUYÊN VĨNH PHÚC

KÌ THI CHỌN HỌC SINH GIỎI KHU VỰC ĐỒNG BẰNG DHBB NĂM 2017 MÔN HÓA HỌC – KHỐI 11 Thời gian làm bài: 180 phút (Đề gồm 05 trang )

ĐỀ THI ĐỀ XUẤT

Câu 1 : (2,0 điểm) Động học (Có cơ chế) – Cân bằng hóa học 1. Sự có mặt của clo ở tầng bình lưu của khí quyển Trái đất dẫn đến sự bào mòn lớp ozon. Quá trình này được mô tả một cách đơn giản hóa như sau: k → 2Cl Cl2  1

Cl + ClO

k O3  → ClO 2

+

+

O2

k → Cl + 2O2 O3  3

k → Cl2 2Cl  4

a. Có thể áp dụng gần đúng trạng thái ổn định cho những tiểu phân nào? Tại sao? b. Tìm biểu thức mô tả tốc độ mất đi của ozon. Bậc riêng phần của Cl2 trong biểu thức bằng bao nhiêu? 2. Một hệ gồm 1 mol CO được đốt cháy với một lượng không khí vừa đủ (thành phần mol: 20% O2; 80% N2) theo phản ứng:  → 2CO2 2CO + O2 ←  Nhiệt độ bắt đầu đốt là 250C, dưới áp suất chung không đổi P = 1 atm. Xác định nhiệt độ ngọn lửa (nhiệt độ cực đại mà hệ có thể đạt được)? Cho các đại lượng nhiệt động ở 298K; Áp suất tiêu chuẩn P0 = 1 atm. Cấu tử O2(k) N2(k) CO(k) ∆G 0 tạo thành (Kj/mol)

S0 (J/mol.K) C 0p (J/mol.K)

–137,3

205 192 198 29,4 29,1 29,1 0 (Chấp nhận C 0p của các chất; ∆S của phản ứng không phụ thuộc vào nhiệt độ) Câu 2 : (2,0 điểm) Cân bằng trong dung dịch điện li

CO2(k) –394,5 214 37,1

1. Để xác định hàm lượng axit salixylic (axit o-hidroxibenzoic) trong một mẫu phân tích với tạp chất trơ, người ta tiến hành như sau: + Hoà tan 1,7614 gam mẫu phân tích vào nước tạo 100 ml dung dịch. + Hoà tan 0,595 gam KBr và 0,167 gam KBrO3 vào nước tạo 50 ml dung dịch. + Trộn 10 ml dung dịch mẫu phân tích với dung dịch H2SO4 loãng dư và 10 ml dung dịch (KBr + KBrO3). Lắc kĩ, đậy nắp kín, để yên vài phút. + Chuẩn độ hỗn hợp thu được hết 12,5 ml dung dịch NaAsO2 0,016M


a. Tính thành phần phần trăm khối lượng của axit salixylic trong mẫu phân tích? b. Chuẩn độ 10 ml dung dịch axit ở trên bằng dung dịch NaOH 0,01M thì nên chọn chất chỉ thị nào sau đây cho phù hợp? (Metyl đỏ-4,2 ; Brom thymolxanh-7,6; Trung tính -8). c. Tính sai số chuẩn độ nếu dùng metyl đỏ? E0 BrO3-/Br2 = 1,52V ; E0 Br2/Br- =1,085V; Pka =2,975; Bỏ qua quá trình phân li của nước.

2. Một dung dịch A gồm HAc 0,010 M và NH4Cl 0,200 M. a. Tính pH của dung dịch A.

b. Chuẩn độ 25,0 ml dung dịch A bằng dung dịch NaOH 0,020 M đến màu vàng rõ của Metyl đỏ (pT=6,2). Tính sai số chuẩn độ. Biết : K NH+ = 10-9,24; KHAc = 10-4,76. 4

Câu 3: (2,0 điểm) Điện hóa học Dung dịch X gồm Fe2(SO4)3 0,1500M ; FeSO4 0,0150M và KCl 2M. 1. Cần đặt điện thế tối thiểu là bao nhiêu để có quá trình oxi hóa và quá trình khử xảy ra đầu tiên ở mỗi điện cực khi điện phân dung dịch X ở pH=0. 2. Điện phân 100ml dung dịch X với cường độ dòng điện một chiều không đổi có I = 9,650A và trong thời gian 150 giây, thu được dung dịch Y. a) Tính khối lượng dung dịch giảm trong quá trình điện phân. b) Tính pH của dung dịch Y. c) Lắp một pin điện gồm một điện cực hiđro tiêu chẩn với một điện cực Pt nhúng vào dung dịch Y. Tính sức điện động của pin khi pin bắt đầu phóng điện và viết sơ đồ của pin. (Giả thiết rằng H2O bay hơi không đáng kể và thể tích của dung dịch không thay đổi trong quá trình điện phân) Cho: E0(Fe3+/Fe2+) = 0,771V; E0(2H+/H2) = 0,00V; β*[Fe(OH)]2+= 10-2,17; β*[Fe(OH)]+= 10-5,92; E0(Cl2/2Cl-)= 1,36V. Câu 4: (2,0 điểm) N – P, C – Si và hợp chất 1. Cho dãy chuyển hóa dưới đây: N2O Na HCl E A → B  → C  → D  →Z → E + Y + A H2O2 p,t NaOCl X + Y → A  → E ← Ure Fe3 O4 NaOH

Biết A và E là những bazơ yếu; X và Y là các đơn chất đều tồn tại ở thể khí, khi phân hủy 1 mol Z thu được 35,5 lít khí Y (đktc). a. Xác định các chất A, B, C, D, E, X, Y, Z. b. Viết đầy đủ các phương trình phản ứng xảy ra trong dãy chuyển hóa đã cho.


2. Một nguyên tố X có nhiều dạng thù hình, đa hóa trị, là nguyên tố thiết yếu cho cơ thể sống, không bao giờ tồn tại ở trạng thái đơn chất trong tự nhiên. Cho 1,55 gam X màu trắng tan hết trong axit HNO3 đặc nóng dư thu được 5,6 lít khí NO2 (đktc) là sản phẩm khử duy nhất và dung dịch Y chỉ chứa axit. Cho sơ đồ phản ứng sau đây: 0

+ ddBa ( OH ) + H SO + ddCuSO +2 NaOH 600 C → A   → B  → D  → E  →F (a) X  2

0

2

4

4

0

0

+ ddAgNO +H O + Ca ,t C 200 C 260 C (b) X   → G  → L  → M  → D  →Q 2

3

Biết A, B, D, E, F, G, L, M, Q đều là hợp chất của X có phân tử khối thỏa mãn: MA +

MG = 449;

MB

+

ML = 100;

MF +

MQ = 444;

MD +

MM = 180

1. Xác định nguyên tố X và các chất A, B, D , E, F, G, L, M, Q. 2. Viết các phương trình phản ứng theo sơ đồ phản ứng ở trên. Câu 5: (2,0 điểm) Phức chất, trắc quang 1. a. Thêm dần dung dịch NaCN vào dung dịch NiCl2 lúc đầu thu được kết tủa xanh X, sau đó kết tủa này tan ra tạo thành dung dịch màu vàng của chất Y. Nếu cho thêm tiếp NaCN đặc thì thu được dung dịch màu đỏ của chất Z. Hãy viết các phương trình phản ứng xảy ra trong thí nghiệm này. b. Cho biết Y và Z đều nghịch từ, dựa theo thuyết liên kết hóa trị (VB), hãy dự đoán cấu trúc phân tử của chúng. 2. Phương pháp phổ đo quang là phương pháp tiêu chuẩn để nghiên cứu các cân bằng hóa học có sự biến đổi màu sắc. Kỹ thuật này dựa vào định luật Beer phát biểu rằng độ hấp thụ tỉ lệ tuyến tính với đường đi của ánh sáng l (quãng đường mà ánh sáng phải đi qua chất) và nồng độ mol của tiểu phân hấp thụ. Xét phản ứng: 2NO2(k) ⇌ N2O4(k). (Lưu ý rằng khí NO2 có màu nâu đỏ còn N2O4 không có màu). Có hai tế bào chứa tỉ lệ NO2/N2O4 và có đường đi của ánh sáng là l1, l2 khác nhau, giả thiết rằng có thể đặt một áp suất p1 và p2 lên các tế bào để cho hai hỗn hợp đều có độ hấp thụ ngang nhau. Như vậy ta có thể xác định được hằng số cân bằng của phản ứng này. a. Xây dựng biểu thức tính hằng số cân bằng Kp phụ thuộc vào p1, p2 và tỉ lệ r = l1/l2 b. Trong một thí nghiệm thì l1 = 250mm và l2 = 50mm. sử dụng các giá trị thí nghiệm cho dưới đây hãy tính hằng số cân bằng Kp ở nhiệt độ thí nghiệm. Thí nghiệm I II

Độ hấp thụ AI AII

P1, mmHg 2,00 4,00

c. Tính tỉ lệ độ hấp thụ AI/AII Câu 6 (2,0 điểm): Quan hệ giữa hiệu ứng cấu trúc và tính chất 1. Cho cấu tạo của hợp chất hữu cơ E

P2, mmHg 11,00 23,5


Hãy chỉ rõ trạng thái lai hóa của từng nguyên tử N ở cấu tạo E và ghi giá trị Pka (ở 25 oC): 1,8; 6,0; 9,2 vào từng trung tâm axit trong công thức tương ứng với E. Giải thích. 2. Cho dãy hợp chất sau:

a. So sánh khả năng phản ứng thế electrophin của A với benzen và cho biết vị trí phản ứng ưu tiên ở A. Giải thích. b. So sánh nhiệt độ nóng chảy, nhiệt độ sôi của dãy hợp chất trên. Giải thích. Câu 7 (2,0 điểm) : Hidrocacbon 1. Một hidrocacbon mạch hở A có công thức phân tử C10H18 (khung cacbon gồm hai đơn vị isopren nối với nhau theo quy tắc đầu – đuôi). Oxi hóa A bằng dung dịch KMnO4 trong H2SO4, thu được hỗn hợp các chất A1, A2 và A3. Chất A1 (C3H6O) tác dụng với H2 (xúc tác Ni) tạo ancol bậc 2. Chất A2 (C2H4O2) phản ứng được với Na2CO3. Chất A3 (C5H8O3) chứa nhóm cacbonyl (C=O), phản ứng được với Na2CO3. a. Viết công thức cấu tạo của A1, A2, A3 và A. b. Viết công thức các đồng phân hình học của A. 2.a. Hãy hoàn thành sơ đồ phản ứng sau, giải thích sự hình thành X5 và X6: t

o

NaI

X1 (C8H 6Br 4)

Mg

X2 (C8 H 6Br 2)

X3 (C8 H 6)

X4 (C16H12) t o X7 (C16H12)

X5

HCl

X6 (C16 H13Cl) ,, (khong mat' mau nuoc brom) -

Br2

o-Xilen

b. Hãy tổng hợp nona-2,7-đien từ những hiđrocacbon chứa không quá 5C chỉ nhờ một phản ứng. Câu 8 (2,0 điểm): Xác định cấu trúc, đồng phân lập thể, danh pháp 1. Viết công thức cấu tạo và gọi tên các đồng phân bixiclooctan. 2. Vẽ công thức cấu trúc của các dẫn xuất 1,4-đioxan là sản phẩm đime hóa hợp chất (R)-1,2-epoxi-2metylpentan. 3. Một peptit X tự nhiên được chiết tách dưới dạng tinh thể màu trắng, có phân tử khối là 485. Thủy phân X và các phương pháp phân tích phù hợp đã xác định được thứ tự sắp xếp các α-amino axit trong X: phenylalanin, alanin, glyxin, prolin, isoleuxin. Biết rằng X phản ứng với axit nitrơ không giải phóng khí nitơ. Hãy xác định công thức cấu trúc của peptit X; công thức cấu tạo của các α-amino axit như sau:


Ph

COOH

COOH

NH2

H2N

COOH NH

NH2

Alanin

Phelylalanin

COOH

COOH

Prolin

Glyxin

NH2

Isoleuxin

Câu 9 (2,0 điểm): Cơ chế phản ứng Hãy trình bày cơ chế của các chuyển hóa sau: O

O O

a) O

MeONa C2H5OH

O O

OH OH Ph

O CH2O

b) NH CHPh2

CSA

Ph N Ph

Ph

Câu 10 (2,0 điểm): Tổng hợp các hợp chất hữu cơ ( Dạng sơ đồ phản ứng). Loline là một thành viên của họ 1- aminopyrrolizidines ( thường được gọi là lolines ), là một ankaloid . Các lolines là những hợp chất diệt côn trùng, ngăn chặn sự sinh sản của cỏ nấm cộng sinh trong cỏ endophytic thuộc chi Epichloë ( loài anamorphic : Neotyphodium ). Loline được tổng hợp theo sơ đồ sau:


HẾT. Mạc Thị Thanh Hà, SDDT0904769299- GV THPT Chuyên Vĩnh Phúc TRƯỜNG THPT CHUYÊN VĨNH PHÚC

HƯỚNG DẪN CHẤM MÔN HÓA HỌC – KHỐI 11 (Hướng dẫn chấm gồm 11 trang )

ĐỀ THI ĐỀ XUẤT Câu

1

ý 1 a b

Nội dung Có thể áp dụng nguyên lý trạng thái tĩnh cho Cl và ClO vì chúng là các gốc tự do rất hoạt động do đó có khả năng tạo phức cao dẫn đến tốc độ sinh ra chậm hơn nhiều so với tốc độ tạo thành nên nồng độ của chúng ít bị thay đổi. Áp dụng nguyên lí nồng độ dừng ta có: d [Cl ] = k1[Cl2 ] − k 2 [Cl][O3 ] + k3 [ClO][O3 ]-k 4 [Cl ]2 = 0 dt d [ClO] = k2 [Cl][O3 ]-k3 [ClO][O3 ] = 0 dt k ⇒ [ClO] = 2 [Cl ] k3 k k1[Cl2 ] − k2 [Cl][O3 ] + k3 . 2 [Cl ][O3 ]-k4 [Cl ]2 = 0 k3

⇒ [Cl ] =

1,0 0,25

0,25

k1 [Cl2 ] k4

d [O3 ] = k2 [Cl][O3 ]+k3 [ClO][O3 ] = 0 dt d [O3 ] k k k1 [Cl2 ][O3 ] = 0 ⇒− = k2 1 [Cl2 ][O3 ]+k3 . 2 dt k4 k3 k 4 −

⇒v=−

Điểm

0,25

1 1 d [O3 ] k k k = 2k2 1 [Cl2 ][O3 ] = 2k2 1 [Cl2 ]2 [O3 ] ⇒ v=2k2 1 [Cl2 ] 2 [O3 ] dt k4 k4 k4

Vậy: * v=2k2

1 k1 [Cl2 ] 2 [O3 ] ; k4

* Bậc của Cl2 là 1/2.

0,25


2

1,0 Phản ứng

CO

+

Ban đầu 1 Cân bằng (1 – x)

½ O2

 → ← 

0,5 0,5(1 – x)

CO2 0 x

(N2)

0,25

2 (mol) 2 (mol)

1

x.(7 − x) 2

→ K P (T ) =

3

(1 − x) 2 0 0 0 (1) = −257, 2 KJ/mol; ∆S 298 (1) = −86,5 J/mol.K → ∆H 298 (1) ≈ −283, 0 KJ/mol; ∆G298

0,25

0 ∆ST0 = −86, 5 J/mol.K; ∆H T0 = ∆H 298 + ∆C P (T − 298) = −281003, 4 − 6, 7T J/mol;

Ta có: ∆GT0 = ∆H T0 − T .∆ST0 = − RT ln K P (T ) 281003, 4 →T = 1 (1) x.(7 − x ) 2 8,314.ln + 79,8 3 (1 − x) 2

0,25

Mặt khác, ta có: Ở 298K, khi tạo thành x mol CO2, nhiệt phản ứng 283.x (KJ) làm 0,25 nóng hệ còn lại từ 298 đến T 283000 x → T = 298 + (2) 102 − 6, 7 x Từ (1) và (2): => T ≈ 2653K 2

1 a

b

-

-

+

5Br + BrO3 + 6H → 3Br2 + 3H2O (o) C6H4(OH)COOH + 3Br2 → C6H2(OH)Br3 + 3H+ + 3Br- + CO2 Br2 + AsO2- + 2H2O → 2Br- + H2AsO4- + 2H+ nBr2 sinh ra = 0,6. 10-3 mol nBr2 pứ AsO2- = 0,2. 10-3 mol => nBr2 pứ axit = 0,4. 10-3 mol n axit/ 10ml = 0,4. 10-3 /3; n axit/ mẫu = 4. 10-3 /3 % Mc7H6O3 = (4. 10-3 /3.138)/1,7614 = 10,45% C6H5OCOOH + NaOH →C6H5OCOONa + H2O Nc7H6O3 = 0,4. 10-3 /3 => Vdd NaOH =40/3(ml) C

0

C7H5O3Na

=

బ,ర య రబ ାଵ଴ య

= 5,7.10-3 (M) (đặt C7H5O3Na = NaA )

 → HA + OHA– + H2O ← 

Kb =

௫మ ହ,଻.ଵ଴షయ ି௫

Kb= 10-11,025

= 10-11,025 => x= 2,32.10-7 (M)

1,0 0,25

0,25


Ph= 7,37 => chọn brom thimol xanh Ph=7,6 c

Nếu dùng metyl đỏ thì Ph= 4,2 => [H+] =10-4,2 Đặt Vdd NaOH đã dùng chuẩn độ = V(ml) Nồng độ ban đầu:

0,25

1 .10 HA (axit salixylic): 75 M V + 10 0, 01.V NaOH: : M V + 10

Dừng chuẩn độ trước điểm tương đương nên thành phần của hệ sau chuẩn độ: HA; A– Điều kiện proton: [H+] = [A-] – C0NaOH 0 [H+] = CHA

Ka – C0NaOH [H ] + K a భ .ଵ଴ ళఱ

ଵ଴షమ,వళఱ

=> 10 = − ௏ାଵ଴ ଵ଴షర,మ ାଵ଴షమ,వళఱ => V ≈12,4422 ml ିସ,ଶ

0,25

+

଴,଴ଵ.௏ ௏ାଵ଴

Vtương đương = 40/3 (ml); Sai số q = – 6,684% 2 a

1,0 0,25

Tính pH của dung dịch A NH4Cl →

NH4+ + Cl -

Trong dung dịch có các cân bằng sau: HAc

 → Ac← 

+

H+

K1 = 10 - 4,76

NH4+

 → NH3 ← 

+

H+

K1 = 10 - 9,24

H2O

 → H+ ← 

+

OH-

Kw=10 -14

K1C1 >> K2C2, KW Bỏ qua sự phân li của nước và NH4+, tính theo:  → AcHAc + H+ K1 = 10 - 4,76 ←  C

0,01

[]

0,01 - x

x

Theo đltdkl ta có: x= [H+] = 4,083.10-4

b

0,25

x

x..x = K1 = 10−4,76 (0, 01 + x) ⇒

pH = 3,39.

Giả sử chuẩn độ hết HAc, chưa chuẩn độ NH4Cl vì KNH4 rất nhỏ

0,25


vì pT = 6,2 << pK : HAc

+

NaOH

NaAc +

H2O

VTD = 25.0,01/0,02 = 12,50 ml. Thành phần tại điểm tương đương: H2O, NH4+, Ac-

0,25

-Tính sai số chuẩn độ Ta có : q = P-1 =

CV C01V 0

−1 =

CV − C01V 0 C 01V 0

=−

C'HAc C HAc

Theo đk proton mức không C/HAc, ,H2O, NH4+, Ac[H+]= [OH- ] + [NH3] – ([HAc] - C'HAc) => C'HAc = [H+ ] - [OH-] - [NH3] + [HAc]

K W  C + C01 C02 K NH +4 h  + 01 − ⇒ q = − h − = -0,0169  01 h  CC C K NH + + h K HAc + h  4

3

q =− 1,69 %.

1 Bán phản ứng đầu xảy ra ở mỗi điện cực là + Điện cực A (+) : 2Cl- ⇌ Cl2 + 2e + Điện cực K (-) : Fe3+ + 1e ⇌ Fe2+ Trong dung dịch X có C(Fe3+) = 0,3M; C(Fe2+) = 0,03M; C(H+) = 1M; C(Cl-) = 2M; Na+; SO42-. Thế khử của mỗi cặp ở mỗi điện cực là: Ea = E(Cl2/2Cl-) = 1,36 + (0,0592/2)lg(1/22) = 1,3422V Ở Ph = 0; không có quá trình proton hóa của ion kim loại, vì vậy ta có: Ec = E(Fe3+/Fe2+) = 0,771 + 0,0592lg(0,3/0,015) = 0,848V => Cần đặt điện thế tối thiểu để xảy ra quá trình oxi hóa Cl- và quá trình khử ion Fe3+ là: V= 1,342 – 0,848 = 0,494 V 2 a

b

Số mol electron trao đổi trong quá trình điện phân là: ne = It/F = 9,65.150/96500 = 0,015 (mol) Các phản ứng xảy ra tại các điện cực: Cực (+): 2Cl- → Cl2 + 2e (1) -3 ← 0,015 n 7,5.10 3+ Cực (-) : Fe + 1e → Fe2+ (2) n 0,015 0,015 Theo (1) và (2) và giả thiết cho thấy ion Cl- và Fe3+ đều dư. Vậy khối lượng dung dịch giảm là: m = m(Cl2) = 71.0,015/2 = 0,5325 gam. Dung dịch Y có: C(Fe3+)=(0,03-0,015)/0,1= 0,15M; C(Fe2+)=(0,0015+ 0,015)/0,1= 0,165M; C(Cl-)= (0,2 – 0,015)/0,1=1,85M; Na+; SO42-. Có cân bằng:

0,5 0,25

0,25

1,5 0,25

0,25 0,25


Fe3+ + H2O ⇌ Fe(OH)2+ + H+ β*[Fe(OH)]2+= 10-2,17 (3) Fe2+ + H2O ⇌ Fe(OH)+ + H+ β*[Fe(OH)]+= 10-5,92 (4) + -14 H2O ⇌ H + OH Kw = 10 (5) 2+ + -2,17 + + -5,92 Xét [Fe(OH) ].[ H ] ≃ 0,15. 10 >> [Fe(OH) ].[ H ] ≃ 0,165. 10 >> Kw Vì vậy Ph là do cân bằng (3) quyết định. Xét cân bằng: Fe3+ + H2O ⇌ Fe(OH)2+ + H+ β*[Fe(OH)]2+= 10-2,17 C0 0,15 [] 0,15 – x x x 2 -2,17 x /(0,15 – x) = 10 => x = 0,0286M => pH = 1,544. c

4

1 a

Theo kết quả tính ở phần (b) và cho thấy ion Fe2+ tạo phức hidroxo không đáng kể, nên ta có: E(Fe3+/Fe2+) = 0,771 + 0,0592.lg(0,15- 0,0286)/0,165 = 0,763 (V) Vậy E(pin) = E(cao) - E(thấp) = 0,763 – 0,00 = 0,763 (V) Do E(Fe3+/Fe2+) > E0(2H+/H2) => sơ đồ pin là: A(-) Pt,H2 (1 atm)/H+ (1M)// Fe3+(0,077M); Fe2+(0,11M) / Pt (+) K.

B: NaNH2

C: NaN3

D: HN3

E: N2H4

X: H2

Viết phương trình phản ứng 2NH3

HN3

0,25

+ 2Na → 2NaNH2

2NaNH2 NaN3

+

+ H2

N2O → NaN3

+ NaOH

+ HCl → NaCl

+

(1) + NH3

HN3

12[N2H5]+[N3]- → 3N2H4 2N2 + 3H2

Fe3O4 ⇀ ↽ P ,t

(NH2)2CO + NaOCl

0,25

(4) + 16NH3

+ 19N2

2NH3

+ H 2 O2 → N 2 H4

(2) (3)

+ N2H4 → [N2H5]+[N3]-

2NH3

0,25

Z: [N2H5]+[N3]-

Y: N2 b

0,25

1,0 0,25

Xác định các chất A -> Z: A: NH3

0,25

(6)

+ 2H2O

+ 2NaOH → N2H4

(5)

0,25

(7)

+ H2O + NaCl+ Na2CO3 (8)

2

1,0 * Theo giả thiết, X phải là phi kim. 0,25

Áp dụng bảo toàn mol electron X

X+n

+

ne

N+5

+

1e

N+4


a

a.n

0,25

0,25

(mol) => a.n = 0,25 (1) => MX. a = 1,55 (2) (1), (2) => M X =

31 .n 5

=> n = 5 ; MX = 31 => X là P (photpho)

* Xác định các chất trong sơ đồ (a) và (b) : - X tác dụng Ca → G là Ca3P2. - MA +

MG = 449 → MG = 449 – 182 = 267, mặt khác X tác dụng với

0,25

dung dịch Ba(OH)2 tạo A phải là muối → A là Ba(H2PO2)2. - G tác dụng với H2O → L là PH3. - MB

+

ML = 100 → MB = 100 – 34 = 66, mặt khác A tác dụng với

H2SO4 tạo B → B là H3PO2 - B tác dụng với CuSO4 có tính oxi hóa tạo D tác dụng được với NaOH → D là H3PO4 → E là Na2HPO4 - MD +

MM = 180 → MM = 180 – 98 = 82, mặt khác L tác dụng với

AgNO3 có tính oxi hóa tạo M, M mất nước tạo D → M là H3PO3. - Nhiệt phân D mất nước tạo Q → Q là H4P2O7 - MF +

MQ = 444 → MF = 444 - 178 = 266, mặt khác nhiệt phân E

tạo F → F là Na4P2O7

Vậy các chất trong sơ đồ phản ứng đã cho là: A – Ba(H2PO2)2 ; F- Na4P2O7 ; b

;

B – H3PO2 ;

D- H3PO4 ;

G - Ca3P2; L – PH3 ;

E – Na2HPO4;

M – H3PO3 ;

Q – H 4 P2 O7

Phương trình phản ứng: 8P

+

3Ba(OH)2 + 6H2O → 3Ba(H2PO2)2

Ba(H2PO2)2

+

H2SO4

→ 2H3PO2

H3PO2 + 2CuSO4 + 2H2O → 2Cu H3PO4

+

2NaOH →

+ H3PO4 Na2HPO4

0

600 C → Na4P2O7 2Na2HPO4 

2P

+

0

t C 3Ca →

+

Ca3P2

+

+

H 2O

+ 2PH3

(1)

BaSO41

(2)

+ 2H2SO4

(3)

H 2O

(4) (5) (6)

0,05 x10


Ca3P2 PH3 +

5

+

3H2O

6AgNO3 + 3H2O → 0

4H3PO3

200 C  →

2H3PO4

260 C  →

0

3Ca(OH)2 + 2PH3 6Ag + 6 HNO3

PH3

+

H 4 P2 O 7

+

(7) + H3PO3

(8)

3H3PO4

(9)

H2O

(10)

1 a

b

1,0 NiCl2 + 2CN– + 2H2O Ni(OH)2↓ (X, xanh) + 2HCN + 2Cl– 0,25 – 2– – Ni(OH)2 + 4CN [Ni(CN)4] (Y, màu vàng) + 2OH 2– – [Ni(CN)4] + CN [Ni(CN)5]3– (Z, màu đỏ) Ni2+ cấu hình d8, ion phức chất [Ni(CN)4]2– nghich từ do vây sẽ lai hóa trong, hai 0,25 e độc thân sẽ ghép đôi. Vói phối trí 4 sẽ phù hợp với dạng dsp2, cấu trúc hình học vuông phẳng. ( có thể suy luận do CN- là phối tử trường mạnh )

Ion phức chất [Ni(CN)5]3– nghịch từ do vậy sẽ lai hóa trong dạng dsp3 lưỡng chóp tam giác. Số 0,25 phối trí 5 trong [Ni(CN)5]3– (có thể suy luận từ sự lai hóa vì ion d8 chỉ còn tối đa 5 AO trống trong trường hợp lai hóa trong).

Cấu trúc hình học

0,25

2 a

Dựa vào phương trình trạng thái khí lý tưởng ta có: C=

n P = V RT

Theo định luật Beer có: A = ε cl = kPl , với A là độ hấp thụ, ɛ và k là những hằng số. Hằng số cân bằng của phản ứng 2NO2(k) ⇌ N2O4(k) là:

1,0 0,25


KP =

pN 2O4 p

2 NO2

=

1 xN2O4 1 (1 − xN2O4 ) . 2 = . 2 P xNO p xNO 2 2

Với p là áp suất chung của hệ; x là phần mol của NO2 Do độ hấp thụ cuat hai cuvet bằng nhau ⇒ p1(NO2).I1 = p2(NO2).I2 (vì chỉ có NO2 hấp thụ ánh sáng). Biểu thức này có thể viết: (1) p1.x1.(NO2).I1 = p2.x2(NO2).I2 Tỉ số phản ứng ở các cuvet phải ngang nhau do các hỗn hợp đều cân bằng. 1 (1 − x1 ( NO2 )) . p1 x12 ( NO2 ) 1 (1 − x2 ( NO2 )) KP = . p2 x22 ( NO2 )

KP =

0,25

(2) (3)

Từ (1), (2), (3) và r = l1/l2 ta có: KP =

r ( r − 1).( p2 − rp1 ) ( r 2 p2 − p2 ) 2

b

Thay các giá trị thực nghiệm vào các biểu thức ta thu được Thí nghiệm 1: KP = 0,0131 Thí nghiệm 2: KP = 0,0120 ⇒ KP = (0,0131 + 0,0120)/2 = 0,01255

0,25

c

Từ định luật Beer ta có với mỗi tế bào (i = 1, 2)

0,25

p ( NO2 ) p x ( NO2 ) AI = = AII Pi ( NO2 ) p x ( NO2 ) I i II

I i II i

I i II i

Với pIi và pIIi là các áp suất chung, xiI và xiII là phần mol của NO2. Áp dụng biểu thức: K P =

(1 − x) px 2

1/2

⇒ x = [-1 + (1 + 4KP.p) ]/2KP.p

Đối với cuvet thứ nhất ta có: AI p I x I ( NO2 ) [-1+(1+4K P . p1I )1/2 ] = II1 1II = = 0, 511 AII p1 x1 ( NO2 ) [ − 1 + (1 + 4 K P . p1II )1/2 ]

Tương tự đối với cuvet thứ hai có: AI p I x I ( NO2 ) [-1+(1+4K P . p2I )1/ 2 ] = II2 2II = = 0,516 AII p2 x2 ( NO2 ) [ − 1 + (1 + 4 K P . p2II )1/ 2 ] ⇒ AI/AII = (0,511 + 0,516)/ 2 = 0,5235

6

1 sp 2

COOH

N

sp 3 N sp 2

*

H

1,0 0,25

NH 2

(E)

- Nguyên tử N nhóm NH ở trạng thái lai hóa sp2, cặp e chưa chia ở obitan p xen

0,25


phủ với 5 obitan p khác tạo thành hệ thơm được lợi về mặt năng lượng nhưng “mất” tính bazơ. - Nguyên tử N thứ hai ở trạng thái lai hóa sp2, cặp e chưa chia ở obitan sp2 không tham gia vào hệ thơm nên còn tính bazơ. - Nguyên tử N nhóm NH2 ở trạng thái lai hóa sp3. * 6,0

0,25

COOH 1,8

H N N

NH 3 9,2

H

- Nhóm NH3+ là axit liên hợp của nhóm H2Nsp3 , nhóm NH+ là axit liên hợp của nhóm Nsp2. - Bazơ càng mạnh thì axit liên hợp càng yếu, vì thế giá trị 9,2 là thuộc nhóm NH3+ còn giá trị 6,0 thì thuộc nhóm NH+. 2 a

0,25

1,0 - Mật độ e π ở mỗi vị trí của A (6e/5 vị trí) lớn hơn ở mỗi vị trí trong vòng benzen 0,25 (6e/6 vị trí) nên A dễ tham gia phản ứng thế electrophin hơn benzen. - Sự tạo thành phức σ ở vị trí 2 (ở giai đoạn quyết định tốc độ phản ứng) bền hơn ở vị trí 3 do điện tích dương 0,25

b

* Nhiệt độ nóng chảy: G > E > D > A > B > 0,25 * Nhiệt độ sôi: G > E > D > B > C > A 0,25 * Giải thích: - G, E và D có phân tử khối lớn hơn và có nhiều nhóm phân cực hơn so với A, B, C; - G tạo liên kết hiđro liên phân tử mạnh hơn E nên tonc và tos đều biến đổi theo thứ tự: G > E > D > A, B, C. - Ở trạng thái rắn, lực Van deVan (Fv~ p.p’/rn với n ≥ 4) phụ thuộc chủ yếu vào khoảng cách giữa các phân tử (r). Vì r < r < r nên tonc theo giảm theo thứ tự A

B

C

A > B > C. - Ở trạng thái sôi, lực Van deVan phụ thuộc chủ yếu vào điện tích p và p’ của lưỡng cực (vì khi đó khoảng cách giữa các phân tử quá lớn). Vì µ B> µ C> µ A nên tos giảm theo thứ tự B > C > A. 7

1 a

1,0 Công thức cấu tạo: A1: CH3 – CO – CH3

A2: CH3 – COOH

0,125 điểm cho mỗi


CH3 - C = CH - CH2 - CH2 - C = CH- CH3

HOOC - CH2 - CH2 - C - CH3

b

A3 : 2 đồng phân hình học H3C

H2C C

H3C

CH3

C H

2 a

CH2

C

CH3

A:

O

H3C

C

H3C

H

CH3

H2C C

CH2

H

0,25 cho mỗi công thức

H

C

C

H3C

chất

C

H3C

CH3

1,0 CH 3

CHBr 2

Br2

o CH3 t

CHBr NaI

CHBr (X2)

(X1)

0,05 cho mỗi công thức

(X7)

Mg

CHBr 2

(X3) (X5)

Cl -

(X6)

H+

to

(X4)

Cl

b 8

1

LnM=C<

H 3CHC CHCH 3

0,5 H 3CHC CHCH 2 CH 2CH 2CH=CHCH3

0,5 Bixiclooctan có các đồng phân sau:

bixiclo[4.2.0]octan

bixiclo[5.1.0]octan

bixiclo[4.1.1]octan bixiclo[3.2.1]octan

0,25

bixiclo[3.3.0]octan

0,25

bixiclo[2.2.2]octan

0,5

2 Công thức cấu trúc của các dẫn xuất 1,4-đioxan thế khi đime hóa hợp chất (R)-1,2epoxi-2-metylpentan: CH3

CH 3

O

n-C 3 H 7

CH 3 C 3 H 7 -n

O

n-C 3 H 7

3

O CH3

O

C 3 H 7 -n CH3

O

n-C 3 H 7

C 3 H 7 -n CH 3

O

O n-C 3 H 7

CH3

O CH 3 C 3 H 7 -n

O n-H 7 C 3 H 3C

H 3C

O

O

O

C 3 H 7 -n

O CH 3 C 3 H 7 -n

C 3 H 7 -n CH 3

n-H 7 C 3 H 3C

CH 3

C 3 H 7 -n CH3

O

O

O C 3 H 7 -n

0,25 CH 3 C 3 H 7 -n

0,25 1,0 * Theo giả thiết, X được tạo bởi các α-amino axit là: Phe: 165; Ala: 89; Gly: 75; 0,25

Pro: 115; Ile: 131. Mỗi liên kết peptit tạo thành từ hai α-amino axit sẽ loại đi 1 phân tử nước. Dãy Phe-Ala-Gly-Pro-Ile có 4 liên kết peptit, số khối mất đi là 4x18 = 72, số khối còn lại là 575 - 72 = 503. * Trong khi đó, phân tử khối của peptit X là 485, sự chênh lệch về số khối là 503- 0,25


485 = 18, đúng bằng phân tử khối của 1 phân tử nước. Mặt khác, X phản ứng với axit nitrơ không giải phóng khí nitơ, chứng tỏ X không còn nhóm NH2 tự do, tức là X có cấu trúc vòng khép kín. * X là một peptit tự nhiên nên các α-amino axit cấu tạo nên X phải có cấu hình 0,25 L (L-Phenylalanin, L-Alanin, L-Prolin và L-Isoleuxin, Glyxin không có cacbon bất đối nên không có đồng phân quang học). Vậy X có công thức cấu trúc như sau (có thể sử dụng công thức chiếu Fisơ để viết công thức của X): Phe

0,25

Ala

O

NH

O

HN

NH

Gly

O HN

N

Ile O

9 a

10

O Pro

2,0 0,25 x2 (viết đúng được 3 chất cho 0,25đ )

b

0,25x 2

c

0,25x 2 (viết đúng 3 chất cho 0,25)

d

0,25x 2

2,0


0,25x 3

0,25x 3

0,25x 2

HẾT. Người ra đề: Mạc Thị Thanh Hà, SDDT0904769299.Trường THPT Chuyên Vĩnh Phúc


KỲ THI HỌC SINH GIỎI CÁC TRƯỜNG THPT CHUYÊN KHU VỰC DUYÊN HẢI VÀ ĐỒNG BẰNG BẮC BỘ LẦN THỨ X, NĂM 2017 ĐỀ THI MÔN: HÓA HỌC Lớp 11 Thời gian: 180 phút (Không kể thời gian giao đề) Ngày thi: 15/4/2017 (Đề thi gồm 3 trang)

ĐỀ CHÍNH THỨC

Bài 1. (2 điểm) 1. Khi đo vo của phản ứng : I- + OCl- Cl- + OI- diễn ra ở 25oC . 1 Trong dung dịch có pH cố định và các nồng độ [I-]o ( mmol.l-1 ) các ion I-, OCl- khác nhau người ta thu được các [OCl-]o (mmol.l-1 ) 1 kết quả sau: v (10-5 mol.L-1) 6,1 Chứng minh rằng cơ chế sau đây phù hợp với các dữ kiện thực nghiệm : k1 ⇀ OCl- + H2O ↽ HOCl + OH(nhanh)

1 1,2 7,3

1,1 1 6,7

1,3 1 7,9

k−1

-

k2  → HOCl + I HOI + Cl(chậm) k3  → HOI + OH H2O + IO (nhanh) 2. Ở 1173K, hằng số cân bằng của quá trình phân ly I2(khí) thành 2I(khí) là Kp = 0,04867. Ở 1073K, áp suất ban đầu của I2 là 0,0631 atm, áp suất chung của hệ lúc cân bằng là 0,0750 atm. a. Tính ∆H° ở 1100 K. (Cho rằng ∆H° không phụ thuộc nhiệt độ). b. Khi cân bằng được thiết lập tại nhiệt độ T, hằng số Kp bằng một nửa áp suất chung. Tính phần mol của I(khí) trong hỗn hợp cân bằng. Bài 2. (2 điểm) Histidine là một amino axit thiết yếu trong cơ thể người, động thực vật. Cấu trúc phân tử của histidine khi đã bị proton hoá như sau: Ở dạng này, histidin được coi như một axit 3 lần axit (kí hiệu là H3A2+) có các hằng số phân li axit tương ứng: pKai = 1,82; 6,00 và 9,17. 1. Hòa tan hoàn toàn 4.10-4 mol H3ACl2 (có thể viết dạng HA.2HCl) trong nước, thu được 40,0 mL dung dịch A. Tính pH của dung dịch A. 2. Nếu dùng dung dịch NaOH 0,01 M để chuẩn độ dung dịch A thì pH của dung dịch thu được là bao nhiêu sau khi đã cho hết 40,0 mL dung dịch NaOH? 3. Điện tích trung bình trên toàn phân tử amino axit trong dung dịch nước có thể được tính theo điện tích của từng dạng tồn tại của các cấu tử trong dung dịch. Hãy tính điện tích trung bình của Histidine trong dung dịch khi pH của dung dịch là 6.0. 4. Trộn 50 mL dung dịch Cu(NO3)2 0,02 M với 50 mL dung dịch Histidine 0,20 M rồi điều chỉnh pH của dung dịch thu được bằng 6,0 thu được 100,0 mL dung dịch B. Biết Cu2+ và Histidine có thể tạo ra hai phức bền CuA+ (β1 = 2,0.108) và CuA2 (β 2 = 8,0.1018 ). β*Cu2+ = 10-8 Tính nồng độ cân bằng của Cu2+ tự do trong dung dịch B. Bài 3. (2 điểm) Ghép 2 điện cực thành một tế bào Ganvani ở 25oC như sau: Điện cực đồng gồm một thanh Cu nhúng vào dung dịch CuSO4 0,15M; Điện cực kẽm gồm một thanh Zn nhúng vào dung dịch ZnSO4 2,00M. 1. Viết sơ đồ của tế bào Ganvani trên. Viết phương trình phản ứng xảy ra tại các điện cực và phản ứng tổng quát khi tế bào hoạt động. 2.Tính sức điện động của pin tại nhiệt độ 25oC. 3. Khi pin phóng hết điện, nồng độ các chất trong mỗi dung dịch là bao nhiêu? 4. Đặt một hiệu điện thế ngoài vào tế bào để tiến hành nạp điện trong vòng 1 giờ với cường độ dòng điện 1,0 A. Hỏi sức điện động của pin ngay sau khi điện phân là bao nhiêu? Biết thể tích dung dịch ở mỗi điện cực luôn không đổi là 1 lít. Bỏ qua ảnh hưởng của quá thế và điện trở của dung dịch.

1


Cho

o o ECu 2+ /Cu = 0,337 V và E Zn 2+ /Zn = -0,760 V .

Bài 4. (2 điểm) 1. Đơn chất X tác dụng mãnh liệt với dung dịch kiềm, nhưng chỉ tác dụng với nước ở nhiệt độ cao (khoảng 800oC). X bền trong các axit, ngay cả nước cường thủy, chỉ tan trong hỗn hợp hai axit HF và HNO3. Bột mịn X tác dụng được với hơi HF hay tác dụng được với Mg ở khoảng 800oC. X lại có thể được điều chế bằng cách dùng Mg tác dụng với oxit của nó khi đốt cháy hỗn hợp. Hãy cho biết X là đơn chất nào? Viết phương trình hóa học minh họa các phản ứng xảy ra. 2. Hydro mới sinh là một tác nhân khử có hiệu quả nhất. Xử lý một lượng dung dịch natri nitrit bằng hỗn hống natri kim loại cho ra một muối X có 43,40% natri và 26,42% nitơ về khối lượng, còn lại là oxi. Một sản phẩm khác của phản ứng này là natri hydroxit. Để giữ sản phẩm tinh khiết, quá trình tổng hợp được tiến hành trong khí quyển trơ như môi trường nitơ hay argon. a. Xác định công thức muối X. Biết mỗi anion trong X chứa 4 nguyên tử. b. Vẽ công thức Lewis anion của muối X. c. Nếu phản ứng được tiến hành trong không khí thì những tạp chất nào có thể sinh ra? d. Viết phương trình tổng hợp muối X. Bài 5. (2 điểm) Axit HIn trong nước phân ly như sau: HIn (màu 1) H+ + In− (màu 2) Đo mật độ quang của dung dịch HIn 5,00.10−4 M trong NaOH 0,1 M và trong HCl 0,1 M ở bước sóng 485 nm và 625 nm với cuvet 1,00 cm. Trong dung dịch NaOH 0,1 M A485 = 0,052 A625 = 0,823 Trong dung dịch HCl 0,1 M A485 = 0,454 A625 = 0,176 1. Tính hệ số hấp thụ mol của In− và HIn ở bước sóng 485 và 625 nm. 2. Tính hằng số phân ly axit của HIn, nếu trong dung dịch đệm pH 5,00 chứa một lượng nhỏ chất chỉ thị có mật độ quang là 0,472 ở 485 nm và 0,351 ở 625 nm 3. Mật độ quang của một dung dịch HIn 2,00.10−4 M tại 485 và 625 nm (cuvet 1,25 cm) là bao nhiêu nếu dung dịch được đệm ở pH bằng 6,00? 4. Chuẩn độ 25,00 mL dung dịch axit hữu cơ tinh khiết yếu HX với chất chỉ thị là phenolphtalein tới điểm cuối chuẩn độ thì cần 24,20 mL dung dịch NaOH chuẩn. Khi thêm đúng 12,10 mL dung dịch NaOH vào 25,00 mL dung dịch axit HX, trong dung dịch có một lượng nhỏ chất chỉ thị HIn, mật độ quang đo được là 0,306 ở 485 nm và 0,555 ở 625 nm (cuvet 1,00 cm). Tính pKa cho HX. Bài 6. (2 điểm) 1. Hằng số tốc độ k của phản ứng xà phòng hóa các dẫn xuất thế m- và p-metylbenzoat bằng NaOH trong đioxan/nước được liệt kê dưới đây. X-C6H4-COOCH3 + NaOH → X-C6H4-COONa + CH3OH k(M-1.phút-1) k(M-1.phút-1) (1) Metyl p-nitrobenzoat 102 (4) Metyl m-brombenzoat ? (2) Metyl m-nitrobenzoat 63 (5) Metyl benzoat 1,7 (3) Metyl m-clobenzoat ? (6) Metyl p-aminobenzoat 0,06 a. So sánh giá trị k của (3), (4), (5). b. Dựa trên cấu trúc (gồm cả các trạng thái cộng hưởng có thể có) giải thích vì sao chất (3) có tốc độ thủy phân cao hơn (4). Còn tốc độ thủy phân của chất (6) rất nhỏ. 2. Axit tactric (axit 2,3-đihidroxibutandioic) A B C D được điều chế theo 4 thí nghiệm thu được 4 dạng [αD, 25oC] -12 +12 0 0 A, B, C, D. Tính chất của mỗi dạng cho ở bảng sau: tnc 171 171 205 140 Xác định cấu trúc của C, D. Độ tan 100g H2O 139 139 21 125 3. Vì sao ở điều kiện thường, mỡ động vật thường là chất rắn, còn dầu thực vật thường là chất lỏng. Bài 7. (2 điểm) 1. Hidrocacbon A quang hoạt có công thức phân tử C8H12. Khi thực hiện phản ứng hidro hóa A bằng xúc tác Pt, đun nóng thu được B[C8H18] không quang hoạt, nhưng nếu dùng xúc tác Lindlar lại

2


thu được C [C8H14] quang hoạt. Cho A phản ứng Na trong NH3 lỏng thu được hợp chất D [C8H14] không quang hoạt. Xác định cấu trúc các chất A, B, C, D. 2. Từ m-xilen, 3-oxobutanal và các hóa chất phụ khác, hãy viết sơ đồ điều chế 2,7-đimetylnaphtalen và cronen (hình bên):

(cronen: C24H12)

Bài 8. (2 điểm) Hiđro hóa hoàn toàn naphtalen người ta thu được đecalin (C10H18). Oxi hóa đecalin thì thu được hỗn hợp các decalon (C10H16O). 1. Trong hỗn hợp decalon nói trên có tối đa bao nhiêu đồng phân ? Giải thích. 2. Hãy vẽ công thức các đồng phân lập thể của decal-1-on, biết rằng hai vòng 6 cạnh trong phân tử decalon đều ở dạng ghế. 3. Hòa tan X (một trong số các đồng phân decalon) vào dung dịch bazơ thì X bị đồng phân hóa thành Y tới 95%. Lập luận để xác định X, Y. 4. Trong dung dịch bazơ, decal-1-on phản ứng với benzanđehit cho hợp chất T, với metyl vinyl xeton cho hợp chất U. T và U đều làm mất màu nước brom. Hãy viết công thức cấu tạo của T và U. Bài 9. (2 điểm) Viết cơ chế các phản ứng sau:

1.

2.

O Br N

Br(H2C)3

Zn

O

THF

OEt

OH N

O

O

N

O

1. HCl, EtOH

O

2. H2O

O

Bài 10. (2 điểm) Loline là một thành viên của họ 1- aminopyrrolizidines (thường được gọi là lolines), là một ankaloid . Các lolines là những hợp chất diệt côn trùng, ngăn chặn sự sinh sản của cỏ nấm cộng sinh trong cỏ endophytic. Loline được tổng hợp theo sơ đồ sau: OMe MeO OMe COOMe OMe COOMe HOH2C H2/Pd MsCl C D OH N N O Cl CH2OAc 1. LiAlH4 H+ SOCl2 H2/Pt MeONa E G H K 2. Ac2O Py OAc C8H13NO2 N

3


NHMe CrO3,H2SO4 EtOH

L

1. NH3 2. Br2/NaOH

M

CH3I

O

Bazo

N

Hãy viết công thức cấu tạo của các chất C, D, E, G, H, K, L, M. -------------- HẾT -------------(Thí sinh được sử dụng bảng tuần hoàn. Cán bộ coi thi không giải thích gì thêm) Họ và tên thí sinh: .................................................................... Số báo danh: …………………

4


KỲ THI HỌC SINH GIỎI CÁC TRƯỜNG THPT CHUYÊN

KHU VỰC DUYÊN HẢI VÀ ĐỒNG BẰNG BẮC BỘ LẦN THỨ X, NĂM 2017 ĐÁP ÁN MÔN: HÓA HỌC LỚP: 11 (Đáp án gồm...... trang) Bài 1. (2 điểm) 1. (Hòa Bình) Khi đo vo của phản ứng : I- + OCl- Cl- + OI- diễn ra ở 25oC . 1 1 1,1 Trong dung dịch có pH cố định và các nồng độ [I-]o ( mmol.l-1 ) các ion I-, OCl- khác nhau người ta thu được các [OCl-]o (mmol.l-1 ) 1 1,2 1 -5 -1 kết quả sau: v (10 mol.L ) 6,1 7,3 6,7 Chứng minh rằng cơ chế sau đây phù hợp với các dữ kiện thực nghiệm : k1 ⇀ OCl- + H2O ↽ HOCl + OH(nhanh)

1,3 1 7,9

k−1

k2  → HOCl + IHOI + Cl(chậm) k3  → HOI + OH H2O + IO (nhanh) 2. (Hà Nam) Ở 1173K, hằng số cân bằng của quá trình phân ly I2(khí) thành 2I(khí) là Kp = 0,04867. Ở 1073K, áp suất ban đầu của I2 là 0,0631 atm, áp suất chung của hệ lúc cân bằng là 0,0750 atm. a. Tính Kp ở 1100 K. (Cho rằng ∆H° không phụ thuộc nhiệt độ). b. Khi cân bằng được thiết lập tại nhiệt độ T, hằng số Kp bằng một nửa áp suất chung. Tính phần mol của I(khí) trong hỗn hợp cân bằng. Ý Nội dung Điểm Giả sử biểu thức tốc độ phản ứng có dạng: v = k . [I-]x . [OCl-]y Xét 2 thí nghiệm đầu ta có : v = k . [1]x . [OCl-]y = k .[OCl-]y , giả sử y = 1 1 Với thí nghiệm 1 : k1 =6,1 . 10-2 s-1 Với thí nghiệm 2 : k2 =6,08 . 10-2 s-1 - x y Xét 2 thí nghiệm sau : v = k . [I ] . [1] = k . [I-]x, giả sử x = 1 Với thí nghiệm 3 : k3 =6,09 . 10-2 s-1 Với thí nghiệm 4 : k4 =6,08 . 10-2 s-1 0,25 Ta thấy : k1 ~ k2 ~ k3 ~ k4, giả thiết là đúng . Vậy, phương trình động học là: v = k . [I-] . [OCl-] 0,25 k k [I ][ClO ][H 2 O] Giai đoạn chậm quyết định tốc độ: v = k2. [HOCl].[I-] = 1 2 0,25 k−1[OH - ] Do pH cố định nên [OH-] cố định, dung dịch rất loãng, H2O không tham gia vào phương trình phản ứng, [H2O] không đổi. k k [H O] 0,25 Vậy , v = k . [I-] . [OCl-] phù hợp thực nghiệm với k = 1 2 2 k−1[OH ] a. Có cân bằng: I2(k) ⇌ 2I(k) P(I2)o – x 2x 2 Ở thời điểm cân bằng: P(I2)cb = P(I2)o – x; Pchung = P(I2)o + x Ở 1073K, x = 0,0750 – 0,0631 = 0,0119 atm P(I)cb = 2x = 0,0238 atm P(I2)cb = 0,0631 – 0,0119 = 0,0512 atm 2 P( I )cb K ∆H o  1 1  o K= = 0, 01106 → ln 1173 = −   ⇒ ∆H = 155, 052 J = 155 kJ 0,25 P( I 2 )cb K1073 R  1073 1173  K ∆H o  1 1  Như vậy ở 1100K ta có : ln 1100 = −   ⇒ K1100 = 0, 01694 0,25 K1073 R  1073 1100  o b. Có cân bằng: I2(k) ⇌ 2I(k) Giả sử P I2 = 1 atm CB 1o – x 2x → Pcb = 1 + x 4 x2 1 1 1 0,25 = PCB = [1 + x ] ⇒ x = KP = 1− x 2 2 3 0,25

1


Áp suất của I2 và I đều là 2/3 → Phần mol I(k) lúc cân bằng là 0,50 Bài 2. (2 điểm) (Hưng Yên) Histidine là một amino axit thiết yếu trong cơ thể người, động thực vật. Cấu trúc phân tử của histidine khi đã bị proton hoá như sau: Ở dạng này, histidin được coi như một axit 3 lần axit (kí hiệu là H3A2+) có các hằng số phân li axit tương ứng: pKai = 1,82; 6,00 và 9,17. 1. Hòa tan hoàn toàn 4.10-4 mol H3ACl2 (có thể viết dạng HA.2HCl) trong nước, thu được 40,0 mL dung dịch A. Tính pH của dung dịch A. 2. Nếu dùng dung dịch NaOH 0,01 M để chuẩn độ dung dịch A thì pH của dung dịch thu được là bao nhiêu sau khi đã cho hết 40,0 mL dung dịch NaOH? 3. Điện tích trung bình trên toàn phân tử amino axit trong dung dịch nước có thể được tính theo điện tích của từng dạng tồn tại của các cấu tử trong dung dịch. Hãy tính điện tích trung bình của Histidine trong dung dịch khi pH của dung dịch là 6.0. 4. Trộn 50 mL dung dịch Cu(NO3)2 0,02 M với 50 mL dung dịch Histidine 0,20 M rồi điều chỉnh pH của dung dịch thu được bằng 6,0 thu được 100,0 mL dung dịch B. Biết Cu2+ và Histidine có thể tạo ra hai phức bền CuA+ (β1 = 2,0.108) và CuA2 β 2 = 8,0.1018 . β*Cu2+ = 10-8 Tính nồng độ cân bằng của Cu2+ tự do trong dung dịch B. Ý Nội dung Điểm Nồng độ H3A2+ trong dung dịch A là 0,01 M. Các cân bằng: 1 H3A2+ ⇌ H2A+ + H+ Ka1 = 10-1,82(1) + HA + H+ Ka2 = 10-6 (2) H2 A ⇌ Ka3 = 10-9,17(3) A- + H+ HA ⇌ H+ + OHKW = 10-14 (4) H2 O ⇌ So sánh (1) – (4) ta thấy: Ka1 >> Ka2 >> Ka3 và Ka1.C >> KW Do đó pH của dung dịch tính theo cân bằng (1) 0,25 H3A2+ ⇌ H2A+ + H+ Ka1 = 10-1,82 (1) C 0,01 [ ] 0,01 – x x x −3 x = 6,876.10 ; pH = 2,16. 0,25 Tại thời điểm cho hết 40,00 mL dung dịch NaOH 0,01 M vào 40,0 mL dung dịch A. Số mol NaOH bằng số mol H3A2+ 2 Phản ứng chuẩn độ xảy ra như sau: H3A2+ + OH- → H2A+ + H2O Dung dịch thu được có H2A+: 5.10-3 M. 0,25

 H 2 A+  .K a 2 Áp dụng biểu thức điều kiện proton.  H  = 1 +  H 2 A+  .K a−11 +

-5

3

4

pH= 4,21 -4

0,25

pH = 6,0→ a(H3A2+) = 3,30 .10 ; a(H2A+) = 0,50; a(HA) = 0,50; a(A-) = 3,38 .10 . 0,25 2+ Vì a(H3A2+) và a(A-) rất nhỏ, do đó có thể bỏ qua nồng độ của H3A và A-. Hai dạng chính là H2A+ và HA có nồng độ bằng nhau. Trong đó H2A+ mang điện tích + 1 còn HA không mang điện tích. → Điện tích trung bình của trên histidin là + 0,5. 0,25 Nồng độ sau khi trộn: Cu2+: 0,01 M; Histidine: 0,10 M. Cu2+ + A− ⇌ CuA+ β 1 = 2,0.108 2+ − Cu + 2A ⇌ CuA2 β 2 = 8,0.1018 β 2>> β1, Nồng độ ion rất nhỏ so với phối tử → chỉ tạo ra phức CuA2. 0,25 Ở pH = 6, sự tạo phức coi như hoàn toàn →TPGH: CuA2: 0,01 M; Histidin: 0,08 M. C [ ]’

CuA2 0,01 0,01 – x

Cu2+ + − x

2

2A− 0,08 0,08+2x

β2−1 = 1,25.10−19


1

x.

.(0, 08 + 2 x) 2 .α A2 −

0,25 Cu   A  1 + *β  H  −19 = = 1, 25 ×10 β = 0, 01 − x [CuA2 ] x= 1,710.10-12 [Cu2+]= 1,693.10-12M Bài 3. (2 điểm) (Nam Định) Ghép 2 điện cực thành một tế bào Ganvani ở 25oC như sau: Điện cực đồng gồm một thanh Cu nhúng vào dung dịch CuSO4 0,15M; Điện cực kẽm gồm một thanh Zn nhúng vào dung dịch ZnSO4 2,00M. 1. Viết sơ đồ của tế bào Ganvani trên. Viết phương trình phản ứng xảy ra tại các điện cực và phản ứng tổng quát khi tế bào hoạt động. 2.Tính sức điện động của pin tại nhiệt độ 25oC. 3. Khi pin phóng hết điện, nồng độ các chất trong mỗi dung dịch là bao nhiêu? 4. Đặt một hiệu điện thế ngoài vào tế bào để tiến hành nạp điện trong vòng 1 giờ với cường độ dòng điện 1,0 A. Hỏi sức điện động của pin ngay sau khi điện phân là bao nhiêu? Biết thể tích dung dịch ở mỗi điện cực luôn không đổi là 1 lít. Bỏ qua ảnh hưởng của quá thế và điện trở của dung dịch. o o Cho ECu 2+ /Cu = 0,337 V và E Zn 2+ /Zn = -0,760 V 2+

2

+

−1

−1 2

Ý 1 2

Nội dung Sơ đồ tế bào Galvani: Zn | ZnSO4 (2,00M) || CuSO4 (0,15M) | Cu. Tại anot: Zn → Zn2+ + 2e. Tại catot: Cu2+ + 2e → Cu. Phản ứng chung: Zn + Cu2+ → Zn2+ + Cu. Sức điện động của tế bào tại 25oC:

Điểm 0,25 0,25

RT 8,314.298 ln[Zn 2+ ] = -0,760 + ln2,00 = -0,751 (V) 2F 2.96500 RT 8,314.298 o ECu2+ /Cu = ECu ln[Cu 2+ ] = 0,337 + ln 0,15 = 0,313 (V) 2+ /Cu + 2F 2.96500 E Zn2+ /Zn = EoZn2+ /Zn +

Epin = 1,064 (V).

3

Eopin Eopin

=

o ECu 2+ /Cu

0,25

-

EoZn2+ /Zn

= 1,097V RT 8,314.298 = lnK ⇒ 1,097 = lnK ⇒ K = 1037,11. nF 2.96500

K phản ứng rất lớn nên coi như phản ứng xảy ra hoàn toàn: Zn + Cu2+ → Zn2+ Ban đầu: 0,15 2,00 Sau phản ứng 0 2,15 Nồng độ ZnSO4 trong dung dịch là 2,15M. Co: Cpu Csau

Cu +

⇒K =

Zn2+ 2,15 x (2,15 – x)

Cu2+ 0,00 x x

+

+

Cu

[Cu 2+ ] x = = 10 - 37,11 ⇒ x = 1,67.10-37 (M) (hoặc 1,63.10-37) 2+ [Zn ] 2,15 - x

Số mol e trao đổi: n e = ⇒ n Zn2+pu = nCu2+sinhra

0,25

Zn

Nồng độ CuSO4 trong dung dịch là 1,67.10-37 M. (hoặc 1,63.10-37) Khi tiến hành điện phân sẽ diễn ra các quá trình:Zn2+ + Cu → Zn + Cu2+.

4

0,25

0,25

It 1.1.60.60 = = 0,037 mol F 96500 = 0,019 mol

⇒ CZn2+saudp = 1,981 M và CCu2+saudp = 0,169 M

3

0,25


RT 8,314.298 ln[Zn 2+ ] = -0,760 + ln1,981 = -0,751 (V) 2F 2.96500 RT 8,314.298 o ECu2+ /Cu = ECu ln[Cu 2+ ] = 0,337 + ln 0,169 = 0,314 (V) 2+ /Cu + 2F 2.96500 E Zn2+ /Zn = E oZn2+ /Zn +

⇒Epin = 1,065 (V)

0,25

Bài 4. (2 điểm) 1. (Hải Dương) Đơn chất X tác dụng mãnh liệt với dung dịch kiềm, nhưng chỉ tác dụng với nước ở nhiệt độ cao (khoảng 800oC). X bền trong các axit, ngay cả nước cường thủy, chỉ tan trong hỗn hợp hai axit HF và HNO3. Bột mịn X tác dụng được với hơi HF hay tác dụng được với Mg ở khoảng 800oC. X lại có thể được điều chế bằng cách dùng Mg tác dụng với oxit của nó khi đốt cháy hỗn hợp. Hãy cho biết X là đơn chất nào? Viết phương trình hóa học minh họa các phản ứng xảy ra. 2. (Sư Phạm) Hydro mới sinh là một tác nhân khử có hiệu quả nhất. Xử lý một lượng dung dịch natri nitrit bằng hỗn hống natri kim loại cho ra một muối X có 43,40% natri và 26,42% nitơ về khối lượng, còn lại là oxi. Một sản phẩm khác của phản ứng này là natri hydroxit. Để giữ sản phẩm tinh khiết, quá trình tổng hợp được tiến hành trong khí quyển trơ như môi trường nitơ hay argon. a. Xác định công thức muối X. Biết mỗi anion trong X chứa 4 nguyên tử. b. Vẽ công thức Lewis anion của muối X. c. Nếu phản ứng được tiến hành trong không khí thì những tạp chất nào có thể sinh ra? d. Viết phương trình tổng hợp muối X. Ý Nội dung Điểm 0,25 Nguyên tố với các tính chất trên là Si . 1 Si + 2KOH + H2O → K2SiO3 + 2 H2. 800o C 0,25 Si + 2H O  → SiO + 2H . 2

2

2

3Si + 4HNO3 + 18HF → 3H2SF6 + 4NO + 8H2O. Si + 4HFhơi → SiF4 + 2H2.

0,25

o

800 C → Mg2Si. Si + 2Mg 

0,25

o

2

t cao → Si + 2MgO SiO2 + 2Mg  a. Từ %m các nguyên tố → Na : N : O = 1 :1 :1, mà anion X chứa 4 nguyên tử → X là Na2N2O2 – natri hyponitrit. b.

O N O

0,25

O

N

N

N

N

O

O

N

N O

c. Sản phẩm có thể chứa tạp chất NaNO3, NaNO2, Na2CO3, NaHCO3. d. 2NaNO2 + 4Na + 2H2O → Na2N2O2 + 4NaOH

4

O

N O

0,25 0,25 0,25


Bài 5. (2 điểm) (Lào Cai) Axit HIn trong nước phân ly như sau: HIn (màu 1) H+ + In− (màu 2) Đo mật độ quang của dung dịch HIn 5,00.10−4 M trong NaOH 0,1 M và trong HCl 0,1 M ở bước sóng 485 nm và 625 nm với cuvet 1,00 cm. A485 = 0,052 A625 = 0,823 Trong dung dịch NaOH 0,1 M Trong dung dịch HCl 0,1 M A485 = 0,454 A625 = 0,176 − 1. Tính hệ số hấp thụ mol của In và HIn ở bước sóng 485 và 625 nm. 2. Tính hằng số phân ly axit của HIn, nếu trong dung dịch đệm pH 5,00 chứa một lượng nhỏ chất chỉ thị có mật độ quang là 0,472 ở 485 nm và 0,351 ở 625 nm 3. Mật độ quang của một dung dịch HIn 2,00.10−4 M tại 485 và 625 nm (cuvet 1,25 cm) là bao nhiêu nếu dung dịch được đệm ở pH bằng 6,00? 4. Chuẩn độ 25,00 mL dung dịch axit hữu cơ tinh khiết yếu HX với chất chỉ thị là phenolphtalein tới điểm cuối chuẩn độ thì cần 24,20 mL dung dịch NaOH chuẩn. Khi thêm đúng 12,10 mL dung dịch NaOH vào 25,00 mL dung dịch axit HX, trong dung dịch có một lượng nhỏ chất chỉ thị HIn, mật độ quang đo được là 0,306 ở 485 nm và 0,555 ở 625 nm (cuvet 1,00 cm). Tính pKa cho HX. Ý 1

Nội dung − Trong dung dịch NaOH: chất chỉ thị chỉ tồn tại chủ yếu ở dạng In− In In A485 = ε 485 .C = 0,052 → ε 485 = 104 In In A625 = ε 625 .C = 0,823 → ε 625 = 1646 − Trong dung dịch HCl: chất chỉ thị chỉ tồn tại chủ yếu ở dạng HIn. HIn HIn A485 = ε 485 .C = 0,454 → ε 485 = 908

2

3

4

HIn HIn A625 = ε 625 .C = 0,176 → ε 625 = 352 Ở pH = 5. HIn In A485 = ε 485 .[HIn] + ε 485 .[In−] = 908. [HIn] + 104 .[In−] = 0,472 HIn In A625 = ε 625 .[HIn] + ε 625 .[In−] = 352. [HIn] +1645 .[In−] = 0,351 [In−] = 1,047.10−4. Giải hệ phương trình tìm được: [HIn] = 5,078.10−4 1, 047.10 −4.10 −5 → Ka = = 2,06. 10−6 = 10-5,686 5, 078.10 −4 Ở pH = 6,000 10−6 [HIn] = 2.10−4 . −6 = 6,53 . 10−5 (M) −5,686 10 + 10 10−5,686 [In−] = 2.10−4 . −6 = 1,347 . 10−4 (M) 10 + 10−5,686 HIn In .[HIn] + ε 485 .[In−] = 908. 6,53 . 10−5 + 104 . 1,347 . 10−4 = 0,0733 A485 = ε 485

Điểm

0,25 0,25

0,25 0,25

0,25

HIn In 0,25 A625 = ε 625 .[HIn] + ε 625 .[In−] = 352. 6,53 . 10−5 +1645. 1,347 . 10−4 = 0,245 Chuẩn độ tới đổi màu phenolphtalein cần 24,20 mL dung dịch bazơ → khi thêm 12,10 mL thì dung dịch chứa HX và X− với hai lượng tương đương. [HX ] → Áp dụng công thức có thể suy ra pKa: pKa = pH + lg − = pH. 0,25 [X ] HIn In A485 = ε 485 .[HIn] + ε 485 .[In−] = 908. [HIn] + 104 .[In−] = 0,306 HIn In A625 = ε 625 .[HIn] + ε 625 .[In−] = 352. [HIn] +1645 .[In−] = 0,555 Giải hệ phương trình tìm được: [HIn] = 3,06.10−4 [In−] = 2,72.10−4. 10−5,686.3, 06.10 −4 → [H+] = = 2,3175. 10−6 → pH = 5,635 → pKa (HX) = 5,635. −4 2, 72.10

5

0,25


Bài 6. (2 điểm) 1. (Đà Nẵng) Hằng số tốc độ k của phản ứng xà phòng hóa bằng NaOH trong đioxan và nước các dẫn xuất thế m- và p-metylbenzoat được liệt kê dưới đây. X-C6H4-COOCH3 + NaOH → X-C6H4-COONa + CH3OH k(M-1.phút-1) k(M-1.phút-1) (1) Metyl p-nitrobenzoat 102 (4) Metyl m-brombenzoat ? (2) Metyl m-nitrobenzoat 63 (5) Metyl benzoat 1,7 (3) Metyl m-clobenzoat ? (6) Metyl p-aminobenzoat 0,06 a. So sánh giá trị k của (3), (4), (5). b. Dựa trên cấu trúc (gồm cả các trạng thái cộng hưởng có thể có) giải thích vì sao chất (3) có tốc độ thủy phân cao hơn (4). Còn tốc độ thủy phân của chất (6) rất nhỏ. 2. (Bắc Giang) Axit tactric (axit 2,3A B C D đihidroxibutandioic) được điều chế theo 4 thí [αD, 25oC] -12 +12 0 0 nghiệm thu được 4 dạng A, B, C, D. Tính chất của tnc 171 171 205 140 mỗi dạng cho ở bảng sau: Độ tan 100g H2O 139 139 21 125 Xác định cấu trúc của C, D. 3. (Thái Bình) Vì sao ở điều kiện thường, mỡ động vật thường là chất rắn, còn dầu thực vật thường là chất lỏng. Ý 1

Nội dung a. Các nhóm thế hút electron hoạt nhóm cacbonyl, làm thuận lợi cho phản ứng thế SN2(CO) do đó nhóm thế m-Cl gây –I mạnh hơn m-Br (do độ âm điện Cl lớn hơn), nên tốc độ của dẫn xuất thế Cl- lớn hơn dẫn xuất brom và lớn hơn chất gốc không có nhóm thế.

Điểm 0,25 0,25

b. Nhóm thế p-O2N đặt điện tích dương lên C cạnh nhóm cacbonyl (cộng hưởng A) nên hoạt hóa nhóm này mạnh hơn nhóm thế m-O2N (xem cộng hưởng B).

0,25 Ngược lại, nhóm p-H2N (gây +C). Vì N lại dễ nhường cặp electron hơn O (độ âm điện nhỏ hơn), nên p-H2N phản hoạt hóa nhóm cacbonyl nhiều hơn.

0,25

2 3.

A, B chỉ khác nhau độ quay cực riêng → A, B là cặp đối quang. C, D không quang hoạt → là đồng phân meso hoặc hỗn hợp raxemic. Vì C có nhiệt độ nóng chảy và độ tan nhỏ hơn → cấu trúc tinh thể chặt chẽ hơn → C là biến thể raxemic Mỡ động vật thường chỉ chứa gốc no → cấu trúc gốc luôn ở dạng ziczac nên tổng thể gốc có cấu trúc thẳng, sắp xếp gọn → Các phân tử dễ sắp xếp chặt chẽ. Dầu thực vật chứa gốc không no, cấu trúc gốc bị uốn gập do luôn ở dạng cis → Các

6

0,25 0,25 0,25


phân tử không thể sắp sếp gọn.

0,25

Bài 7. (2 điểm) 1. (Bắc Ninh) Hidrocacbon A quang hoạt có công thức phân tử C8H12. Khi thực hiện phản ứng hidro hóa A bằng xúc tác Pt, đun nóng thu được B[C8H18] không quang hoạt, nhưng nếu dùng xúc tác Lindlar lại thu được C [C8H14] quang hoạt. Cho A phản ứng Na trong NH3 lỏng thu được hợp chất D [C8H14] không quang hoạt. Xác định cấu trúc các chất A, B, C, D. 2. (Bắc Giang)Từ m-xilen, 3-oxobutanal và các hóa chất (cronen: C 24H12) phụ khác, hãy viết sơ đồ điều chế cronen: Ý

Nội dung

Điểm

1 0,25 x4

2

NBS (1:1) H3C CH3

H3C

Mg/ete Br

O CHO

MgBr

H3C O

HO

O

OH

O

O

H2O H3C

OH

O O

OH

H3O + H3C

+

H

Zn Cl

H2SO4 H3C

H3C

OH

NCS

OH

CH3

0,5

o

Pd, t C

Cl

(cronen: C24H12)

7

0,5


Bài 8. (2 điểm) (Hùng Vương) Hiđro hóa hoàn toàn naphtalen người ta thu được đecalin (C10H18). Oxi hóa đecalin thì thu được hỗn hợp các decalon (C10H16O). 1. Trong hỗn hợp decalon nói trên có tối đa bao nhiêu đồng phân ? Giải thích. 2. Hãy vẽ công thức các đồng phân lập thể của decal-1-on, biết rằng hai vòng 6 cạnh trong phân tử decalon đều ở dạng ghế. 3. Hòa tan X (một trong số các đồng phân decalon) vào dung dịch bazơ thì X bị đồng phân hóa thành Y tới 95%. Lập luận để xác định X, Y. 4. Trong dung dịch bazơ, decal-1-on phản ứng với benzanđehit cho hợp chất T, với metyl vinyl xeton cho hợp chất U. T và U đều làm mất màu nước brom. Hãy viết công thức cấu tạo của T và U. Ý Nội dung Điểm Trong hỗn hợp có 8 decalon đồng phân gồm: 1 trans-1-decalon: 2 đối quang; cis-1-decalon: 2 cặp đối quang 0,5 trans-2-decalon: 2 đối quang; cis-2-decalon: 2 cặp đối quang. (hoặc giải thích do có 2C* khác nhau) 4 đồng phân lập thể của 1-decalon: 2 0,25 O O x2 O O

O

O

3 Ở đồng phân decan-1-on, nguyên tử Cα trở thành cacbanion, cacbanion có thể đảo cấu hình rồi kết hợp với H+. Nếu Cα đó là C* thì sẽ xảy ra đồng phân hóa. Cân bằng đồng phân hóa sẽ nghiêng về phía đồng phân trans bền hơn: 0,25

H

H

H

- H+ + H+

- H+ O

O

O

H

+ H+

X 4 T là sản phẩm ngưng tụ, U là sản phẩm cộng Michael rồi ngưng tụ tiếp theo: O O T:

CHC6H5

U:

Bài 9. (2 điểm) Viết cơ chế các phản ứng sau

8

H Y

0,25 0,25 x2


Ý

Nội dung

1 Br(H 2C)3 O N

O

(CH 2)3Br O-

O

Điểm Br O-

O N

N

-Br-

O

O

0,5

N

2

0,5

3

0,5

9


4

0,5

Bài 10. (2 điểm) (Vĩnh Phúc) Loline là một thành viên của họ 1- aminopyrrolizidines (thường được gọi là lolines), là một ankaloid . Các lolines là những hợp chất diệt côn trùng, ngăn chặn sự sinh sản của cỏ nấm cộng sinh trong cỏ endophytic. Loline được tổng hợp theo sơ đồ sau:

OMe OMe

COOMe

HOH2C

MsCl

C

D

MeO OMe COOMe H2/Pd

N

OH

N

O

1. LiAlH4 SOCl2 H+ H /Pt E G 2 H 2. Ac2O Py

Cl

CH2OAc MeONa OAc

N

K C8H13NO2

NHMe CrO3,H2SO4

1. NH3

L

EtOH

2. Br2/NaOH

CH3I

M

O

Bazo

N

Hãy viết công thức cấu tạo của các chất C, D, E, G, H, K, L, M. Đáp án: MeO COOMe OMe

N

C O

MeO

CH 2OH D

O CH2OAc

HO

OMe

N

MeO

COOMe

CH2OMs

O

OAc

E

N

CH2OH

CH2OAc

COOEt

O OAc G

N

H

OAc N

K

10

OMe CH2OAc

NH2

O N L

O N

M

N


Mỗi chất xác định đúng cho 0,25 đ -------------- HẾT --------------

11


HỘI CÁC TRƯỜNG CHUYÊN VÙNG DUYÊN HẢI VÀ ĐỒNG BẰNG BẮC BỘ

ĐỀ THI MÔN HÓA HỌC KHỐI 11 NĂM 2017

TRƯỜNG PT VÙNG CAO VIỆT BẮC ĐỀ THI ĐÈ XUẤT

Thời gian làm bài 180 phút (Đề này có 4 trang, gồm 10 câu)

Câu 1: (2 điểm) Tốc độ phản ứng- cân bằng hóa học Xét sự thủy phân của este metyl axetat ở 250C trong các môi trường sau: Trường hợp 1: Trong môi trường kiềm, khi tăng nồng độ kiềm lên gấp đôi thì tốc độ phản ứng tăng lên hai lần. Nhận xét này cũng được thấy khi tăng nồng độ của este lên hai lần. Trường hợp 2: Phản ứng thuỷ phân được thực hiện trong môi trường đệm Trường hợp 3: Người ta tiến hành phản ứng thuỷ phân trong môi trường axit HCl 0,05M dư. Động học của phản ứng được nghiên cứu bằng cách chuẩn độ 25ml hỗn hợp phản ứng bằng dung dịch NaOH ở từng thời điểm t với kết quả như sau : t [phút]

0

21

75

119

VNaOH [cm3]

24,4

25,8

29,3

31,7

47,2

a. Hãy viết phương trình động học của phản ứng. Cho biết bậc của phản ứng trong từng trường hợp. b. Trong TH1: Nếu cho 0,01 mol xút và 0,01 mol este vào 1lit nước (xem như thể tích thay đổi không đáng kể). Sau 200 phút thì 3/5 lượng este chưa bị phân huỷ. Tính hằng số tốc độ phản ứng k1 c. Trong TH 3: Hãy tính hằng số tốc độ phản ứng k3 và thời gian để este phân huỷ hết 50%. Từ đó hãy so sánh giá trị k1 và k3 Câu 2: (2 điểm) Cân bằng trong dung dịch điện li 1. Tính pH của dung dịch H3PO4 0,1M 2. Cần cho vào 100ml dung dịch H3PO4 0,1M bao nhiêu gam NaOH để thu được dung dịch có pH= 4,72.


3. Trộn 20,00 ml dung dịch H3PO4 0,50 M với 37,50 ml dung dịch Na3PO4 0,40 M, rồi pha loãng bằng nước cất thành 100,00 ml dung dịch A. Cần phải thêm bao nhiêu ml dung dịch HCl 0,050 M vào 20,00 ml dung dịch A để thu được dung dịch có pH =5,00 (metyl đỏ đổi màu). Cho: H2SO4 : pKa2 = 2 ; H3PO4 : pKa1 = 2,23 , pKa2 = 7,21 , pKa3 = 12,32 Câu 3: (2 điểm) Điện hoá học Xét khả năng phản ứng của Cl-, Br- với KMnO4.Biết E Br0

2

ECl0

2

/ 2 Cl −

/ 2 Br −

= 1,085V ;

0 = 1,359V ; E MnO = 1,51V . − / Mn 2 + 4

a. Ở pH=0 b. Trong dung dịch axit axetic 1,00 M. Biết CH3COOH có Ka=10-4,76. Câu 4: (2 điểm) Bài tập tính toán vô cơ tổng hợp Kim lo¹i A ph¶n øng víi phi kim B t¹o hîp chÊt C mµu vµng cam. Cho 0,1 mol hîp chÊt C ph¶n øng víi CO2 (d−) t¹o thµnh hîp chÊt D vµ 2,4 gam B. Hßa tan hoµn toµn D vµo n−íc, dung dÞch D ph¶n øng hÕt 100 ml dung dÞch HCl 1 M gi¶i phãng 1,12 l khÝ CO2 (®ktc). H·y x¸c ®Þnh A, B, C, D vµ viÕt c¸c ph−¬ng tr×nh ph¶n øng x¶y ra. BiÕt hîp chÊt C chøa 45,07 % B theo khèi l−îng; hîp chÊt D kh«ng bÞ ph©n tÝch khi nãng ch¶y. Câu 5: (2,0điểm) Phức chất. Cấu hình electron của nguyên tố M ở trạng thái cơ bản chỉ ra rằng: M có 4 lớp electron, số electron độc thân của M là 3. a. Dựa vào các dữ liệu trên cho biết M có thể là các nguyên tố nào. b. M tạo được ion phức có công thức [M(NH3)6]3+, phép đo momen từ chỉ ra rằng ion này là nghịch từ. - Cho biết tên gọi của [M(NH3)6]Cl3 - Cho biết trạng thái lai hoá của M trong ion phức trên và chỉ ra dạng hình học của ion phức này. Câu 6: ( 2 điểm) Quan hệ giữa cấu trúc và tính chất 1. Sắp xếp (có giải thích) theo trình tự tăng dần tính axit của các chất trong từng dãy sau: a. Axit: benzoic, phenyletanoic, 3-phenylpropanoic, xiclohexyletanoic, 1-metylxiclohexan-cacboxylic.


COOH

b.

N

;

COOH

;

; N

(A)

CH2COOH

COOH

(D)

(C)

(B)

2. Sắp xếp (có giải thích) theo trình tự tăng dần nhiệt độ nóng chảy của các chất sau: COOH

S

(A)

COOH

COOH ;

; N

(B)

(C)

Câu 7: ( 2 điểm) Hidrocacbon Trộn hiđrocacbon khí A với oxi theo tỉ lệ thể tích A:O2 = 1:9 rồi cho vào bình kín thấy áp suất trong bình là 1 atm ở 00C. Bật tia lửa điện để A cháy hết, hh sau pư có áp suất là 1,575 atm ở 136,50C. a.Tìm công thức phân tử của A? b. Viết công thức cấu tạo có thể có của A biết tất cả các nguyên tử cacbon trong A đều có cùng một dạng lai hóa? c. Chọn công thức cấu tạo của A ở trên để viết sơ đồ tổng hợp :

+ Chất B(Anthracen) có CTCT như sau: + bixiclo[ 4.1.0] heptan + Đietyl phtalat( DEF) Câu 8: (2,0 điểm) Xác định cấu trúc, đồng phân lập thể, danh pháp Hợp chất A đóng vai trò là chất gốc để tạo thành các hợp chất E1 và E2


(Gợi ý: Các hợp chất E1 và E2 là trans-diol có tỉ lệ 1: 1). 1. Xác định cấu trúc của A, D, E1 và E2 trong sơ đồ phản ứng trên. 2. Viết tên các hợp chất A, B, C, D, E1 và E2. 3. Thêm các chất phản ứng cần thiết bổ sung ở những nơi có dấu "?". 4. Đánh dấu tất cả các trung tâm bất đối của các hợp chất A, B, C, D, E1 và E2. Câu 9: (2,0 điểm) Cơ chế phản ứng 2,7-đimetylnaphtalen có thể được tổng hợp bằng phản ứng giữa tác nhân Grignard A và axetal B theo sơ đồ sau: i MgBr

A CHO

O

ii

O

O

+A/H+

O

B

a. Đề nghị điều kiện tạo thành A và B. Viết phương trình phản ứng b. Trình bày cơ chế phản ứng tạo thành 2,7-đimetylnaphtalen Câu 10. (2 điểm): Tổng hợp các chất hữu cơ Xác định các chất A, B, C, D,E trong sơ đồ phản ứng sau: COOH OH

AlCl3

(CH3CO)2O A

N H

Br2 (CHCl3) B

C

D LiAlH4 H2/Pd-C

Salbutamol

E


ĐÁP ÁN VÀ THANG ĐIỂM

Nội dung

Câu

Điểm

Câu 1 a.

2,0 Nhận xét: TH1: v = k1[este]a.[OH-]b → khi tăng nồng độ của este hoặc bazơ lên gấp hai lần thì v cũng tăng lên gấp đôi → a = b = 1 → v = k1[este][OH-] Vậy trong TH1 bậc của phản ứng là bậc 2 TH2: Ta có v = k[este][OH-] . Nhưng trong môi trường đệm nên [OH-] =

0,5

const → v = k2[este] với k2 = k[OH-] TH3: v = k[este][axit] Do axit lấy dư nên k[axit] = const =k3 → v = k3[este] Vậy trong TH2, 3 bậc phản ứng là bậc 1

b.

Vì nồng este và axit bằng nhau và [A0] = 0,01M nên ta có 1 1 − = kt với [A0] là nồng độ của este; [A0] – x là nồng độ của [ A0 ] − x [ A0 ]

este tại thời điểm t Lượng este chưa bị thuỷ phân là 2/5[A0] →

1 2 [ A0 ] 5

1 = kt [ A0 ]

0,75

→ k= 0,75 mol-1.L.phút-1 = k1

c.

Ta có k =

[ A0 ] 2,303 lg t [ A0 ] − x

Nếu V∞ ở thời điểm t = ∞ là thể tích ứng với sự kết thúc thuỷ phân este trong môi trường axit, V0 là thể tích ứng với thời điểm t= 0 thì hiệu V∞ - V0 sẽ tỉ lệ với nồng độ đầu của este. Còn hiệu V∞ - Vt sẽ tỉ lệ với nồng độ este tại thời

0,75


điểm t ( Vt là thể tích ứng với thời điểm t ). Do đó: t [phút]

21

75

119

VNaOH [cm3] 25,8

29,3

31,7

k3 phút-1

0,003224

0,003244

0,003016

k 3 = 0,003161 phut −1 Vậy k1 gấp k 3 khoảng 237,27 lần.

t k 3 = ln2 → t = 219 phút Câu 2 1

2,0 H3PO4

H+ + H2PO4- (1) K1 = 10-2,23 H2PO4-

H+ + HPO42- (2) K2 = 10-7,21

HPO42-

H+ + PO43-

(3) K3 = 10-12,32

H2 O

H+ + OH-

(4) Kw

K3 << K2 << K1 ⇒ chủ yếu xảy ra cân bằng (1) H+ + H2PO4-

H3PO4

K1 = 10-2,23

0,5

C(M) 0,1 [ ](M) 0,1 – x

x

x

x2 = 10-2,23 ⇒ x2 + 10-2,23 x – 10-3,23 = 0 (0,1 - x) ⇒

x = 0,0215 (M)

NaOH + H3PO4

⇒ pH = 1,66

= NaH2PO4 + H2O

NaOH + NaH2PO4 = Na2HPO4 + H2O NaOH + Na2HPO4 = Na3PO4 + H2O 2

Trung hòa nấc 1: pH1 =

pK1 + pK2 2.23 + 7.21 = 4,72 = 2 2

⇒ trong dung dịch thu được có pH = 4,72 chỉ chứa NaH2PO4: nH3PO4 = 0,1 x 0,1 = 0,01 (mol) ⇒ nNaOH = 0,01 (mol) mNaOH = 0,01 x 40 = 0,4(g)

0,5


C H3PO4 =

0,50.20 0,40.37,5 = 0,10 (M); C Na 3PO4 = = 0,15 (M). 100 100

C Na3PO4 = 1,5. CH3PO4 → phản ứng xảy ra như sau:

3

H3PO4 + PO34 0,1

0,15

0

0,05

= HPO 2+ 4

H 2 PO-4

10,17 K1 = Ka1. K -1 a3 = 10

1,0 0,1

H 2 PO-4 + PO34

0,1 5,11 K2 = Ka2. K -1 a3 = 10

= 2 HPO 24

0,1

0,05

0,1

0,05

0

0,2

Dung dịch A thu được là hệ đệm gồm: H 2 PO-4 0,05 M và HPO 24 0,2 M pH = 5,00 ≈ pH (NaH PO ) ≈ 2

4

pK a1 + pK a2 = 4,68 → có thể coi lượng HCl thêm 2

vào 20,00 ml dung dịch A sẽ phản ứng vừa đủ với HPO 24 tạo thành H 2 PO 4 :

+ HPO 2→ H 2 PO-4 4 + H

→ VHCl =

0,2.20 = 80 (ml) 0,05

Câu 3 a

2,0 Các cặp oxi hoá- khử: 2Br-

Br2 + 2e

0 E Br

2

/ 2 Br −

ECl0

2

/ 2 Cl −

= 1,085V

(1) 2Cl-

Cl2 + 2e

= 1,359V

(2) MnO4- + 8H+ + 5e

Mn2+ + 4H2O

0 E MnO = 1,51V − / Mn 2 + 4

(3) Từ (1) và (2) ta thấy thế của các cặp không phụ thuộc vào pH( trong môi trường axit), tuy vậy thế của cặp MnO4-/Mn2+ lại phụ thuộc pH: E = E0 +

[ [

][ ] ]

MnO4− 8 0,0592 . lg .H+ → 2+ 5 Mn

0,5


E = E0 +

] ] [MnO ] − 0,095 pH + 0,0118 lg [Mn ]

0,0592 . lg H + 5

8

[ ]

E=E

0

+

[ [

MnO4− 0,0592 . lg 5 Mn 2+

− 4 2+

(4) Ở pH = 0

0,5

Ở điều kiện tiêu chuẩn E = E = 1,51V > E 0

0 Cl2 / 2 Cl −

>E

0 Br2 / 2 Br −

. Vì vậy trước hết

MnO4- oxi hoá Br- thành Br2 và sau đó Cl- thành Cl2.

b

Trong dung dịch CH3COOH 1,00M H+ + CH3COO-

CH3COOH C

1,00

[]

1,00-x

x

[H+] = x =

Ka = 10-4,76

x 1,0

K a = 10-2,38 → pH = 2,38

Từ (4) rút ra: E = E0- 0,095pH = 1,51- 0,095.2,38 = 1,28V Bởi vì E Br0

2

/ 2 Br −

< E < E Cl0

2

/ 2 Cl −

, nên trong dung dịch CH3COOH 1M, MnO4- chỉ

oxi hoá được Br- thành Br2 mà không oxi hoá được Cl- thành Cl2. Câu 4

2,0 nHCl = 0,1 mol ; nCO2 = 0,05 mol Dung dÞch D ph¶n øng hÕt 0,1 mol HCl gi¶i phãng khÝ CO2 →

nH + nCO2

=

0, 1 = 0, 05

2 1

suy ra h¬p chÊt D lµ muèi cacbonat kim lo¹i. h¬p chÊt D kh«ng bÞ ph©n tÝch khi nãng ch¶y, vËy D lµ cacbonat kim lo¹i kiÒm.

2 H+ + CO32- = H2O + CO2


C + CO2 = D + B → C lµ peroxit hay superoxit, B lµ oxi. §Æt c«ng thøc ho¸ häc cña C lµ AxOy . L−îng oxi trong 0,1 mol C (AxOy ) lµ 16 x 0,05 + 2,4 = 3,2 (g); mC =

3, 2.100 45, 07

= 7,1 gam Mc = 7,1 : 0,1 = 71 (g/mol). mA trong C = 7,1 - 3,2 = 3,9 (g). x:y=

3, 9 3, 2 → MA = 39 (g). VËy A lµ K ; B lµ O2 ; C lµ KO2 ; D lµ : M A 16

K2CO3 C¸c ph−¬ng tr×nh ph¶n øng:

K + O2 → KO2 4 KO2 + 2 CO2 → 2 K2CO3 + 3O2 ↑ K2CO3 + 2 HCl → 2 KCl + H2O + CO2 ↑

Câu 5 1

2,0 Vì có 4 lớp điện tử do vậy phân lớp cuối cùng trong phân bố điện tử chỉ có thể là 4S, 3d, 4P. Vì có 3 điện tử độc thân do vậy, phân lớp cuối cùng chỉ có thể là 3

2

2

6

2

6

3

1,0

2

3d → Cấu hình hoàn chỉnh 1S 2S 2P 3S 3P 3d 4S Nguyên tố 23V 3d7 → Cấu hình hoàn chỉnh 1S2 2S2 2P6 3S2 3P6 3d7 4S2 Nguyên tố 27Co 3P3 → Cấu hình hoàn chỉnh 1S2 2S2 2P6 3S2 3P6 3d10 4S2 4P3 Nguyên tố 33As 2 Tạo phức với NH3 có công thức [M(NH3)6]3+ do vậy không thể là As. Vì phức nghịch từ do vậy không có điện tử độc thân → M chỉ có thể là Coban [CO] Tên gọi [CO(NH3)6]Cl3

:

Hexa amin coban (III) Clorua:

CO3+: 1S2 2S2 2P6 3S2 3P6 3d6 Vì NH3 là phối tử trường mạnh do vậy khi tạo phức có sự dồn 2 electron vào vậy:

1,0


NH3 NH3 NH3 NH3NH3NH3

Vậy Co lai hoá d2sp3 Hình dạng phân tử bát diện: NH3 NH3

H3N Co NH3

H3N NH3

Câu 6

2,0 a +I2

H3C COOH

CH2COOH

+I1

<

<

<

+I1

CH2COOH

-I1CH2CH2COOH

-I2

-I1

0,75

<

<

+I2

COOH

-I3

<

-I2

<

-I3

Các gốc hiđrocacbon có hiệu ứng +I lớn thì Ka giảm và -I lớn thì Ka tăng

b

-I1

CH2COOH

COOH -I2

< (D) Vì:

< (C)

C O H O (A)

-C3 N -I3

COOH -I4 < N -C4 (B)

- I1 < - I2 nên (C) có tính axit lớn hơn (D). (A) và (B) có N nên tính axit lớn hơn (D) và (C) (A) có liên kết hiđro nội phân tử làm giảm tính axit so với (B).

0,75


2

Tăng dần nhiệt độ nóng chảy của các chất:

COOH

COOH <

<

S (A)

(C)

0,5

COOH

N (B)

Vì: M C < MA. (B) có thêm liên kết hiđro liên phân tử với N của phân tử khác.

Câu 7

2,0 Gọi CxHy là công thức của A ta có: y 4

y H2 O 2

CxHy +(x+ ) O2 → xCO2 + Mol:

a

y 4

a(x+ )

ax

a

y 2

+ Số mol khí trước pư = a + 9a = 10a mol 1

+ Số mol khí sau pư = ax + a V bình kín =

0,5

y y y + 9a – a(x+ ) = 9a + a 2 4 4

10a.273 (9a + 0, 25ay ).409,5 n1RT1 n 2 RT2 hay: = y = 6. = 1 1,575 P1 P2

+ Vì A là chất khí nên A có thể là: C2H6; C3H6 và C4H6 0,5 đ

Vì tất cả các nguyên tử C trong A đều có cùng một dạng lai hóa nên CTCT của A: 2

CH3-CH3(sp3); (CH2)3(xiclopropan = sp3); CH2=CH-CH=CH2(sp2) và CH

CH2

0,5

(sp3)

CH2 CH

Ta chọn A là buta-1,3-đien để hoàn thành sơ đồ: buta-1,3-đien → butan → etilen. 3 HC HC

C H2 C H2

+ C H2 C H2

Cho sp cuối cùng trong sơ đồ pư với H2/Ni, t0 thì thu được B

+

1,0


CH2N2 Cu,t0

CH2=CHCH=CH2

0,5 đ

Br + CH2=CHCH=CH2

NBS Br -HBr

+ C2H5OH

C6H4(CO)2O

SP

-H2

+O2

Câu 8 OH

*

* *

A

D

*

*

OH

*

S

R R

O

1,4

OH

OH

S

*

OH

E1

*

OH

E2

OH

C

1,0

A: xiclohexanol; B: xiclohexen; C: cis-xiclohexan-1,2-diol; D: xiclohexanepoxit E1: (1R, 2R)-xiclohexan-1,2-diol; E2: (1S, 2S)-xiclohexan-1,2-diol 2,3

B B

KMnO4 lo·ng, l¹nh mCPBA

D

1,0

C +

H3O

E1 + E2

axit meta-Chloroperoxybenzoic (mCPBA) là một axit peroxycarboxylic sử dụng rộng rãi như một chất oxy hóa trong tổng hợp hữu cơ.


Câu 9

2,0

Mg/THF

NBS

MgBr Br

A

HO

a. CHO

H+

O

1,0

O

HO O

O

B

O O

O

A +

HC

H+/H2O B

b.

OH

OH HC+

H

OH

OMgBr

SE

+

OH

OH

1,0 -2H2O


Câu 10

2,0 A: HO

B:

O

HO

O

O

OH O

O C:

D:

O

HO

HO

OH

OH

N

Br

O

O HO

OH E:

N OH

O


TRƯỜNG THPT CHUYÊN LÊ QUÝ ĐÔN ĐIỆN BIÊN

ĐỀ THI MÔN HOÁ HỌC LỚP 11 Thời gian làm bài : 180 phút;

ĐỀ THI ĐỀ XUẤT ĐỀ BÀI Câu 1 (2 điểm): Động học

Để phân hủy hiđro peoxit (H2O2) với chất xúc tác là ion iođua trong dung dịch có môi trường trung tính, người ta trộn dung dịch H2O2 3% (chấp nhận tương đương với 30 gam H2O2 trong 1 lít dung dịch) và dung dịch KI 0,1 M với nước theo tỉ lệ khác nhau về thể tích để tiến hành thí nghiệm xác định thể tích oxi ( VO ) thoát ra. Thí VKI (ml) VH O (ml) VH O (ml) υ O (ml/phút) nghiệm ở 298 K và 1 2

2

2

2

2

atm 1 25 50 75 4,4 2 50 50 50 8,5 3 100 50 0 17,5 4 50 25 75 4,25 5 50 100 0 16,5 1. Xác định bậc phản ứng phân huỷ đối với H2O2 và đối với chất xúc tác I-. 2. Viết phương trình hoá học và biểu thức tính tốc độ phản ứng. 3. Tính nồng độ mol của H2O2 khi bắt đầu thí nghiệm 4 và sau 4 phút. 4. Cơ chế phản ứng được xem là một chuỗi hai phản ứng sau: H2 O2 + I -

k

1 → H O + IO  2

-

(1)

k2

IO- + H2O2 → O2 + I- + H2O (2) Hãy cho biết hai phản ứng này xảy ra với tốc độ như nhau hay khác nhau? Phản ứng nào quyết định tốc độ phản ứng giải phóng oxi? Giải thích. Câu 2 (2 điểm): Cân bằng trong dung dịch điện li Trộn 10,00 mL dung dịch CH3COOH 0,20 M với 10,00 mL dung dịch H3PO4, thu được dung dịch A có pH = 1,50. 1. Tính CH3PO4 trong dung dịch H3PO4 trước khi trộn. 2. Tính độ điện li của CH3COOH trong dung dịch A. 3. Thêm từ từ Na2CO3 rắn vào dung dịch A cho đến pH = 4,0, thu được dung dịch B. Tính số gam Na2CO3 đã dùng. Cho biết: H3PO4: pKa1 = 2,15; pKa2 = 7,21; pKa3 = 12,32; CH3COOH: pKa = 4,76; CO2 + H2O có pKa1 = 6,35; pKa2 = 10,33;

1


Câu 3 (2 điểm): Điện hóa học Cho sức điện động của pin: AgCl Ag Ag AgNO3 0,001M Na2S2O3 0,10M HCl 0,05M 1. Viết phương trình phản ứng khi pin hoạt động .

Là 0,341V.

0

2. Tính E Ag ( S 2O3 )32− / Ag 3. Tính KsAgCl . 4. Thêm 0,01 mol KCN vào 1 lít dung dịch ở anot .Tính Epin 0 Cho: E Ag =0,80V , Ag+ + 2S2O32Ag(S2O3)23- lgβ1 =13,46 / Ag +

Ag+ + 2CN-

Ag(CN)2- lgβ2 = 21

Câu 4 (2 điểm): N –P, C- Si và hợp chất 1. Cho 3 gam kim loại M có hoá trị 2 tác dụng với dung dịch HNO3 đặc, nóng, dư. Khí tạo thành cho hấp thụ hoàn toàn vào dung dịch NaOH vừa đủ , thu được dung dịch X. Cô cạn dung dịch X rồi nhiệt phân hoàn toàn chất rắn thu được trong môi trường trơ thì được chất Y. Lượng chất Y này làm mất màu vừa hết 0,5 lit dung dịch KMnO4 0,2M trong môi trường H2SO4. a) Xác định kim loại M. b) Viết các phương trình phản ứng. 2. Trong điều kiện thường, NF3 là chất khí không màu, hóa lỏng ở -129 độ C, hóa rắn ở -2090C. Còn NH3 cũng là chất khí nhưng hóa lỏng ở -33,350C, hóa rắn ở -77,750C. Amoniac dễ tham gia phản ứng cộng, có tính khử, còn NF3 thì không có những tính chất này. a) So sánh góc HNH và góc FNF, giải thích ? b) Dựa vào cấu trúc phân tử, hãy giải thích sự khác nhau về các hiện tượng thực nghiệm trên. Câu 5 (2 điểm): Phức chất Hòa tan 1,066 gam tinh thể hiđrat A của một kim loại M tan trong nước được dung dịch màu xanh. Cho dung dịch này tác dụng với dung dịch AgNO3 dư, thu được ngay 1,148 gam kết tủa trắng X và dung dịch B. Kết tủa X là chất dễ bị phân hủy. Đun nóng B với H2O2 trong môi trường kiềm thu được dung dịch C. Cho C tác dụng với dung dịch BaCl2 dư thu được 1,013 gam kết tủa Y màu vàng; Y đồng hình với BaSO4. Dung dịch của A nếu pha loãng hoặc đun nóng đều có màu sắc thay đổi. Khi theo dõi quá trình phân hủy nhiệt của A trong môi trường HCl thấy 2


trước khi A bị phân hủy ở khoảng nhiệt độ <1800C thì khối lượng giảm 6,75%. Nếu ở 3000C thì khối lượng giảm 40,52%. Hãy xác định các công thức của A, B, C, X, Y và viết các phương trình hóa học xảy ra trong các thí nghiệm trên. Câu 6 (2,0 điểm): Quan hệ giữa hiệu ứng cấu trúc và tính chất 1. Vẽ cấu trúc của 1,3-diazole (imidazole, C3H4N2), anion imidazol-1-yl, cation imidazolyl, 1,3-oxazole (oxazole, C3H3NO) and 1,3-thiazole (thiazole, C3H3NS). Cấu trúc nào thơm? 2. Sắp xếp imidazole, 1,3-oxazole and 1,3-thiazole theo chiều giảm dần nhiệt độ sôi, nhiệt độ nóng chảy, giải thích? 3. Sử dụng cấu trúc viết quá trình ion hóa imidazole, oxazole, và thiazole trong nước. Sắp xếp theo chiều giảm dần tính bazơ và giải thích? Câu 7 (2,0 điểm): Hidrocacbon 1. Hợp chất 2,2,4-trimetylpentan (A) được sản xuất với quy mô lớn bằng phương pháp tổng hợp xúc tác từ C4H8 (X) với C4H10 (Y). A cũng có thể được điều chế từ X theo hai bước: thứ nhất, khi có xúc tác axit vô cơ, X tạo thành Z và Q; thứ hai, hiđro hoá Q và Z. a) Viết các phương trình phản ứng để minh họa và tên các hợp chất X, Y, Z, Q theo danh pháp IUPAC. b) Ozon phân Z và Q sẽ tạo thành 4 hợp chất, trong đó có axeton và fomanđehit, viết cơ chế phản ứng. 2. Hợp chất A có công thức phân tử C9H16. Khi cho A tác dụng với hiđro dư, xúc tác Ni, thu được hỗn hợp D gồm ba chất đồng phân của nhau là D1, D2, D3 có công thức cấu tạo lần lượt là:

(D1)

(D2)

(D3)

a) Hãy xác định công thức cấu tạo của A. b) Viết công thức cấu dạng bền có thể có của D1 và D2. 3


Câu 8 (2,0 điểm): Xác định cấu trúc, đồng phân lập thể, danh pháp Khi xử lí chất A (C13H18O2) bằng dung dịch HCl loãng, thu được chất B (C11H14O) không quang hoạt. Khi B phản ứng với Br2/NaOH, sau đó axit hóa sản phẩm phản ứng, thu được chất C. Khi đun nóng B với hiđrazin/KOH trong etylen glicol, thu được chất D. Đun B với benzanđehit trong môi trường kiềm, thu được chất hữu cơ E (C18H18O) duy nhất. Khi bị oxi hóa mạnh, các chất B, C, D và E đều cho axit phtalic (axit benzen-1,2-đicacboxylic). Xác định công thức cấu tạo của các chất A, B, C, D và E. Câu 9 (2,0 điểm): Cơ chế phản ứng Cho các quá trình phản ứng sau: a)

COPh

Et

O Et

(1)

Ph

Ph

(2)

AlCl3 /xilen

COEt

AlCl3 /xilen

o

o

t , 48h H = 35%

t , 4h H = 90%

b) O

Br H

O

o

KOH/EtOH, t

Br

OH

O O

Dùng cơ chế để giải thích sự tạo thành sản phẩm trong các quá trình phản ứng trên. Câu 10 (2,0 điểm): Tổng hợp các hợp chất hữu cơ ( Dạng sơ đồ phản ứng). Cấu tạo của hợp chất K (tách từ quả hồi) đã được xác định theo sơ đồ phản ứng sau: K (C 7 H10O 5)

O3

Me2S

L (C 7H 10 O7 )

CH3OH

H

+

M

HIO4

N

H3O+

OHCCHO + OHCCH(OH)CH2 COCOOH

1. Hãy vẽ công thức cấu tạo của L, M, N và K, biết rằng K không chứa nhóm chức ancol bậc ba. 2. Hãy viết sơ đồ phản ứng tổng hợp ra K từ những hợp chất chứa không quá 4C. -------------------Hết--------------------------

4


HƯỚNG DẪN CHẤM ĐỀ ĐỀ XUẤT LỚP 11 Câu

Nội dung Điểm 1. Từ phương trình phản ứng: 2 H2O2 → 2 H2O + O2 ta có: thể tích oxi thoát ra trong 1 đơn vị thời gian tỉ lệ thuận với tốc độ phản ứng Theo các thí nghiệm 1, 2, 3 khi tăng gấp đôi thể tích dung dịch H2O2 và giữ nguyên thể tích của dung dịch KI thì tốc độ phản ứng tăng gấp đôi, điều đó có nghĩa là tốc độ phản ứng tỉ lệ thuận với nồng độ của H2O2 → phản ứng là bậc 1 đối với H2O2. Tương tự, từ các thí nghiệm 2, 4, 5 ta thấy tốc độ phản ứng tỉ lệ thuận với nồng độ 0,25 của I- → phản ứng là bậc 1 đối với I-. 0,25 2. Phương trình phản ứng: 2 H2O2 → 2 H2O + O2 0,25 Biểu thức của định luật tốc độ phản ứng: V = k . CH O .C I2

2

3. Khi pha loãng 3 lần thì nồng độ của H2O2 (C0) ở thí nghiệm 4 giảm 3 lần: → C0 = 10 gam H2O2/1 lit. Hay C0 =

Câu 1

10 = 0,294 M. 34

Vì phản ứng xảy ra chậm nên có thể coi như tốc độ phản ứng (thể tích oxi thoát ra) không thay đổi trong khoảng thời gian ngắn (4 phút). 0,25 Sau 4 phút sẽ thoát ra: 4,25 . 4 = 17 (ml) oxi, khi đó: n O2 =

P.V 1 . 17.10-3 = = 0,695.10-3 (mol) R.T 0,082 . 298

Lúc đầu có: n H O = 0,294 . 0,15 = 44,1.10-3 (mol). Sau 4 phút, số mol H2O2 chỉ còn: 44,1.10-3 – 2 . 0,695.10-3 = 42,71.10-3 (mol). 2

2

0,25

0, 04271 Vậy sau 4 phút: CH2 O2 = = 0,285 (M). 0,15

0,25

4. Phản ứng: -

I 2 H 2 O 2  → 2 H 2O + O2

v=−

(*)

1 d [ H 2O 2 ] 2 dt

Cơ chế: k1 H 2 O 2 + I-  → H 2 O + IO-

-

k2

-

IO + H 2 O 2 → H 2 O + I + O 2

(1) (2)

Xét 3 trường hợp: 1/ Nếu phản ứng (1) chậm và quyết định tốc độ thì tốc độ của phản ứng tổng hợp (*) bằng tốc độ của phản ứng (1): v=−

1 d [ H2O2 ] = k1[H 2 O 2 ][I - ] 2 dt

0,125

Cơ chế phù hợp với định luật tốc độ. 2/ Nếu phản ứng (2) chậm thì: v=−

1 d [ H 2O2 ] = k2 [H 2 O 2 ][IO- ] 2 dt

(a)

Chấp nhận nồng độ của IO- là ổn định ta có: d [IO - ] k = k1[H 2 O 2 ][I- ] − k2 [IO- ][H 2 O 2 ] = 0 → [IO - ] = 1 [I- ] dt k2

(b)


Thay (b) vào (a) ta được: v=−

1 d [ H2O2 ] = k1[H 2 O 2 ][I- ] 2 dt

0,125

Cơ chế phù hợp với định luật tốc độ. 3/ Nếu hai phản ứng có tốc độ xấp xỉ nhau thì: v=−

1 d [H 2 O 2 ] 1 = ( k1[H 2 O 2 ][I- ] + k2 [H 2 O 2 ][IO - ]) 2 dt 2

Chấp nhận nồng độ của IO- là ổn định, rồi tính [IO-] như ở trường hợp 2 và thay vào biểu thức trên ta được: v=−

1 d [ H 2O2 ] = k1[H 2 O 2 ][I- ] 2 dt

0,125

Cơ chế phù hợp với định luật tốc độ. Trong 3 trường hợp, trường hợp đầu hợp lí hơn cả vì ở đây không cần chấp 0,125 nhận điều kiện gì; mặt khác ở trường hợp 2, nếu đã giả thiết phản ứng (2) là chậm thì việc chấp nhận nồng độ của IO- ổn định là không hợp lí. 1. pHA = 1,50 → không cần tính đến sự phân li của nước Các quá trình xảy ra trong dung dịch A: H3PO4 ⇌ H+ + H 2 PO -4 Ka1 = 10-2,15 (1) CH3COOH ⇌ H+ + CH3COOH 2 PO -4 ⇌ H+ + HPO 24

Ka = 10-4,76 Ka2 = 10-7,21

(2) (3)

⇌ H+ + PO3Ka3 = 10-12,32 (4) 4 Vì Ka1 >> Ka >> Ka2 >> Ka3 nên pHA được tính theo (1): H3PO4 ⇌ H+ + H 2 PO -4 Ka1 = 10-2,15 [ ] 0,5C – 10−1,5 10−1,5 10−1,5 → C H PO = C = 0,346 M HPO 24

3

2. []

→ x = 5,49.10−5 M → αCH3COOH = 3. Tại pH = 4,00 ta có:

0,25

4

CH3COOH ⇌ H+ + CH3COO0,1-x 10−1,5 x

Câu 2

0,25

Ka = 10−4,76 5, 49.10−5 .100 = 0,055% 0,1

0,25 0,25

K 10 −2,15 [H 2 PO 4− ] [H 2 PO −4 ] = a1 = → = 0,986 [H 3PO 4 ] [H + ] 10 − 4 [H 2 PO 4− ] + [H 3PO4 ]

K a2 10 −7, 21 [HPO 24 − ] − = = = 10−3,21 → [ HPO 24 ] << [H 2 PO 4 ] − − 4,0 + [H 2 PO 4 ] 10 [H ]

0,25

[CH3COO − ] [CH 3COO − ] K a 10 −4, 76 = + = − 4,0 → = 0,148 [CH3COOH] [H ] 10 [CH 3COO − ] + [CH3COOH]

Tương tự:

10 −10,33 [CO32 − ] = << 1 → [ CO32− ] << [ HCO3− ]; 10 − 4 [HCO3− ] [HCO3− ] 10 −6,35 = << 1 → [ HCO3− ] << [CO2]. −4 10 [CO2 ]

Như vậy khi trung hòa đến pH = 4,00 thì chỉ có 14,8% CH3COOH và 98,6% nấc 1 của H3PO4 bị trung hòa, còn bản thân Na2CO3 phản ứng với H+ của 2 axit tạo thành CO2: 2 H3PO4 + CO32− → 2 H 2 PO 4− + CO2 + H2O

0,25


2 CH3COOH + CO32− → 2 CH3COO− + CO2 + H2O −3 n CO2- = 0,5.(14,8%. n CH3COOH + 98,6%. n H3PO4 )= 0,5.20.10 (14,8%.0,1+ 98,6%.0,173)

0,25

3

→ n CO2- = 1,85.10−3 (mol) → m Na 2CO3 = 0,1961 (gam) 3

0,25 +

2-

Ag(S2O3)23-

1+2. Ở điện cực trái : Ag + 2S2O3 0,001 0,10 0,098 0,001 23Do S2O3 rất dư nên [Ag(S2O3)2 ] ≈ 0,001(M) Xét cặp Ag(S2O3)23-/Ag: Ag(S2O3)23Ag+ + 2S2O32- - lgβ1 Ag+ + e Ag lgK1 Câu 3

Ag(S2O3)23- + e

Ag + 2S2O32- lgK2=lgK1.( - lgβ1)

0,25

0,25

E20 E10 = − lg β1 ⇒ E20 = E10 − 0,0592 lg β1 = 3,168.10− 3 (V ) 0,0592 0,0592 0

Vậy E Ag ( S 2O3 ) 32− / Ag =3,168.10-3 (V) Khi pin hoạt động:

AgCl + e Ag + 2S2O32AgCl + 2S2O32-

3. E A = E 20 + 0,0592 lg

0,25 Ag + ClAg(S2O3)23- + e Ag(S2O3)23- + Cl-

0,25

[ Ag ( S 2 O3 ) 32− ] 0,001 = 3,168.10 −3 + 0,0592 lg ≈ −0,055(V ) 2− [ S 2 O3 ] 0,098 2

EC = Epin + EA = 0,341 +(- 0,055) = 0,286(V) Xét điện cực phải: Ta có : AgCl + e Ag + Cl0 0 E AgCl / Ag = E Ag + / Ag + 0,0592 lg Ks AgCl

0,25

0 − 0 − ⇒ EC = E AgCl / Ag − 0,0592 lg[Cl ] = E Ag + / Ag + 0,0592 lg Ks AgCl − 0,0592 lg[Cl ]

⇒ lg Ks AgCl =

0 EC − E Ag + 0,0592 lg[Cl − ] + / Ag

0,0592

≈ −10

Vậy KsAgCl = 10-10 4. Ag(S2O3)23- + 2CNAg(CN)2- + 2S2O32- K = 107,54 0,001 0,01 0,098 8.10-3 0,001 0,1 + Ag(CN)2 Ag + 2CN Kkb= 10-21 C: 0,001 8.10-3 []: 0,001-x x 2x+8.10-3 x(2 x + 8.10 −3 ) ⇒ K kb = = 10 − 21 0,001 − x 10 −21.0,001 Coi x << 0,001 ⇒ x ≈ ≈ 1,5625.10 − 20 ( M ) << 0,001 (8.10 −3 ) 2

EA = E0(Ag+/Ag) + 0,0592lg[Ag+] =-0,373(V) Epin = 0,286 – (- 0,373) = 0,659 (V)

0,25

0,25 0,25


1. M + 4HNO3  → M(NO3)2 + 2H2O + 2NO2 (1) x 2x 2NO2 + 2NaOH NaNO3 + NaNO2 + H2O (2) 2x x x toC NaNO3  → NaNO2 + 1/2O2 5NaNO2+2KMnO4 +3H2SO4  → 5NaNO3 + K2SO4 +2MnSO4+ 3H2O (4) Từ phương trình 4 : n NaNO2= 5/2. n KMnO4 =0,25 mol Đặt nM=x từ các phương trình ta thấy nNO2 =2x nNaNO3=nNaNO2(2) =x Vậy 2x= 0,25 nên x=0,125 MM= 3/0,125 = 24(g/mol) vậy M là Mg 2. (sp3) (sp3) Nɺɺ

Câu 4

0,5

0,25 0,25

Nɺɺ

H

H F F H F Đều là tháp tam giác. Góc liên kết giảm theo chiều HNH > FNF vì độ âm điện của F lớn hơn của H làm điện tích lệch về phía F nhiều hơn. b. - NH3 có liên kết H liên phân tử còn NF3 thì không có, do đó amoniac có nhiệt độ sôi và nhiệt độ nóng chảy cao hơn NF3. - Số oxi hóa của Nitơ trong NH3: -3 Số oxi hóa của Nitơ trong NF3: +3 Như vậy, nguyên tử Nitơ trong amoniac với số oxi hóa thấp nhất dễ bị oxi hóa lên mức oxi hóa cao hơn. - NH3 ngoài tính khử còn có thể tham gia phản ứng cộng do khả năng tạo liên kết cho nhận của cặp e tự do trong nguyên tử Nitơ. Tuy nhiên, nguyên tử F có độ âm điện cao nhất nên có khả năng hút electron mạnh, làm giảm mật độ của cặp e tự do của nguyên tử Nitơ trong NF3. Do đó khả năng tạo liên kết cho nhận trong phản ứng cộng bị giảm mạnh. Khi cho dung dịch tác dụng với AgNO3 dư thu được kết tủa trắng, dễ bị phân hủy

0,25 0,25

0,25

0,25

→ Kết tủa trắng là AgCl.

nAgCl = 8.10-3 mol → Số mol Cl- tạo được kết tủa là 8.10-3 Đun dung dịch B với H2O2 trong môi trường kiềm thu được dung dịch C, cho C tác dụng với BaCl2 dư thu được kết tủa Y màu vàng, là đồng hình với BaSO4 → Y là Câu 5

BaCrO4 → A là tinh thể hiđrat của kim loại crom -3

0,25

-3

nBaCrO4 = 4.10 mol → Số mol kim loại crom = 4.10 mol nCr : nCl − = 1: 2

0,25

Đun nóng tinh thể ở 3000C, trong môi trường HCl thì thấy khối lượng giảm 40,52% → Trước khi bị phân hủy khối lượng giảm 40,52% → đó là toàn bộ nước trong A

Số mol H2O trong A = 0,024 mol → Số phân tử H2O = 6

0,25


- Theo đề bài m = 1,066 gam → Số mol A= 0,04 → M = 266,5 Công thức CrCl2.nH2O → n = 8 → vô lí - Ta thấy khối lượng H2O giảm ở 2 giai đoạn → có 2 loại H2O khác nhau (H2O 0,25 trong cầu nội, H2O trong cầu ngoại) Khi làm khô tinh thể A bằng H2SO4 đặc thấy khối lượng giảm 6,75 % → , đó là khối lượng nước trong cầu ngoại. Số mol H2O = 4.10-3 mol Số mol H2O trong cầu nội = 0,02 Thành phần A có Cr, Cl, H2O Tổng số mol Cl trong A = (1,066-4.10-3.52-0,024.18):35,5 = 0,012 mol

0,25

Tuy nhiên chỉ có 8.10-3 mol Cl- tạo kết tủa với AgNO3 → có clo trong cầu nội của A Số mol Cl trong cầu nội = 0,012-8.10-3 = 4.10-3 mol Công thức của A [CrxCly(H2O)z]Cla.bH2O

0,25

x : y : z : a : b = 0,004 : 0,004: 0,02: 0,008 : 0,004 = 1: 1: 5: 2: 1 Công thức của A là: [CrCl(H2O)5]Cl2.H2O [CrCl(H2O)5]Cl2.H2O → [CrCl(H2O)5]2+ + 2Cl- + H2O Cl- + Ag+

0,25

→ AgCl (X)

[CrCl(H2O)5]Cl2 + 2 AgNO3 → [CrCl(H2O)5](NO3)2 + 2 AgCl B 2[CrCl(H2O)5]

2+

-

+ 3H2O2 + 10OH → 2CrO42- + 2Cl- + 18H2O

0,25

C CrO42- + Ba2+ → BaCrO4 ↓ (vàng) (Y)

0

1.(0,75 điểm) Cấu trúc Imidazole (C3H4N2)

Câu 6

Thơm hay không thơm thơm

Imidazol-1-yl (C3H3N2)

anion

Imidazolyl (C3H5N2)

cation

thơm thơm

Oxazole (C3H3NO)

thơm

Thiazole (C3H3NS)

thơm

2. (0,75 điểm)


Tonc

Imidazole > Thiazole > Oxazole Imidazole lớn nhất do liên kết H liên phân tử. Thiazole lớn hơn oxazole do khối lượng phân tử và độ phân cực lớn hơn. Imidazole > Thiazole > Oxazole Imidazole lớn nhất do liên kết H liên phân tử. Thiazole lớn hơn oxazole do khối lượng phân tử và độ phân cực lớn hơn.

Giải thích Tos Giải thích 3. (0,5 điểm) Phản ứng ion hóa

Kb Lý do

Imidazole > Thiazole > Oxazole Axit liên hợp của imidazole có sự giải tỏa đối xứng, tạo liên kết H mạnh hơn với nước, do đó có lực bazo mạnh hơn oxadiazole và thiazole. O có độ âm điện lớn hơn N và S, nên làm giảm mật độ e trên N ở oxazole, làm giảm độ bền của axit liên hợp của oxazole nên lực bazo yếu nhất.

1. a (0,75 ®Óm). H3C

CH3 C

CH3 CH2

+

H

C

o,

CH3

t p

H3C

CH3 2-Metylpropan (Y)

2-Metylpropen (X)

CH3 H

CH3

C

C

C

H

H (A)

CH3

CH3

Bước thứ nhất gồm tương tác giữa hai phân tử trong môi trường axit: Câu 7 H 3C

CH 3 2 H 3C

C

CH 2

H+

CH 3 H

CH 3

C

C

C

CH 2

CH 3 H

H 3C

2,4,4-trimetyl-1-penten

CH 3 H

CH 3

C

C

CH 3

C

CH 3 2,4,4-trimetyl-2-penten


H3 C

CH3 H

CH3

C

C

C

CH2

+

H2

CH3 H

H 3C

Ni , t

CH3 H

CH3

C

C

C

CH3

+

o

CH3 H

CH3

C

C

C

CH3 H

H

H 3C

H2

CH3

CH3 .

b (0,75 ®Óm). R1

C

O CH2

O3

O

C CH2

R1

CH2 O

C

R1

Zn/H3O

+

R2

C

C

O

O O3

R2

CH3

H

C

CH3

R2

CH3

H

O

O

O C

O + O

CH2

CH3

Z CH3

C

R1

CH3

CH3

CH3

O

O

O

C O

C

CH3 CH3

H Zn/H3O

Q R2HC

+

O + CH3COCH3

2. (0,5 điểm) a. Công thức cấu tạo của A là: CH3 H3C

b. D1 và D2 có thế ở dạng cis hoặc trans, nên D1 có thể có cấu dạng bền: hay

H3CH2C

CH3

H3CH2C

CH3

trans-

cis-

D2 có thể có cấu dạng bền: CH2CH3 H3C

CH2CH3

hay H3C

trans-

cis-

(0,75 điểm) - Khi bị oxi hóa mạnh, các hợp chất B, C, D và E đều cho axit phtalic chứng tỏ các hợp chất này là dẫn xuất của benzen bị thế hai lần ở vị trí 1,2. B có độ không no k = 5 và có phản ứng bromofom nên B có thể là: Câu 8

CH2CH2CH3 COCH3 B1

C2H5

CH(CH3)2

CH2COCH3

COCH3 B2

CH3

B3

CH(CH3)COCH3 B4

(0,75 điểm) - Do B không quang hoạt nên loại B4. Đun B với benzanđehit trong môi trường

( ) (


kiềm, thu được chất hữu cơ E (C18H18O) duy nhất nên loại B3 do B3 ngưng tụ với benzanđehit tạo ra 2 sản phẩm hữu cơ khác nhau. C3H7

C 3H7

N2H4/KOH

COCH3

C 2H5

B

D

C 3H 7

PhCHO/OH

C 3 H7

-

COCH3

COCH=CHPh

B

E

(0,5 điểm) - Khi xử lí hợp chất A (C13H18O2) bằng dung dịch HCl loãng thu được hợp chất B chính là phản ứng thủy phân xetal. C3H7

C3H7 H3O+

CH3 A

O

OH

+ HO COCH3

O

B

Vậy A là một trong hai công thức dưới đây: CH2CH2CH3

CH(CH3)2

CH3 O

CH3

O

O

O

a) (1,0 điểm) Cl3Al R

1

Cl3Al

O

2

R

R

1

O

R

R1

2

R2

O AlCl3

R

chậm

R2

1

-AlCl3

X1

X2

b) (1,0 điểm) Câu 9

..O O

Br H

H

O

Br O

O

Br

-

Br OH

O

OH O

Câu 10

1(1,0điểm)

H

O

O

OH Br

Br O OH

O

OH O

O


COOH HO

COOH O3

HO

Me2S

HO OH K

COOH

COOH

COOH

HO O CHO

O O OH

HO

OH

O O

CH3OH H+

OMe

HO OH

OH L

HIO4

M

O O CHO OHC N

2. (1,0điểm) Viết sơ đồ phản ứng tổng hợp ra K từ những hợp chất chứa không quá 4C.

--------------------Hết ------------------

OMe


Turn static files into dynamic content formats.

Create a flipbook
Issuu converts static files into: digital portfolios, online yearbooks, online catalogs, digital photo albums and more. Sign up and create your flipbook.